96
הההההההה הה ההה הההההההה הה ההה ןןןן ההההה הההה ההההההההה הה הההה ההההה ה"ה הההה הההה1-4 הההה ההההההההה ההה ה"ה הההה הההה4-5 ההההה ההההה הההההה ה"ה הההה הההה5-6 הההההההה הההה הההההה ה"ה הההה הההה, ה"ה ההההה6-8 הההה הה הההההה הה' ההה ההההההה8-15 הההה ההההההה הההההה הההה' הההה15-18 ההההה ההההה ההההההה הההה' הההה18-22 ההההה ההההה ההההההה ה"ה ההההה22-25 הההההה ההההההה ההההה הההה' הההה ההההה25-28 ההה הההה ההההההההההה( ) ה"ה הההה הההה28-30 הההה הה ההההההה הההה' ההההההה30-32 ההההה ההההההCABG ה"ה הההה הההה32-38 ההההה הההההה ההה ההההה ה"ה ההההה38-42 ההההה הה הההה, ההההה ההה הההההה ה"ה הההה הההה42-45 ההההה הההההההה הההה ההההה הההה הההה ה"ה הההה הההה45-50 ההההה הההההההה הההה ההההה הההה הה הההה ה"ה הההה הההה50-52 ההההההה הההה ההההה ה"ה הההה הההה52-57 הההה הההה הההה ההההה ה"ה הההה הההה57-59 0

עינייםimg2.tapuz.co.il/CommunaFiles/39910353.doc · Web viewגם אם לאחר הפרוצדורה מתפתח PR, הלב הימני סובל את זה מצויין. נדיר

Embed Size (px)

Citation preview

Page 1: עינייםimg2.tapuz.co.il/CommunaFiles/39910353.doc · Web viewגם אם לאחר הפרוצדורה מתפתח PR, הלב הימני סובל את זה מצויין. נדיר

כירוגיית לב חזה

חזה לב כירוגיית תוכן

1-4ד"ר עוזי יזהרריאתי ממצא או מדיאסטנלי לגוש הגישה4-5ד"ר עוזי יזהרחזה לכירוגיית מבוא

5-6ד"ר עוזי יזהרניתוחי סיכון הערכת6-8ד"ר עוזי יזהר, ד"ר דביריודגשים חזרה – צינטורים

8-15דר' אלי מילגרטרמולדים לב מומי15-18פרופ' מריןמסתמים לניתוחי מבוא

18-22פרופ' מריןהמיטרלי המסתם מחלות22-25ד"ר רודיסהאאורטי המסתם מחלות25-28פרופ' אמיר אלעמיהעולה האאורטה ניתוחי

28-30ד"ר עוזי יזהר( ) פנאומוטורקס אויר חזה30-32פרופ' גלנווילאיסכמית לב מחלת32-38ד"ר עוזי יזהרCABG – מעקפים ניתוח38-42ד"ר רודיסהכושל בלב כירוגי טיפול42-45ד"ר עוזי יזהרניתוחי תוך ריאה, ניתור לב מכונת מהלך – ניתוח לאחר המודינמי ניתורתקין

45-50ד"ר עוזי יזהר

לא מהלך – ניתוח לאחר המודינמי ניתורתקין

50-52ד"ר עוזי יזהר

52-57ד"ר עוזי יזהרניתוח לאחר סיבוכים57-59ד"ר אורי רודסניתוח לאחר מדמם חולה

מאירי גליה ע"י נכתבבלמור ג'ינג'י רונן ע"י והוספות עריכה

2006- כרם עין הדסה – ירושלים – שישית שנה

0

Page 2: עינייםimg2.tapuz.co.il/CommunaFiles/39910353.doc · Web viewגם אם לאחר הפרוצדורה מתפתח PR, הלב הימני סובל את זה מצויין. נדיר

כירוגיית לב חזה

הגישה לגוש מדיאסטינלי או ממצא ריאתי

מדיאסטינום מדיאסטינום = חלל המיצר = חלל הביניים: חלל שגבולותיו הם הסרעפת )גבול תחתון(, הפלאורה המדיאסטינלית

)מלמעלה(, הסטרנום )מלפנים( ועמוד השדרה )מאחור(. thoracic inlet)בצדדים(, ה- בין הסטרנום לבין הפריקרד. מכיל בתוכו את התימוס. –מדיאסטינום קדמי

הפלקציות של הלב. כולל את הלב וכלי דם גדולים, הטרכאה והקרינה.2 בין –מדיאסטינום אמצעי בין הפריקרד האחורי לעמ"ש. כולל את הושט, אאורטה יורדת, חו"ש, –מדיאסטינום אחורי azygous vein-ו

thoracic duct.

הרחבה של צל המדיאסטינום בצילום חזה.SVC כפתור האאורטה וה-–צל המדיאסטינום נראה מעבר לגבולות התקינים

.PA ולא APלא לבלבל עם מריחה של המדיאסטינום בגלל צילום אבחנה מבדלת:

הגדלת בלוטות שפירה / ממאירה, לימפומה, תימומה, גידולי חו"ש, שערי הריאה. –תהליכים רקמתיים נוזל פריקרדיאלי )הגדלת צל הלב היא גם הגדלת מדיאסטינום(.–הצטברות נוזל .)תהליכים זיהומיים עם קולקציה של נוזל )נדיר אנוריזמה, דיסקציה. –תהליכים וסקולריים .)דימום מדיאסטינלי )טראומה

הסתמנות קלינית: ( סימני לחץcompression :)

oלחץ על הטרכאה או ברונכוס )סימפטומים נשימתייםo :כלי-דםsuperior vena cave syndromeo כאבים בבליעה, קושי בבליעה, ירידה במשקל.–ושט o למשל –עצבים rec laryngealשיגרום צרידות

:סימני האטיולוגיהoחולשה, ירידה במשקל, הזעות. לימפומה -o האם יש –תימומה myasthenia gravis? oSCLC – .תסמונת פאראנאופלסטית

מהחולים. 50%אסימפטומטי - ממצא מקרי, עד נותן הרבה אינפורמציה לגבי האנטומיהCT )לאחר אנמנזה כללית ומכוונת וצל"ח(. ה-CT scanהשלב הבא הוא

עם מתן חומר ניגוד כדי להבחין בין איברים דמיים לביןCTשל הממצא, ומיקומו ביחס לסביבה. עושים את ה- לימפומה נראית שונה מתימומה.– יכול מאד לכוון לאבחנה CTהתהליך החשוד. המראה של הממצא ב-

MRI )? עדיף לגבי ממצאים וסקולריים ולגבי עירוב של חוה"ש )שואנומה לוחצת לתוך הממצא. אם ביופסיה תחת הדמיה לא מצליחה ניתן לעשות ביופסיהCT-guided בדר"כ מבוצעת –ביופסיה

ב כאשר מתאים. טיפול בוחרים התהליך, של הפתולוגית האבחנה לאחר ,lymphomaפתוחה. thymoma מתחילים בהקדם בהקרנות.

Solitary Pulmonary Nodule זה שם שעבר זמנו. ע"פ הגדרה: קשרית"coin lession"קשרית ריאתית היא ממצא מאד שכיח בצילומי חזה.

לא רואים כלי דםCT מבנה שב – solid ס"מ. חלוקה נוספת היא בין 3 מעל – ס"מ, גוש ראתי 3 עד –ראתית מבנה אשר בתוכו ניתן לראות גם כלי דם ומבנה ריאה תקינה. למשל– non-solidומבנה ריאה בתוכו. לעומת זאת

.ground glassמראה ממוקם של

1

Page 3: עינייםimg2.tapuz.co.il/CommunaFiles/39910353.doc · Web viewגם אם לאחר הפרוצדורה מתפתח PR, הלב הימני סובל את זה מצויין. נדיר

כירוגיית לב חזה

קלסיפיקציהNatural historyגם קיים בין תהליך ממאיר לשפיר. אם אותו התליך ביותר באבחנה הוא הפקטור החשוב

בצילומים קודמים, והוא לא השתנה, זה מאד-מאד תומך בתהליך שפיר. ישנם גם ממצאים רנטגניים שיכוליםלתמוך בכך שהממצא שפיר.

הגישה הכירורגית לגרורה בודדת בחולה עם רקע אונקולוגי, הנחת העבודה היא שמדובר בגרורה. במקרה כזה, צריך הדמיה מקיפה כדי לברר שאין

גרורות נוספות בריאה או במקום אחר, ואין גידול ראשוני חדש. בחירת סוג ההדמיה תלויה במיקום הגרורות האופייני של הממאירות הראשונית.

לחולה, כדי להבטיח שאין עוד ממצאים גידוליים. לאחר בירור מקיף, אם מתבררPET scanמקובל גם לעשות ולא secondשיש רק קשרית יחידה בריאה היא חשודה לגרורה. חייבים להוכיח שהקשרית הזו היא גרורה

primary.ברורים לגבי הניהול שלguidelinedלאחר ביופסיה, אנחנו נמצאים בקטגוריה של "גרורה בודדת בריאה".אין

המצב הזה..cureתחת התנאים המתאימים, טיפול כירוגי מעלה סיכוי ל

התנאים לטיפול כירורגי בגרורה בודדת: גרורה בודדת באתר יחיד )ריאתית, כבדית, לפעמים גם מוחית(..1שליטה מוחלטת באתר הגידול הראשוני: אין שום טעם לנתח גרורה לפני שהסתיים הטיפול הכרורגי .2

והאונקולוגי בגידול הראשוני, כי אז סביר שיתקיימו עוד גרורות סמויות. מוחלט, ולכן אין טעם בניתוחcure מטרת הניתוח היא להשיג compele resection נגיש, –ממצא נתיח .3

, כי אז נצטרך כמותרפיה.complete resectionאם לא ניתן לסיים אותו בתוצאה של יציב, מסוגל לעמוד בניתוח. סיכון ניתוחי מתקבל על הדעת.–חולה נתיח .45.disease free interval –ככל שהוא יותר ארוך, הפרוגנוזה יותר טובה. אם החולה נקי ממחלה זמן קצר

בלבד ומצאנו גרורה, סביר שיש עוד גרורות שלא ניתן לזהותן. התוצאות טובות בסרטן קולון, כליה וסרקומה. התוצאות לא טובות בשד או מלנומה. –מקור הגידול .6כאשר אין טיפול יותר טוב. כל למשל בסרטן שד יש טיפול טוב אחר..7

יש הצדקה לתת גם טיפול כמותראפי משלים כדי לשלוט בגרורות שלא ניתן לזהותן, אם טיפול כזה קיים לסוג הממאירות ואם החולה לא קיבל טיפול משלים לאחר כריתת הממאירות המקורית. לפעמים ניתן טיפול כימי טרם הניתוח, כדי להרוג גרורות סמויות וגם להקטין את הגרורה הריאתית. אם לאחר הכמותראפיה עדיין קיימת רק גרורה אחת והיא עומדת בתנאים, נכרות אותה.תמיד קיימת האפשרות שלאחר הניתוח, עדיין יתפתחו גרורות

נוספות.ממאירות ראשונית של הריאה

הטיפול הוא לאSCLC, ובהם יש טיפול כירורגי. ב-NSCLC( החולים עם ממאירות ראתית הם 80%מרבית )כירורגי, זו מחלה של גרורות סיסטמיות, והיא קשה וקטלנית.

של הגידול קובע את הטיפולstage. ה-stage היא מה הNSCשאלת המפתח לגבי קשרית ריאתית ממאירה מסוג שלstage מגדירה את ה-TNM. כאשר קומבינציה של TNM מבוססת על שיטת stageשניתן לחולה. קביעת ה-

החולה: T1 –3 פחות מcmללא חדירה למבנים סמוכים T2 – 3 מעלcmללא חדירה למבנים סמוכים או לחלופים ביטוי של lobar atelectasisאו עירוב ה visceral pleaura.T3 –קרוב ל carinaפחות מ( 2cm.או חודר לדופן בית החזה )T4 –חודר ישירות ל mediastenumאו גורם ל pleural effusionעם תאים ממאירים. לחלופין מס' מוקדים נפרדים

באותה אונת ריאה.N1 – ipsilateral hilar –.בלוטות בתוך הריאה עצמה

N2 – mediastenal N3 –מחות למדיאסטנום או בלוטות בצד הגוף הנגדי M1 – ,גרורות אל: אונה אחרת של הריאה, מוח, עצמות adrenal.

2

Page 4: עינייםimg2.tapuz.co.il/CommunaFiles/39910353.doc · Web viewגם אם לאחר הפרוצדורה מתפתח PR, הלב הימני סובל את זה מצויין. נדיר

כירוגיית לב חזה

Stage 1&2 –-מחלה מוגבלת לריאה עצמה )כולל בלוטות תוך-ראתיות( הם חולים שיופנו לניתוח. חולים ב stage , ב-N2יותר מתקדם לא יופנו לטיפול ראשוני שהוא כירורגי, אלא לטיפול אונקולוגי. חלק מהחולים עם מחלת

stage 3a-יתאימו לניתוח אחרי טיפול נאו-אדג'ובנטי. ל ,staging!!! יש גם משמעות פרוגנוסטית עצומה NSCLS של stagingהגישה ל-

. CT בדיקת – Tהערכת , אבל כאן האמינות מוגבלת. קביעה דפנטיבית של מעורבות הבלוטה היאPET + בדיקת CT בדיקת – Nהערכת

ועדיין להיות רק ראקטיבית(. לכן, הקביעה של סטטוסPETרק בפתולוגיה )בלוטה יכולה להיות מוגדלת וקולטת ב-N היא, בעצם, רק לאחר הניתוח. זה בעייתי כי דרגת N-משוקללת בקביעה של ה stageולכן בקביעה האם לנתח

(,stage 2 )בלוטות הילאריות( אין לכך משמעות על ההחלטה הניתוחית )ממילא N1את החולה. בחשד קליני של מאד תומך, אבל לא מכריע( כיPET ביופסיה של הבלוטה ומידע היסטולוגי )חייבים, N3 או N2אבל בחשד של

מושפע מאד, ולכן גם הטיפול. stageה-CT + PET = שניהם שליליים N0 .שניהם חיוביים = מחשיד מאד לבלוטה נגועה ,

מחייבת ביופסיה. N2/N3 לא מחייבת המשך בירור. בלוטה N1בלוטה שהיא ניתוח בהרדמה כללית.– mediastinoscopyהביופסיה נעשית ע"י

של הגידול עצמו כדי לקבל בו"ז אבחנה היסטולוגית ו-FNAגישה אחרת היא לעשות מדיאסטינוסקופיה לפני stagingזה מבוסס על ההנחה שהבלוטה מייצגת את הגידול. אם תתקבל בלוטה ראקטיבית, לא תהיה ברירה .

לגידול הראשוני לשם אבחנה היסטולוגית של הגידול.FNAאלא לעשות גם -הPET גם אם הוא שלילי, לא ימנע את הצורך במדיאסטינוסקופיה ,

/CT כלל גופי מזהה את הרב המוחלט של הגרורות, וזו בדיקת הבחירה. לגרורות מוחיות צריך M – PETהערכת MRI-כי ה( PET .)קולט דיפוזית במח

של הגידול בריאה ? tissue diagnosisהאם חייבים stage, לכאורה ניתן לחכות לניתוח לקבלת דגימה היסטולוגית. )כאשר זה stage 1-2כאשר החולה נמצא קלינית ב-

חייבים ביופסיה, כי אף אונקולוג לא ייתן כמותרפיה ללא היסטולוגיה(. 3-4 האם זה ישנה את תהליך קבלת ההחלטות ? –העקרון: ההחלטה לגבי ביצוע פעולה רפואית )כלשהי(

כי ממילא חייבים ניתוח )לא נוכל לסמוך עלFNAאם יש ממצא מאד חשוד שממילא חייב לצאת, ניתן לוותר על כשיש חשד קליני גבוה(. ההבדל הוא בתכנון של הניתוח: סרטן מחייב כריתה אנטומית )FNAה- lobectomy,

pneumonectomy,בעוד שבממצא לא ממאיר ניתן להסתפק בכריתה אזורית. לכן, כשאין ביופסיה קדם-ניתוחית ) ולפי תוצאותיו מחליטים אם להרחיב את הניתוח.frosen sectionעושים כריתה אזורית ושולחים ל-

הרי שלאNSCLC, שאם היינו יודעים שאינו SCLCבהיעדר ביופסיה נוכל לגלות בדיעבד שנכרת ממצא שהוא שאין לו שום גרורות נלוות הניתוח מוצדק.SCLCהיינו מנתחים. עם זאת, ב-

ברונכוסקופיה יעילה רק אם יש חשד לעירוב תוך-ברונכיאלי. במקרים שבהם אין סימפטומים )כמו שיעול, ליחה,המופטיזיס( אין שום תועלת בברונכוסקופיה.

הטיפול הניתוחי. הכריתה צריכה להיות אנטומית )NSCLCכריתה מלאה )אם היא אפשרית( היא טיפול הבחירה לחולים עם

lobectomy, bi-lobectomy, pneumonectomy מאד לא מומלץ לעשות .)wedge resectionאבל לפעמים עושים , היא סיכון מוגברwedgeזאת כאשר לחולה אין מספיק רזרבה ראתית לכריתה אנטומית מלאה. הבעיה של כריתת

לחזרה מקומית כי לא כורתים את הניקוז הלימפתי של הגידול )מה שכן מושג ע"י כריתה אנטומית(. ואז דיסקציה אנטומית של האונה הריאתית שרוצים לכרות. צריךtoracotomyברמה הטכנית, הניתוח נעשה ע"י

לזהות את עורקי הריאה, וריד הריאה והברונכוס ולבודד אותם. חייבים ידע אנטומי מדוקדק ויכולת לדיסקציהעדינה בשערי הריאה וגם בסדק הבין-אונתי.

הערכה לפני ניתוח.)הערכה קדם-ניתוחית כללית )מחלות רקע ריאתי כריתת חלק מהריאה.–תפקוד נשימתית לאחר רזרבה לוודא שלחולה תהיה מספיק חייבים

. אם לחולה יהיה אחרי הניתוחFEV1ההערכה נעשית ע"י ספירומטריה פשוטה. הפרמטר החשוב הוא ה-FEV1>800ml .הוא לא יהיה נכה נשימתית

3

Page 5: עינייםimg2.tapuz.co.il/CommunaFiles/39910353.doc · Web viewגם אם לאחר הפרוצדורה מתפתח PR, הלב הימני סובל את זה מצויין. נדיר

כירוגיית לב חזה

הקדם-ניתוחי. זה מוטה כאשר התפקוד הריאתיFEV1 השארי היא אחוזית מתוך ה-FEV1הערכת ה- שמסופקת ע"יFEVפגוע ע"י הגידול, ואז צריך לעשות מיפוי ריאה כמותי כדי לראות את פרקציית ה-

החלק שעתיד להכרת. בחולים שהם מאד פגועים נשימתית או קרדיאלית, עושים מבחן מאמץ נשימתי.התפקיד של טיפול כמותראפי

, שהם לא מועמדים לטיפולstage 3/4טיפול כמותראפי הוא טיפול הבחירה לחולים שהם ב-טיפול אדג'ובנטי: לא קיבלו טיפול כמותרפי אדג'ובנטי.– שעברו ניתוח עם כריתה מלאה stage 1/2כירורגי. עד לאחרונה, חולים ב-

. )בהדסה מקובל כיום לתתsurvivalבשנה האחרונה התפרסמו עבודות מכובדות שהעידו על שיפור מסויים ב-(. Ib, IIa, IIbטיפול אדג'ובנטי בדרגות

, אבל ישנם כיום מחקרים שבודקיםstage 1/2כיום לא מקובל לתת טיפול נאואדג'ובנטי ב-טיפול נאו-אדג'ובנטי: יותר נמוך(stage, אולי יועיל גם ב-stage IIIa זה מועיל ב- –את ההצדקה של טיפול כזה )הלוגיקה

שלהם יורד, כןstage, שהם לכאורה לא כירורגיים, נותנים טיפול כמו-רדיותראפי. אם ה-stage IIIaבחולים שהם שולחים אותם לניתוח. התברר שניתן לשפר את הפרוגנוזה שלהם.

גילוי מוקדם לסרטן ריאה , עישון כבד.45 מעל גיל – במינון קרינה נמוך. זה מתאים לחולים בסיכון CTהבדיקה המוכחת היחידה היא סריקת

ברורים.guidelinesעדיין אין

מבוא לכירוגיית חזה

. )במקומות אחרים בעולם, זו התמחות בפני עצמה(.cardiothoracic surgeryניתוחי חזה הם תת-תחום בתוך מנתחי חזה עוסקים בחלל הפלאורלי, בפרנכימה הריאתית ובמדיאסטינום.

- עושים חתך צדדי, בכל מיקום בין ה-Thoracotomyניתוחי חזה מבוצעים ע"י midlineהקדמי לאחורי. לאחר החתך, עושים דיסקציה של השרירים עד לצלעות. נכנסים לחלל בית החזה בין הצלעות )מרחיבים את המרווח

הבין-צלעי, לפעמים שוברים צלעות(. - דרך חתכים קטנים מכניסים סיב אופטי לתוך החללThoracoscopyישנם מקרים שבהם הניתוח נעשה ע"י

הפלאורלי, ועושים ניתוח בטכניקות דמויות לפרוסקופיה. מהלך פוסט-ניתוחי בניתוחי-חזה

טיפול בכאב הכאב הוא סוגיה מאד-מאד-מאד מרכזית בניהול פוסט-אופרטיבי בניתוחי חזה. בטכניקה של טורקוטומיה יש הזזה

של לב לניתוחי )בניגוד קשה מאד להיות יכול הניתוח אחרי הכאב ולכן שלהן, ושבירה צלעות -midשל sternotomyמלבד הסבל של החולה, הכאב גם מפריע להחלמה מהניתוח - חולה כאוב לא יצא מהמיטה ולא .)

(. 4/10<יתאושש נשימתית. לכן, חייבים להשיג אנלגזיה טובה )הגדרה - כאב בניתוח לא מזוהם )ממאירות למשל(: בזמן ההרדמה לניתוח מחדירים גם צנתר אפידורלי שמחובר למשאבה

שמקומו נשאר האפידורלי הצנתר המשכי. באופן משככי-כאבים מונעת3-4שמזריקה לא האנלגזיה ימים. מהחולה להתהלך בחופשיות.

שמזריק מורפין לפי דרישת החולה )עם אוIV PCA( משיגים אנלגזיה ע"י empyemaבניתוח מזוהם )למשל ניקוז PCAבלי הזרקה בזאלית(. צריך לאזן את מינון המורפין כך שתהיה אנלגזיה אבל החולה לא יהיה מאד ישנוני. ה-

מושאר מספר ימים, לפי הצורך. התאוששות נשימתית

התחום המרכזי שאליו חייבים להתייחס: מחוץ למיטהעידוד לשיעול

משחקי נשימהפזיותרפיה נשימתית

ניטור ההמודינמימשמעותי בניתוחי-לב, לא מאד חשוב בניתוחי-חזה.

גישה כירורגית לזיהומים ראתיים white lung :אבחנה מבדלת -

4

Page 6: עינייםimg2.tapuz.co.il/CommunaFiles/39910353.doc · Web viewגם אם לאחר הפרוצדורה מתפתח PR, הלב הימני סובל את זה מצויין. נדיר

כירוגיית לב חזה

( לאחר כריתת ריאהpneumonectomy.בחלל שנוצר מצטבר נוזל הנראה כריאה לבנה .) :פלאורה - תפליט שגורם תמט

oHemothoraxoChylothoraxoאקסודטoטרנסודט

: פרנכימהoתמט ריאתי מלאoתסנין ענק

שלבים במהלך לא תקין של דלקת ריאות1.pneumonia2.para-pneumonic effusion .תפליט קטן, אקסודטיבי, ראקטיבי - 3.complicated pleural effusion:מוגדר מכמה גישות .

a..אנטומית: תפליט בחלל הפלאורלי ; ספטציותb. ; ביוכימית: אקסודטpH<7.2 ; Glu↓ ; LDH↑ ליזיס נויטרופילים( ; חלבון( ↑.c.מיקרוביולוגית: לרב מזוהם, אבל תרבית אינה גדלה )אם צומחת תרבית זו אמפיאמה, אבל יש גם

אפשרות שהתרבית לא תמצח כי החולה מקבל אנטיביוטיקה(.4.Empyema.)חלל מזוהם )במקרה זה - חלל פלאורלי -

a..אנטומית: כנ"ל + פלאורה פריאטלית מעובהb..ביוכימית: ממצאים כנ"לc.מיקרוביולוגית: נוזל מוגלתי / צביעת גראם חיובית או תרבית חיובית. הפתוגן הוא בדר"כ אותו

הפתוגן כמו המחולל של הפנאומוניה. אבחנה מבדלת של אמפיאמה:

( 60-70%פנאומוניה)s/p thoracotomy.טראומה

טיפול כירורגי בסיבוכי הפנאומוניהpara-pneumonic effusion .לא דורש טיפול. התפליט נספג מעצמו אחרי רזולוציה של הפנאומוניה :complicated pleural effusion .מצב ביניים. חלק יסתדרו בטיפול שמרני, אחרים יצטרכו ניקוז - empyema.מחלה כירורגית שמחייבת ניקוז -

ניקוי החלל הפלאורלי.מטרת הטיפול: התפשטות תקינה של הריאה.

(. המנקז בעייתי כאשר יש ספטציות, כי צריך נקז לכלCT guidedהמנקז מוכנס תחת הדמיה )שימוש במנקז חזי: חלל. ניתן לתת טרומבוליזה )סטרפטוקינאז( כדי לפרק את הספטציות. זו היתה פרקטיקה מקובלת, אבל לפני

עבודה ענקית שבה נמצא כי טרומבוליזה = פלצבו. המגבלה של העבודה -NEJMמספר חודשים התפרסמה ב-לא נעשתה הפרדה של הנבדקים עפ"י שלבי המחלה, ואמפיאמה אינה שווה לתחילת נרתוק של הנוזל.

drainage of pleural effusion & de-cortication :ניתוח שמטרתו לנקות את החלל הפלאורלי, הפרדה פיזית של הספטציות והוצאת הפיברין )דמוי ג'לי(. בנוסף, עושים דה-קורטיקציה, כלומר מקלפים את שכבת הפיברין שמצפה את הריאה )פלאורה ויסצרלית( כדי לנטרל את הבעיה הרסטרקטיבית. זה נעשה ע"י הפרדה בין מישור

הפלאורה לבין שכבת הפיברין. ההפרדה הזו עלולה לפגוע בפלאורה ולגרום דלף אויר )פנאומטורקס(. אבל בשיטה זו יש בעיה של ביצועthoracoscopy. ניתן לעשות open thoracotomyהניתוח בדר"כ מבוצע ע"י

נקזים פלאורליים. 1-3דה-קורטיקציה. בסיום הניתוח משאירים ואת מידת ניתן לראות את הקונסולידציה הניתוח מבוצע על ריאה שסובלת מפנאומוניה קשה. בזמן הניתוח

המחלה של הריאה. יש סכנה לא מבוטלת של גרימת בקטרמיה תו"כ ניתוח. בטיפול ממשיכים מלאה, לרזולוציה עד הפנאומוניה. של איטית( )די הדרגתית רזולוציה יש הניתוח לאחר אנטיביוטי, עושים מעקב חום וספירה לבנה. אסור לשכוח את האופציה של בקטרמיה. חשוב מאד לעודד את

5

Page 7: עינייםimg2.tapuz.co.il/CommunaFiles/39910353.doc · Web viewגם אם לאחר הפרוצדורה מתפתח PR, הלב הימני סובל את זה מצויין. נדיר

כירוגיית לב חזה

פזיותרפיה נשימתית. הנקזים מושארים עד שהם מנקזים ואז200cc/24hr<החולה לנשימה עמוקה, שיעול, , מוצאים בהדרגה.

הערכת סיכון ניתוחי

יום שאחריו, מכל סיבה30תמותה ניתוחית: תמותה במהלך הניתוח או ב-סיכון ניתוחי = תחלואה + תמותה

גורמי התחלואה הפוסט-ניתוחית העיקריים בכירוגיית לב חזה:CVAדימום)מדיאסטיניטיס )זיהום עמוק בפצע הניתוח

גורמי החולה בסיכון הניתוחי: גורמי סיכון לתמותה:

משפיע על תמותה ניתוחית. –גיל גורם סיכון מאד חזק לתמותה ניתוחית בניתוחי-לב. דרגה תפקודית –דרגה תפקודית מעידה על3-4

תפקוד לבבי מאד ירוד והיעדר רזרבות. כל חריגה ממהלך תקין עלולה להיות קטלנית. כאשר לחולה יש –תפקוד לבבי EFנמוך או חדרים מאד מוגדלים זה מעלה מאד את הסיכון לתמותה

ניתוחית בגלל העדר רזרבות להתמודדות עם הניתוח. כלייתי היכולת שלהן לשאת היפו-פרפוזיה במהלך–תפקוד כאשר מצב הכליות טרם הניתוח פגוע,

הניתוח או אחריו נמוכה ועלולה להתפתח איס"ק כליות. ככל שהניתוח יותר דחוף יש יותר סיכון לתמותה ניתוחית, כי הדחיפות מעידה על–דחיפות הניתוח

דרגות דחיפות: 3המצב הבסיסי של החולה )חולה יציב לא ילקח לניתוח חירום(. יש oEmergency – שעות מרגע ההחלטה. 24 הניתוח חייב להתקיים תוך oUrgent –.)הניתוח מתקיים במהלך האשפוז )לא יציב לשחרור oElective

גורמי סיכון לתחלואה: ( כרונית ריאות למשל( COPDמחלת הגמילה– הניתוח. לאחר משמעותית תחלואה לגרום צפויה

כלל(, הסיכון להתפתחות לגמול את החולה נוכל לא )ולפעמים להיות קשה יכולה ממכונת הנשמה פנאומוניה מאד גבוה. הסיבוכים הנ"ל עלולים לגרום לתמותה, בעקיפין.

גורם סיכון משמעותי לזיהום בפצע הניתוח, שיכול להיות עמוק. עלול להתפתח מדיאסטיניטיס–סוכרת שיצריך טיפול כירורגי ועלול להיות קטלני.

כמו סוכרת–השמנה גורמי הניתוח בסיכון הניתוחי:

מיומנות כירורגיתשימוש במכונת לב-ריאה)דחיפות הניתוח )דחוף - חולה לא יציב + ניתוח בלחץ( שילוב פרוצדורותCABG ,2+מסתם)מסתמים )ניתוח חוזר )הדבקויות, קשיים טכניים)אינדיקציה )דיסקציה חריפה > אנוריזמה של האאורטה, למרות שהניתוח עצמו זהה.משך הניתוח - בעיקר תחלואה )זיהומים(, ולא הגברת תמותה

cross-clamp time –.זמן בו אין זרימה בלב. לא אמור להגביר סיכון, כי יודעים לשמור על הלב הסיכון לתמותה ניתוחית בניתוחי לב:

:2-3%ניתוח מעקפים :4-6%מעקפים + מסתם :4%מסתם אאורטי

6

Page 8: עינייםimg2.tapuz.co.il/CommunaFiles/39910353.doc · Web viewגם אם לאחר הפרוצדורה מתפתח PR, הלב הימני סובל את זה מצויין. נדיר

כירוגיית לב חזה

:7%מסתם מיטרלי :5-8%שני מסתמים

הנתונים הם ממוצע התמותה לכל אוכלוסיית המנותחים. הסיכון יכול לעלות מאד או לרדת מאד בחישוב לחולההפרטני. ישנם מודלים ונוסחאות שמשקללים את כל גורמי הסיכון ונותנים אמדן של הסיכון הניתוחי.

חזרה ודגשים–צינטורים

מה ניתן להדגים בצינתור ? עורקים כליליים )אנומליות(.–אנטומיה היצרויות כליליות.–פתולוגיות תפקוד החדרים, לחצים, זרימות.–פזיולוגיה פתו-פזיולוגיה–( דלפים VSD, MR, AR חישוב ,)shunt.

מתבססים על נוסחאות פיק אשר קובעות:shuntלצורך חישוב Cardiac outputRt= )O2 consumption( / )Sat% pulmonary .v. – Sat% pulmonary .a.(COLt= )O2 consumption( / )Sat% Aorta – Sat% Vena Cava(

( נחלק את הנוסחאות זו בזו ונקבל:LtRt shunt ) אם אין .pulm .v דומה לזו של ה Aortaנניח שסטורציית CORt/COLt = )Sat% Ao - Sat%VC( / )Sat%Ao - Sat%PA(

.shuntע"פ אחוז זה נמצא את אחוז האת מי מצנתרים ?

טיפולי± צנתור אבחוני –מחלה כלילית ידועה או חשודה :חולה לפני ניתוח לתיקון מסתם

o הפיך ? קבוע ?–יתר ל"ד ריאתי o גיל, מין וגורמי סיכון ?–מחלה קורונרית נלווית

לפני ניתוח לתיקון מומים מולדים קשיםo( אנומליות קורונריותTGA, Fallot.)o.הדגמת מערכת כלי הדם הגדולים

אנטומיה קורונריתLt main circumflex marginals – מספקים קיר לטרלי של LV.

LAD perforants –מספקים את ה septum diagonals – קיר קידמי של LV

RCA PDA perforants –מספקים את ה septum pos lat branch –מספק את חדר ימין

acute marginals –נקראים כך שכן יוצאים בזוית חדה . כאשר ב:PDAדומיננטיות נקבעת על פי מי מספק את ה

RCA זהו ה 85%-).circumfex ) Lt coronary .a ה10%--5% codominant –.אספקה משוטפת מימין ומשמאל

ולכן נקראdiagonal ולא marginal זהו עורק ביניים, כלומר לא Lt mainאם יש עורק משני שיוצר מהפיצול של ramus medianus.

.obtuse marginal מכונה גם circumfex הראשון שיוצא מהmarginalהעורק הגישות וטכניקות בצנטור קורונרי

סוג הצנתור: צנתור שמאלי: עורק גדול )פמורלי, רדיאלי( ומהלך רטרוגרדי אל לב שמאל.צנתור ימני: כניסה בוריד גדול ומהלך אנטרוגרדי לכיוון לב ימין

מנחי שיקוף בצנתור:

7

Page 9: עינייםimg2.tapuz.co.il/CommunaFiles/39910353.doc · Web viewגם אם לאחר הפרוצדורה מתפתח PR, הלב הימני סובל את זה מצויין. נדיר

כירוגיית לב חזה

Rt Ant. Oblique - RAO – 45מעלות מימין. הצנתר )אחורי( בצד שמאל של התמונה. מדגים טוב את .LADפיצולי ה

Lt Ant. Oblique - LAO – 45-מעלות משמאל ל midlineצנתר בצד ימין. מדגים טוב את פיצולי ה .circumflex.

Caudal – מדגים טוב את פיצולי הסופיים של – מלמטה LADCranial – מלמעלה. מכונה גם spider שכן מדגים טוב את פיתולי ה LMAל LAD, circumflex.

וכן היכן אבי העורקים )ע"פ הצנטר ולזהות את המנחים נתבונן היכן עמוד השדרה ביחס ללב כדי להתבונן שבתוכו(

ביחס ללב. בצנתור: LADזיהוי .מגיע עד לאפקס-ביחס ל( עורק קדמיcircumflex)-90יציאת סעיפים ספטליים ביציאת סעיפים דיאגונליים

הזרקה לחדר שמאלכיום זו הזרקה נדירה, כי ניתן להשיג את רב המידע באקו.

AS –מדידת לחצים. גם במידה שמודדים לחצים בצנתור, לא מקובל להזריק חומר ניגוד כי זה חדר עם diastolic dysfunction-מ"ל( תגרום לעליה70 קשה והיענות נמוכה מאד, ולכן הנפח של חומר הניגוד )כ

והחולה עלול לפתח בצקת ריאות קשה.LVEDPניכרת ב-VSD –הערכת דלף MR –מדידת דלף

שעבורו גםLMA בשני מנחים. למעט 50%< במבט אנגיוגרפי אחד, או מעל 70% –חסימה קורונרית משמעותית במנח יחיד היא משמעותית. 50%חסימה של

מומי לב מולדים

מומים חדשים בשנה בישראל(.1000 לידות יש מום לב )1:100ב- ,עוברי אמורלב אקו שכיחות הולכת ויורדת. קשים וה-לבמומיתינוקות עם יש יותר הפלות של בשנים האחרונות

ילדים. ללכאורה, לב בכירורגית הצורך את יש, בפועלבטל וכן מומים לפספס וניתן מושלמת לא הבדיקה ומיעוטים( שלא מפסיקים הריון למרות המום )כ-אוכלוסיות )בעיקר דתיים מההריונות נמשכים למרות60%

(. המום )בעיקרTORCH, סכרת הריונית, זיהומי SLEהאטיולוגיה העיקרית של מומי-לב היא גנטית. גורמי סיכון נוספים

Rubella)כבר לא ממש רלוונטי ,,Ductusמומים כמו ASD -ו VSDקטן, לא מאובחנים פעמים רבות מיד לאחר הלידה לעומתם יש מומים.

)מומים כחלוניים(. שמתגלים מיד לאחר הלידה לעיתים צריך לעשות גם, אך רוב האבחנות כיום נעשות באקו (Coarctation of the Aortaהרחבה של חלק ממקרי למשל:לעיתים הצנתור הוא טיפולי )צנתור אבחנתי.

שיסגרו לבד, מום )שכיח( כמוPDA או ASD מהמומים אינם מצריכים טיפול. דוגמאות למומים כאלו הם 40%congenital bicuspid aortic valve 40 שמתגלה רק בגיל ..

להבחין בין ניתוח פליאטיבי ודפניטיבי. סגירה של דוקטוס היאשאר מומי הלב מחייבים טיפול )לרב ניתוחי(. צריך טיבי.פליאשהוא tetralogy of Fallotילד עם תסמונת , לעומת שאנט )זמני( בטיפול דפניטיבי

נותרות הפרעות מבניות בלב,עדיין (. שכן cureבמקרים לא מעטים גם הטיפול ה"דפנטיבי" אינו ריפוי מלא ) 90%כיום ב- )או כאשר התינוקת שעברה תיקון תהיה בהריון(. בסה"כ, בעשורים מתקדמיםויתבטאהן ולעיתים

או פליאציה טובה. אך חייבים להישאר במעקב ותמיד יתכנו בעיות בהמשך.cureמשיגים מהניתוחים

8

Page 10: עינייםimg2.tapuz.co.il/CommunaFiles/39910353.doc · Web viewגם אם לאחר הפרוצדורה מתפתח PR, הלב הימני סובל את זה מצויין. נדיר

כירוגיית לב חזה

מומים לא-כחלוניים( דלף משמאל לימיןLt to Rt shunt :)VSD, ASD, AV canal, PDA, truncus, TPAVD מומים בעורקים הגדולים– vascular ring / sling - מומים חסימתייםaortic coarctation, cong AS, cong PS.

Ventricular Septal Defect - VSD VSD(. סוגים אחרים של membraneous septum )פגם שנושק ל-peri-membraneous VSDהסוג הכי שכיח זה

.muscular, inlet, outlet ASDsהם פתופזיולוגיה

סגורים והעליות מתמלאות. בדיאסטולה הדם מועבר לחדרים. לחדר שמאל ישAVבסיסטולה שני המסתמים פחות היענות מחדר ימין, והלחץ בו יותר גבוה, וכך נוצר דלף משמאל לימין בסיסטולה הבאה. בנוסף, חדר ימין

נפח מהרגיל ולכן הריאות מוצפות בדם. 2-3מקבל נפח דם גדול ומזריק פי קליניקה

בגיל גבוהה. הריאתית התנגודת כי מתבטא לא הפגם לחיים הראשונים ירידת4-6בשבועות עם שבועות, 10אוכל , מתעייף מאד באכילה ואי-עליה במשקל )הילד בנשמתהתנגודת, מופיעים סימפטומים של איס"ק לב:

פונטנלות בולטות. יש, ובצבירת מים בכבדגם . אי ספיקת הלב מתבטאת (מזיע ולא רוצה עודמתנשם ול ומיד "מ סימני גודשבתינוקות, לא רואים בצקות כמו במבוגרים ואין הרבה קרפיטציות בריאות. דלקת ריאות לעיתים.

ריאתי: הילד מתעייף בכל נסיון אכילה, מתנשם, לא עולה במשקל.(.VSDבבדיקה ניתן לשמוע אוושה )עוצמת האוושה ביחס הפוך לגודל ה-

אם הפגם חמור ולא מתוקן בזמן מתפתחת תסמונת אייזנמנגר - יתר ל"ד ריאתי עם היפוך של השאנט והיווצרותמצב כחלוני )שאינו הפיך(. בשלב זה האוושה נעלמת / נחלשת.

טיפולVSD-קטן ואסימפטומטי לא מחייב סגירה ומחייב רק מניעה ל SBEמומים גדולים צריך לסגור בניתוח. לא רוצים .

מצד שני לא רוצים גם לעקוב אחרי הילד עד שהוא יעשה כבר יסגרו בעצמם. VSDלנתח מוקדם מדי, כי חלק מה-יזנמנגר. כיום, ניתן להעריך את המצב ע"י האקו.יפתח תסמונת אי

זה שהתפרים לאVSDהדבר החשוב ביותר בתיקון של ה- בניתוח תופרים טלאי של פריקרד שמכסה על הפגם. אם לא, אבל ואז הילד צריך קוצב. הניתוח הוא לא קלComplete heart blockיהרסו את מערכת ההולכה ויהיה עושים נזק יהיה לרוב ריפוי מוחלט.

, כלומר לשים טבעת על ה-PA banding מאד סימפטומטי ולא ניתן לתיקון ביילוד, ניתן לעשות VSDכאשר ה-pulmonary artery .כדי להגביל את זרימת הדם לריאה ולהפחית את ההצפה

התוויות לניתוח:פגם בoutletבכל גיל ילד סימפטומתי בכל גיל 6-12ילד בגילmo עם התפתחות pulmonary HTN 24ילד מעל גילmo לכיוון של זרימה ריאתית עודפת.2:1 בו יחס הזרימה במוצאי החדרים עולה על

Atrial Septal Defect - ASDASD חור במחיצה בין העליות שגורם לביטול הפרש הלחצים בין העליות. יחס = F:M 3:1 של.

. 0-5mmHg הלחץ הוא RA וב- mmHg 6-12 הלחץ הוא בין LAבמצב נורמלי ב- ASD secundum – 80% -מה ASD-חסר של רקמה באזור ה .Fossa ovale.ASD primum -נראה כחלק מ - AV canalCommon atrium –חסר שלם של המחיצה Sinus venosus ASD -חסר עליון היכן שה - SVC נכנסת לעליה ימנית היא מחלה אסימפטומטית. האבחנה נעשית ע"י בדיקה פיסיקלית. כאשר נמצא פיצולASD מהמקרים 99%ב-

relativ, אוושה סיסטולית של pulmonic ejection click מודגש, P2רחב של קול שני ממצאים אפשריים נוספים: PS שכן דרכה עובר נפח רב. בצורה דומה תתכן גם אוושה דיאסטולית בשל relative TS.

9

Page 11: עינייםimg2.tapuz.co.il/CommunaFiles/39910353.doc · Web viewגם אם לאחר הפרוצדורה מתפתח PR, הלב הימני סובל את זה מצויין. נדיר

כירוגיית לב חזה

דלף של דם מהעליה השמאלית לעליה הימנית בגלל הפרש הלחצים )הלחץ בעליה ימין יותר נמוך(. פתופזיולוגיה: התוצאה היא הגדלה של העליה הימנית )בגלל עומס הדם(. בהמשך תהיה הגדלה של חדר ימין )בשל עומס נפח(

וכן הגדלה של עליה שמאל ) שמנסה לספק נפח מספק לחדר שמאל(. קשורים בעיקר בהגדלת עליה ימין והתפתחות אריתמיות עלייתיות )פלפיטציות(. בדר"כ אין איס"ק סימפטומים:

לב משמעותית כי הפרשי הלחץ לא גדולים. ניתן בדר"כASD secondum קטן יכול להסגר מעצמו. ASDבדר"כ לא מתקנים את המום בגיל צעיר, כי הטיפול:

זהו ניתוח שעובר כמעט ב-סגירה ראשונית או ע"י טלאי של פריקרד.לסגור בצנתור. אם צריך ניתוח, עושים הצלחה. 100%

PFO )Patent foramen Ovale( שכיחות( הוא יישאר סגור בשל הלחץ בצד שמאל PFO = 'אלא20-30% באוכ ) .RtLt shuntאם במהלך החיים מסיבות שונות יווצר יתר לחץ דם בצד ימין של הלב ואז דרכו יווצר

Complete A-V Canal ביטוי של הלב. מרכז בהתפחות ,ASDבעיה VSDו מום שאופייני מסת ילדיבם אטריו-ונטריקולרי משותף.

חודשים כי אחרת4-6פני גיל תסמונת דאון. הטיפול הראשוני הוא תרופתי תומך, אבל חייבים תיקון כירורגי ל בניתוח סוגרים את פגמי המחיצה ע"י טלאי ומחלקים את המסתם המשותף.יתפתח סנדרום ע"ש איזנמנגר.

התיקון מאד קשה ובדר"כ לא מושלם.Patent Ductus Arteriosus - PDA

משמש בעובר לעקיפת מחזור הדם הריאתי )שאינו דרוש(. הדוקטוס אמור להסגר בתוךductus arteriosusה- זה לא תמידבפגים הוא חמצון תקין, והגירוי לסגירה של הדוקטוסיום-יומיים. לפעמים זה לא קורה, בעיקר בפגים.

ונגרמת אי סםיקה נשימתית.הדוקטוס מציף את הריאותמתחילה לרדת, התנגודת הריאתית מתקיים. כאשר ע"י הדוקטוס לסגירת להביא ניתן פירוק indomedבדר"כ )הגברת PGבחלק פתוח. הדוקטוס את ששומר

יכולה להיעשות בצנתור שמחדיר לכלי סלילPDAיש צורך בטיפול מכאני. סגירת זה לא עוזר ואז מהמקרים בניתוח צריך להיזהרהמעודד יצירת קריש דם במקום או בניתוח קל יחסית )טורקוסקופיה, ללא מכונת לב-ריאה(.

. recurrent laryngeal nerveלא לפגוע בואגוס וב- Aorto-Pulmonary windowחיבור עקב לימין ה-דלף משמאל בין ascending aorta-ל PA( ורק מעליהם מתרחשתקינים שני מסתמים

בגיל הולך לרוב הביתה ללא אבחנהבדר"כ . התינוק (החיבור וחולה, אבל רזה כי חודש הוא מגיע התנגודת ספיקת לב. בהאזנה יש איוושה ממושכת שממשיכה גם-אימתפתחת מציף את הריאות. ה ודם הריאתית ירד

בדיאסטולה. האבחון הוא באקו. בניתוח יש להפריד בין האאורטה והעורק הפולמונלי.Vascular Ring

וריאציות אנטומיות שונות. 20אנומליות של קשת האאורטה או של עורקי הריאה. יש מעל פתופזיולוגיה: - התפצלות לאאורטה אחורית וקדמית שמתחברות אח"כ באאורטהdouble aortic archהדוגמא הקלאסית היא

סביב הטרכאה והוושט ולוחץ עליהם. עיקר הלחץ הוא על הקנה, וישvascular ringהיורדת. כלי הדם יוצר מעין tracheomalacia .

ולא עולה במשקל. יש סטרידורים חודשים הוא לא אוכל מוצק4-6בגיל , אבל התינוק נולד ונראה תקין קליניקה:וושיעול נבחני )הלחץ הפולסטיבי על הטרכאה וההיצרות יוצרים אקספיריום ממושךגם לעיתים air trapping

. על פני הלב לא שומעים כלום. בריאות( )בליעת בריום שמדגימה כיווץ בוושט שלא משתנה בין צילום לצילום(. esophagograph-קה ויקלינ אבחנה: ככל שעושים את הניתוח מוקדם יותר הניתוח קל יותר ויש פחותניתוק הטבעת ופתיחת ההיצרות. הטיפול:

זמן השיקום לאחר הניתוח תלוי במהירות האבחון.טראכומלציה.Coarctation of Aorta

Coarctation( היצרות של האאורטה, מול חיבור הדוקטוס - aortic isthmus .). CHD, M:F=2-5:1 מכלל 5-6%השכיחות: מורפולוגיה

או להופיע בשילוב מורכב של מומי-לב. כאשר יש מוםVSD, להופיע יחד עם isolatedהקוארקטציה יכולה להיות משולב, הגילוי יהיה מוקדם יותר בתקופת הינקות בשל אס"ק לב קשה.

10

Page 12: עינייםimg2.tapuz.co.il/CommunaFiles/39910353.doc · Web viewגם אם לאחר הפרוצדורה מתפתח PR, הלב הימני סובל את זה מצויין. נדיר

כירוגיית לב חזה

)מוגבלת לאזור מול חיבור ה- discreteהמורפולוגיה יכולה להיות ductus או שתתקיים )tubular hypoplasia וגם חלק גדול מהקשת(.isthmus)עירוב של ה-פתופזיולוגיה

פולשתductus כלומר שרקמת ductusאם עושים דגימת רקמה מאזור הקוארקטציה, התשובה תתקבל כרקמת לאאורטה היורדת, וכאשר הדוקטוס נסגר )בתגובה לחמצון( נוצרת היצרות. עם זאת, יש מקרים שכבר ברחם

רואים את הקוארקטציה, כך שחייבים להיות מנגנונים נוספים. VS. אם יש גם coarctation ובאאורטה עד ל- LV גדול לחדר שמאל ולכן לחץ גבוה ב- afterloadההיצרות עושה

D -השאנט דרכו יהיה מוגבר כי הלחץ בחדר שמאל גדל בשל ה ,coarctationזה גורם לכך שריאות מוצפות ויש . החמרה ניכרת באס"ק לב.

בנוסף, האאורטה דיסטלית להיצרות אינה מקבלת דם כראוי, ולכן פלג הגוף התחתון אינו מקבל פרפוזיה. כמנגנון (. בדר"כ המעקףcoarctation כדי לתת אספקת דם לחלק הגוף התחתון )דיסטלית ל-colateralsהגנה, מתפתחים

שיוצא מה- internal mammary )thoracic( arteryהוא ע"י subclavian artery -פרוקסימלית ל coarctation. שיוצאים מעורק זה, והעורקים הללו גדלים ומתנפחים )הופכים להיותintercostalsמתרחש היפוך בכיוון זרימה ב-

בחלק התחתון של הצלעrib notchingגורם סיכון לדמם משמעותי בניתוחי חזה, למשל(. בעבר ניתן היה לראות (. notching הגדולים )כיום האבחון מקדים את ה-intercostalsבצל"ח, משני ל-

קליניקהלחיים, לעיתים המום חמור מאוד ומתגלה ביום הראשוןקוארקטציה היא ספקטרום של הסתמנות - במקריםו

.לשכת גיוסבדיקה בבאחרים המום מתגלה רק בילדות מאוחרת או אפילו המשמעות ההמודינמית של הקוארקטציה תלויה בקצב סגירת הדוקטוס, חומרת ההיצרות, התפתחות קולטרלים

ובקיום וחומרת מומים נוספים.עם נשמתמספר ימיםבן ים קשים, ההסתמנות היא של תינוק במקר ,חדר שמאלי היפרטרופי, גודש ריאתי,

הילד "מנסה למות".ברגליים. יש קצת אצידוזיס. קרה וללא דפקים פריהיפרשמטרתו לשמור את)Prostacyclin )PGE2הפתרון במקרה כזה הוא התחלה מיידית של טיפול של טיפול ב-

הדוקטוס פתוח. הדוקטוס משמש כמעקף של אזור ההיצרות. הפרוסטציקלין גם פותח את ההיצרות עצמה, כי היא עשויה מרקמה היסטולוגית של דוקטוס. כמו-כן, צריך להנשים את התינוק. לפעמים גם ניתן ביקרבונט כדי

לשפר את מצב האיברים הסובלים. השלב הבא הוא התערבות כירורגית. הקליניקה של קוארקטציה, גם אם היא קשה, לא תתבטא בימים הראשונים לחיי התינוק, כי הדוקטוס עדיין

פתוח. אם לא נבצע בדיקת יילוד קפדנית, הוא עלול להשתחרר לביתו, ולהגיע למיון כשהוא "מנסה למות". בבדיקת היילוד חשוב להקפיד על בדיקת דפקים פריפריים: הדפקים הפמורליים יהיו מוחלשים או חסרים בנוכחות

(.Left subclavianקוארקטציה )דפקים פריפריים בידיים יהיו תקינים, כי לרב ההיצרות היא מתחת למוצא ה- בין דופק רדיאלי ובין דופק פמורלי. ניתן גם למצוא בבדיקה יתר לחץ דם )בילדים, ל"ד שחורגdelayבנוסף, יש

עד שלא הוכח אחרת. Coarctation הוא בדר"כ אורגני(. בקיום ממצאים אלו מדובר ב - normogramsמה- קוארקטציה קלה מתגלה בדר"כ בילדים גדולים, שהם בדר"כ אסימטומטיים וללא בעיות התפתחות. ההסתמנות אצלם נובעת מגודש דם בפלג הגוף העליון )כאבי ראש, דימום מהאף( ואיסכמיה יחסית של פלג הגוף התחתון

בתרשים האק"ג. LVH בזמן מאמץ(. בדר"כ יהיה ל"ד גבוה בגפיים העליונות ועדויות ל-claudication)כאבי ". reversed 3 או סימן "rib notchingכיום נדיר לראות

.50 מהחולים יגיעו לגיל 10% הוא שרק natural historyקוארקטציה לא מטופלת היא מחלה קטלנית. כאשר ה-ל- קורונרית משנית מחלה במוח, דם המוות: שטפי ההיצרות,endoarthritis, התפתחות HTNסיבות באזור

התפתחות מפרצת שנקרעת ועוד. טיפול ניתוחי

כיום מקובל לתקן כירורגית את האאורטה מוקדם לאחר האבחנה )הסיכון הניתוחי לא גבוה ומולו עומדים סיבוכים)גם ניתן לעשות תיקון בשלב אחד - נלווים שנוי במחלוקת יל"ד(. העיתוי של תיקון מומים ארוכי טווח של

של ה-banding, עושים VSDהקוארקטציה וגם המומים הנוספים( או תיקון בשלבים )למשל, במקרה של PA בזמן תיקון הקוארקטציה ומתקנים את המחיצה במועד אחר(.

ואז הגישהLt thoracotomyהניתוח נעשה בגישה של גם מום תוך-לבבי, רוצין לתקן )למעט במקרים בהם .mid-sternotomyהכירורגית היא ע"י

11

Page 13: עינייםimg2.tapuz.co.il/CommunaFiles/39910353.doc · Web viewגם אם לאחר הפרוצדורה מתפתח PR, הלב הימני סובל את זה מצויין. נדיר

כירוגיית לב חזה

עקרונות בתיקון כירורגי של קוארקטציה: כריתת האזור המוצר ואזור הדוקטוס במלואם.-מוביליזציה מספקת של האאורטה למניעת תפרים תחת מתח. -.rec laryngeal nerve וה-intercostal arteriesהגנה על ה--שאיפה להמנע מגופים זרים )שתלים(, במידת האפשר-שמירה על פוטנציאל הגדילה של האאורטה.-תיקון פוטנטי, ללא מפל לחצים לאחר הניתוח. -

: coarctationטכניקות כירורגיות לתיקון 1.end-to-end anastomosis-ומתחת ל - חסימת האאורטה מעל coarctation-וה , הסרת האזור המוצר

ligamentum וחיבור , end to endשל האזורים מעל ומתחת להיצרות. מחייב מוביליזציה עדינה של קו ביילוד, ולתפור אותם. לקרב את הקצוות כדי ההיצרות, לאזור ודיסטלית פרוקסימלית האוארטה

. לכן, פתרון30%התפרים גוזל בעצמו חלק מנפח החיבור ולכן הסיכוי להיצרות מחודשת באותו מקום כ-זה טוב לילדים הצעירים )בילדים גדולים המוביליזציה קשה(.

Extended end-to-end anastomosis מקטעים בקוטר2 - כריתת מקטע ארוך של האאורטה וחיבור של גדול. סיכויי החזרה קלושים ביותר. טכניקה זו אפשרית ביילוד.

2.subclavian flap aortplasty-השיטה המועדפת ביילוד. מנתקים את ה - Lt Subclavian arteryפותחים , אותו לרוחבו, ומששתמשים בו להרחבה של אזור ההיצרות. נותרת קשת אאורטה עם שני עורקים ולא

)לרב יווצרו קולטרלים, ואפילו יתכן דופק(. סיכוןsubclavianשלושה. היד בדר"כ לא ניזוקה מהקרבת ה-. 5%החזרה הוא כ-

3.synthetic graft דקרון או( כריתת אזור ההיצרות והחלפתו ע"י שתל מלאכותי - gore-texמבוצע בעיקר .) בילדים גדולים או במתבגרים - אצלם המוביליזציה של האאורטה קשה ואין את הבעיה של הצורך בגדילת

האאורטה עם גדילת הילד. Subclהמעקף המקובל הוא לטפל בקורארקציה. במבוגר, בניגוד לילד, ניתן לעשות מעקף להצרות במקום .4

avian to Descending Aorta.סיבוכי הניתוח

: מהאאורטה יוצא העורק ע"שהשדרה עקב סגירת האאורטהחוט הסכנה העיקרית בניתוח היא איסכמיה של Adamkievich-שמתחבר אל ה ,anterior spinal arteryשמספק את חוט השדרה. בכל טכניקה ניתוחית נדרש

cross clamp פרה-פלגיה. סיכון של 0.4% דקות ולכן יש 15-20 למשךהדרכים לצמצם את הסיכון:

צמצום זמן איסכמיה למינימום האפשרי. .1 )מדידה רקטלית( מעלות34ל- הורדת חום הגוף .2ר בשדה הניתוחי(.שופכים נוזל קקירור מקומי של חוט השדרה ).3 על חוט השדרה.וסטרואידים - הגנהקוקטייל של הפרין, מניטול, לידוקאין .4את לחץ הפרפוזיה של חוט השדרה.כדי לשפר CSF והוצאת LPביצוע .5

בלבד )mild coarctationמבוגר יחסית עם חולה הוא המקרה הכי קשה לתיקון והכיאין לו קולטרלים( כך ש אז אין בעיהיש לרוב הרבה קולטרלים וקריטית, היצרות אם יש לאותו חולה לא לנתח אם ניתן.מסוכן. לכן, עדיף

)הקולטרלים יגנו על חוט השדרה. לנתחסיכונים נוספים של הניתוח:

דמם מאסיבי פגיעה בכליות בגלל איסכמיה-נזק לrec laryngeal nerve שגורם צרידות

תסמונת הורנרChylothorax -נזק מקומי ליד בגלל הקרבת הsubclavian.

בקוארקטציה קשה )לפני תיקון( המעי רואה זרימת דם למינרית )דם עובר את ההיצרות בסיסטולה ובדיאסטולה . לכן, לא מזינים אתparalytic ileusגם יחד(. אחרי הניתוח יש זרימה פולסטילית והמעי "מתבלבל" ומגיב ע"י

ולכן מקובלNEC מעלים את הסיכון של umbilical artery & vein lineהתינוק עד שיש פעולת מעיים. בנוסף - לא להזין פרנטרלית עד להוצאת הצנתרים.

12

Page 14: עינייםimg2.tapuz.co.il/CommunaFiles/39910353.doc · Web viewגם אם לאחר הפרוצדורה מתפתח PR, הלב הימני סובל את זה מצויין. נדיר

כירוגיית לב חזה

(. תמותה מאוחרת משתנה מאד כפונקציה של מורכבות וחומרת5%<התמותה הניתוחית היא נמוכה ביותר )(. 5yS 40-95%המומים הנלווים. )טווח של

היא re-coarctationהשכיחות הממוצעת של )תלוי במידת ההיפופלזיה של הקשת ובטכניקת התיקון(.5-20% רה-קוארקטציה מתרחשת בדר"כ בשנה הראשונה לאחר הניתוח. במקרים רבים ניתן לטפל בהיצרות החוזרת ע"י

יזדקקו להתערבות צנתורית(.20%צנתור. )כאשר הניתוח מבוצע לפני גיל חודש - Interrupted Aortic Arch עורקים: 3בקשת האאורטה יוצאים Innominate ; Left carotid ; Lt subclavian במצבים של .interrupted

aortic arch :יש חסר של חלק מהקשת Type A-חסר דיסטלי ל - Lt subclavian )35%(Type B חסר בין - Lt carotid לבין Lt subclavian )65%(Type C-חסר בין ה - inominate-ל Lt carotid )5%( .

,VSDהפגם בהתפתחות הקשת מלווה כמעט תמיד במומים תוך-לבביים ) truncus arteriosus, AP window, TGA, AS-ב .)type B שכיח למצוא Rt subclavian.אברנטי

גורמת לקריסה המודינמית שמחייבת מתןPDAאספקת הדם לפלג הגוף התחתון היא ע"י . סגירת הדוקטוס פרוסטין, וניתוח מוקדם )לרב ניתוח משולב לתיקון כל המומים בו"ז(.

הפרעה אחרת - קשת אאורטה נורמלית היא שמאלית )עוברת מצד שמאל אל האספקט הקדמי של הטרכאה(. יפנה לצד שמאל.innomonate של העורקים הגדולים ואז ה- mirror imageכאשר יש קשת ימנית מתקיים

Congenital ASvalvular AS

יש לרוב פתח צדדי ולא מרכזי., המסתם האאורטלי מעובההסוג השכיח ביותר והיחיד שמופיע כבר ביילוד. קוצר נשימה, אנגינה וסינקופה–בתינוק הסימפטומים הם איס"ק לב עם כחלון וקשיי נשימה. בילד יותר מבוגר

במבוגר עם כמו )טריאדת שרקו, או סביבתו מלוותASבמאמץ אורטלי היצרויות של מסתם בנוסף ניווני(. . ASבשכיחות מוגבת של מוות פתאומי. בהאזנה ללב - אוושה גסה של

הוא ע"י ASהטיפול ב- ballon valvotomy או valvotomy .)בניתוח )אין טיפול תרופתי AS מולד היא מחלה - גם לאחר תיקון התוצאות לא מספיק טובות וההיצרות תתפתח מחדש. אממארת שלם יש היצרות קריטית

ישרדו לאורך זמן.50%רק המסתם האאורטי שמתבטאת בילוד - צריך בדר"כ להחליף את המסתם לתותב. ההשרדות אחרי החלפת מסתםcongenital ASבילדים גדולים עם

יותר טובה, אבל יש בעיה של אנטי-קואגולציה ושל גדילת הילד ביחס למסתם. ניתן לפתור בעיה זו ע"י שימוש ב-allograft .)החלפת המסתם האאורטי במסתם פולמונלי, והשתלת מסתם פולמונלי תותב(

sub- valvular AS , מפריעה למעבר הדם במוצא חדר שמאל. החדר עובד חזק יותרמסתם ס"מ מתחת ל1/2-כטבעתית ממברנה

אחד העלים של המסתםאם AR, ויכול גם לגרום ילך ויחמירמום כזה עם הזמן להזרמת דם ועובר היפרטרופיה. בטיפול הניתוחי כורתים את הממברנההאבחנה של מום זה היא אינדיקציה לניתוח. לא נסגר טוב עקב הממברנה.

של חזרה(.30% כדי למנוע הישנות )אחרת יש septum myomectomyו-supra-valvular AS

)מעל ה-sinu-tubular junctionכל היצרות של מוצא האאורטה, בדר"כ ההיצרות היא ב- sinus of Valsalva.) יכולה להיות גם היצרות באאורטה העולה, אאורטה בטנית או אפילו בכלי הדם הריאתיים ובמוצא חדר ימין.

הוא בדר"כ חלקsupra-valvular ASהאטיולוגיה היא פגמים בייצור של אלסטין במדיה של כלי הדם הגדולים. ייחודיים ו-Williams-Beureneמתסמונת )שבה יש גם פיגור שכלי, תווי פנים peripheral pulmonary artery

stenosis .אבל יכול להיות גם מבודד ,)והחלפתו בשתל סינטטי או - כריתת האזור המוצר כירורגי . במקריםend-to-end anastomosisהטיפול הוא

מסויימים יש צורך להחליף גם את המסתם.Asymmetric Septal Hypertrophy (ASH)

מ- כחלק הספטום של Hypertrophisהיפרטרופיה Obstructive Catdiomyopathyעם גנטית מחלה , שהיא . המהלך הקליניMR. עיבוי הספטום גורם להיצרות אאורטית תת-מסתמית. במקרים רבים יש גם ADתורשה

13

Page 15: עינייםimg2.tapuz.co.il/CommunaFiles/39910353.doc · Web viewגם אם לאחר הפרוצדורה מתפתח PR, הלב הימני סובל את זה מצויין. נדיר

כירוגיית לב חזה

. בניתוח יש סכנה לפגיעה במערכתmyomectomyמשתנה - לפעמים די בטיפול תרופתי בומקרים אחרים דרוש ההולכה שעלולה לחייב קוצב.

Congenital PS )לא נסגר פזיולוגית כי הלחץ בעליה הימנית גבוהPFO. בדר"כ יש גם fused leafletsבמרבית המקרים מדובר ב-

ממה שהוא בדר"כ(. במקרים של היצרות קשה התינוק יהיה כחלוני ויסבול מאיס"ק ימנית. ,PRלאחר הפרוצדורה מתפתח ניתן לפתוח את המסתם הפולמונלי ע"י בלון ולרוב לא מגיעים לניתוח. גם אם

.cong PS נדיר לנתח הימני סובל את זה מצויין.הלב

מומים כחלוניים apnea. החסרון שלו הוא גרימת אירועי Prostinבמומים כחלוניים חייבים לשמור על הדוקטוס פתוח ע"י מתן

של עורק ווריד הטבור( ולכן צריךline )בפרט עם יש NECשמחייבים הנשמה. חסרון נוסף הוא הגברת הסיכון של או ניתוח מוקדם(. TPNלשקול מניעת כלכלה )מתן

. בדר"כ זה לאRashkind baloon septostomy ע"י ASD לא מספיק לייצוב החולה, ניתן גם ליצור PDAאם ה-דרוש )מנתחים מספיק מוקדם(. Tetralogy of Fallot (ToF)

ממצאים:4סינדרום הנוצר עקב "הזה" לשמאל של המחיצה. מאופיין בשילוב של 1.PS או pulm atresia2.Ventricular Septal Disease3.Overriding Aorta.)על הספטום( 4.Right Vetricular hypertophy.

, עםנולד ונראה בסדרהחסימה קשה: התינוק אם התמונה הקלינית נקבעת ע"י מידת הירידה בזרימה הריאתית. יש לתת פרוסטגלנדינים8-12בגיל , אבל סטורציה טובה נסגר. כי הדוקטוס וכמעט מת שעות הוא משחיר

זה עושה פלאים. לפתיחת הדוקטוס באופן מיידי, ו)פרוסטין( ,היאבעיה האם הילד לא פתח את הדוקטוס לאחר הטיפול כמעט תמיד מאחר שהתרופה כה יעילה

.באינפוזיה איבד את ההכרה. ואז בכה עם סיפור שהוא חצי שנה, התינוק יכול להגיע רק בגיל ToFבמקרים קלים מאד של

(.3חולף לבד עד גיל )שbreath holding spell את האפשרות של DDxבמקרה כזה יש ב- , כדי לאפשרPAל- Subclavian arteryיבור של ה- שבו יוצרים חBlalock-Taussig shuntהוא הראשוני הניתוח

ניתן להביא את הילד לתיקון מלא. ,אם יש עורקי ריאה בגודל סביר בשלב השני, מעבר של דם לריאות. בסיום תגרםזה נסבל היטב. לעיתים לרב אך , PR, כך שלמעשה יתקיים ישארו בלי מסתם פולמונליהניתוח רב הילדים

קן את המסתם הטרי-קוספידלי.תואז בגיל מאוחר צריך לנתח ול, TRהרחבה של חדר ימין ו-כיום יש מנתחים רבים שמאמינים בביצוע הניתוח הדפנטיבי כבר בשלב הראשון.

Transposition of Great Arteries (TGA). )לא נדון בטרמינולוגיה המפורטת(.אאורטה יוצאת מחדר ימין וה יוצא מחדר שמאלPAה- מום שבו

וקורס המודינמית -הם נסגרים הילד נהיה כחלוניהתינוק שורד, אבל כאשר ולכן פתוח ודוקטוס PFOבלידה יש . אבלמעלה מיד את הסטורציההפרוצדורה . ASDאם זה לא עוזר מספיק יש להגדיל את ה- ויש לתת פרוסטין

ימותו בתוך שנה.90% מהתינוקות ימותו בתוך חודש, 50%ללא טיפול דפנטיבי, ( שבו יוצרים "תעלהatrial switch תיקון ברמת העליות )– Mustard או ניתוח Senningפותחו ניתוח 60בשנות ה-

במקום ל-MVל-מדור מיוחד הדם עובר דרך עלייתית" שמאפשרת מעקף למעבר הדם ) TV . ברועמשם הוא ולחדר ימין(. החסרון הוא שהחדר הימני נשאר החדרTVל-רידי הריאות בוזר ולחדר שמאל ולריאות ומשם ח

הסיסטמי ובהדרגה מתפתחת איס"ק. ניתוחים כאלו עם100 היו סדרות של 70בשנות ה- ניתוחית, אבל התברר שיש תמותה אפס תמותה עד20%

(. ולכן נוצר צורך למצוא פתרוןRt failureלניתוח )שבוצע בגיל שנה(. בנוסף יש תוצאות גרועות לטווח ארוך )בגלל יותר טוב.

(. כאשרArterial switch )כירורג ברזיאלי בשם ג'אטין הראה שניתן להחליף בין העורקים, 80בתחילת שנות ה- שהחדר השמאלי התרגל תמותה כי הניתוח בוצע אחרי 60%ניסו לעשות את הניתוח בגיל שנה-שנתיים היתה

14

Page 16: עינייםimg2.tapuz.co.il/CommunaFiles/39910353.doc · Web viewגם אם לאחר הפרוצדורה מתפתח PR, הלב הימני סובל את זה מצויין. נדיר

כירוגיית לב חזה

, ולא היה מסוגל להתאים את עצמו ללחץ הסיסטמי. מצד שני, התוצאות של השיטה הישנה )החלפהללחץ נמוך.arterial switchניסו לפתח שיטות לשיפור ה- עלייתית( לא היו מזהירות גם הן, ולכן

ורק לאחר חודש עשוכדי להעלות את העומס השמאלי )לאמן את החדר( PA של ה-bendingתחילה היו עושים שבועות חיים(.2-3החלפה העורקית. זה נעשה גם כיום, רק אם הניתוח מאוחר )מעל את ה

, לפני שחדר שמאל יתרגל ללחץקם מנתח מצרי שהציע לעשות את הניתוח הזה מיד לאחר הלידהבשלב הבא )ביןקשה מאוד הוא הניתוח . גם היום בילדים קטנים כ"כהניתוח לקח זמן רב עד שלמדו לעשות את זה נמוך.

גדולה בכלים הקורונרייםהשאר בגלל - וריאביליות )כיום יש אחוזי תמותה קטנים מאוד( אבל רמת( ו2%< ניתוח זה.תודות להבעיות המאוחרות ירדה מאוד

Univentricular Heartתפקודית. הגדרה מהמומים 10-20%זוהי להגדרת המערכות- UVHנכנסים לשתי דם אחד שמוציא חדר

: univentricular heart צורות מבניות של 3(. ישנם הריאתית והסיסטמית) אסימפטומטי. בלידה הילד מסתמים תקינים 2עם PA. אאורטה ו-UVH מהמקרים של 30% עורקים: 1.2

וחדר שמתפקד כראוי. זרימת הדם לריאות מוגבלתבחלל החדר, ועשוי להשתחרר לביתו. יש ערבוב דם הילד(, הריאות מוצפות ו שבועות6גיל . כאשר התנגודת יורדת )בneonatal pulmonary resistanceי ע"

הואהדחוף , הזעה במאמץ האכילה. הטיפול לצוצר נשימהיפתח אס"ל עם בצקת ריאות קשה שגורמת כדי להפחית את הזרימה לריאות. לפעמים צריך לעזור PA banding ע"י ע"י קצת משתניםלתינוק

)כעבור זמן קצר ניתן להפסיק(. הריאות. בלידה pulmonary atresiaהאאורטה יוצאת מהחדר, אבל יש עורק ריאתי שאינו קשור ללב: .2

שעות, כאשר הדוקטוס נסגר, 12מקבלות דם ע"י הדוקטוס. כעבור הסטורציותאין מחזור דם ריאתי ו )חיבור בין עורק גדולBT shunt, וכעבור כמה ימים לעשות טיפול בפרוסטיןמייד תחיל החייבים ל. צונחות

גם סוגרים את ה-(. PAל- ואיס"ק. DAבאותו מועד כדי למנוע זרימת-יתר ללריאות צריך לתת לילד כדי לשמור על השאנט פוטנטי.אספירין

יש זרימה לא מופרעת לאאורטה ויש מהילדים. מום מאוזן - pulmonary stenosis: 10% עורקים עם 3.2pulmonary stenosisשנ כך והריאתי. , הסיסטמי המחזור בין איזון היאוצר אלו בילדים הסטורציה

, אין אס"ק לב וגם לא כחלון רציני ואין צורך בטיפול בינקות. 80-85% מחזורי הדם. 2כאשר הילד מעט יותר גדול, רוצים לעשות ניתוח דפנטיבי שיפריד את

, שבו למעשה מבטלים את החדר הימני: הורידים הגדולים מתנקזים אלFontan procedureבמקור הוצע לבצע עליה ימין וממנה אל עורק הריאה )מבלי לעבור בחדר(. הדם החוזר מהריאות נכנס לעליה שמאל וממנה אל החדר

היחיד, שמשמש כחדר שמאלי, ומשם מוזרק אל האאורטה. , וזה מספיק כדיLAP ל-CVPבהיעדר מומים נוספים )בעיות מסתמיות, עץ ריאתי פגום( מתקיים פער גדול בין ה-

כללים שחייבים להתקיים על מנת שהילד יהיה מועמד10להניע את הדם במערכת הריאתית. )גם כיום, ישנם (fontanable" child"לניתוח פונטן -

לפחות ונמצא שיש 4בשנים הראשונות הניתוח בוצע בילדים בני תמותה ניתוחית, ועוד 25-30% תמותה5% של סטורציה על לשמור כדי הסיבה: ביותר. גרועות תוצאות כמובן יחס80%לשנה, שהן שיתקיים צריך ,

P:S=2:1 יחידות נפח2 נפח כדי להצליח להעביר נפח פעימה אחד לפריפריה )3, כלומר החדר השמאלי מזרים פי כרוני ועדvolume overload שנים סבל לאורך זמן רב מ-4הולכות שוב לריאות(. לב שעשה עבודה כזו עד גיל

לניתוח זה כבר לב סמרטוט. חודשים,8-9(. הניתוח מבוצע בגיל Hemi-Fontan procedure )או Glenn shuntבנסיון לפתור את הבעיה, פותח

אלSVCה- ( ובמקומו מחברים אתbanding עם ה-PAובו מנתקים את אספקת הדם אל הריאות )השאנט או ה- מההחזר הורידי אל הריאות, וזו זרימת הדם היחידה לריאה. שאר ההחזר הורידי40%. בשיטה הזו, נותנים PAה-

סטורציה(. זה80-85%מגיע לעליה שמאל ופוגש את הדם שחוזר מהריאות, ואז מוזרק למערכת הסיסטמית ) . volume overloadמבטל את ה-

)הגיל דרוש כדי SVC מהחדר ומחברים אותו ישירות אל ה-IVC עושים ניתוח נוסף שבו מנתקים את ה-4בגיל עוקפים לחלוטין את ,Fontan procedureמשלימים את שניתן יהיה ליצור חיבור ע"י צינור פוטנטי(. בניתוח בעצם

. 97%חדר שמאל. הסטורציה מתייצבת על רמה של

15

Page 17: עינייםimg2.tapuz.co.il/CommunaFiles/39910353.doc · Web viewגם אם לאחר הפרוצדורה מתפתח PR, הלב הימני סובל את זה מצויין. נדיר

כירוגיית לב חזה

, תפוקת שתן ירודה וחמצת( בגלל הגדלתLAP ו- CVP )Low COלפעמים אחרי הניתוח נצפה מצב של לא מספיקbanding גדול מדי או shunt )נוזלים או דם( ; DD: pneumothorax ; hemothoraxהתנגדות ריאתית. ב- גדול מדי( ; היצרות העץ הריאתי עצמו.pulmonary resistantטרם הניתוח שגרם

ובין העליה. חיבור זה מפחית אתFontan - יוצרים חיבור בין ה-Fenestrated Fontanניתן לפתור את הבעיה ע"י ירידה הוא וישלח למערכת הסיסטמית(. המחיר יכנס לחדר הורידי )חלק מההחזר בריאות הלחץ של הדם

שמור. CO(, אבל מרוויחים 90%~בסטורציה ) שמאפשרlinea albaכיום הפנסטרציה נעשית בכל ילד בתום הניתוח, כאשר משאירים חוט בין הפנסטרציה ל-

.desaturatedלסגור אותה )או להקטין את קוטרה( אם הילד נהיה יותר מדי תמותה לשנה,1% בשלושת הניתוחים ביחד, ואח"כ יש עוד 5%תוצאות הניתוח: יש סיכון מצטבר לתמותה של

כי לפעמים החדר השמאלי "לא קרא את הספר" ואינו מסתדר.

מבוא לניתוחי מסתמים

מסתמים חלופיים עדיף לשמור על המסתם הטבעי )לתקן אותו( ולא להחליפו. עם זאת, תיקונים אינם–בכל מקרה שבו זה אפשרי

. tricuspid או mitral )הוא מסוייד(. תוצאות התיקון יותר טובות ב-Aortic Valveמוצלחים ב-מסתמים חלופיים – יתרונות וחסרונות

המודינמיקזמינותזיהומיםT/EACעמידותה

+++)-(++++++משתנה~++ביולוגי

+++)-()-()-(++++++מכאני

)-(+++++++++++++אוטוגרפט

מסתם ביולוגי בצורת מסתם. מסתם שנלקח מחזיר או פריקרד של בקר שמולבש על תבנית-מסתם ביולוגי

ניתן להשתיל מסתם ביולוגי בעמדה אאורטית או מיטרלית. מסתם ביולוגי אינו אופטימלי מבחינה המודינמית כי הוא מתלבש על טבעת דקרון )כדי שנוכל–המודינמית

לתפור אותו(, כך שהפתח האפקטיבי יותר קטן מאשר החלל הפוטנציאלי של המסתם, ולכן יש למעשההיצרות מסויימת של חלל המסתם.

תלויה בשני גורמים: –עמידות o העמדה שבה המסתם יושב–mitral מחזיק יותר מאשר aortico רק30בגיל המנותח - ככל שגיל המנותח יותר מבוגר כך העמידות יותר גבוהה, באופן פרדוקסלי. בגיל

בגיל 90%, לעומת 10y מהמסתמים ישרדו 50% החולה המבוגר לא יצטרך החלפת–. )בפועל 65 מסתם חוזרת(. הסיבה לא ברורה.

( תרומבואמבוליותT/E( ואנטיקואגולציה )ACהתרומבוגניות של מסתם ביולוגי היא נמוכה מאד. הסיכון :) AC. לכן, נותנים ACלקרישים ותסחיפים אינו אפסי, אבל שכיחות הקרישים יותר נמוכה משכיחות סיבוכי ה-

חודשים לאחר הניתוח עד להחלמת התפרים. )כמובן שאם יש אינדיקציה אחרת ל-3רק במשך ACכמו , פרפור פרוזדורים, לא ניתן להמנע ממנה(.

( זיהומים: שכיחות הזיהומים של המסתםSBEגבוהה, כי המסתם הוא גוף זר. העובדה שמקור המסתם ) מהחי אינה משמעותית כאן.

לוקחים פריקרד של פרה ותופרים אותו בצורה– גבוהה. במקרים רבים המסתם מיוצר מלאכותית –זמינות של פריקרד.

מסתם מכאני

16

Page 18: עינייםimg2.tapuz.co.il/CommunaFiles/39910353.doc · Web viewגם אם לאחר הפרוצדורה מתפתח PR, הלב הימני סובל את זה מצויין. נדיר

כירוגיית לב חזה

מקסימלי.–תפקוד המודינמי .עמידות מעולה, אין צורך להחליף את המסתם שוב השכיחותT/E מאד גבוהה ומחייבת AC .לכל החיים שכיחות גבוה, לא שונה מהשכיחות במסתם ביולוגי.–זיהומים מוחלטת )מוצר מדף, ייצור מסחרי לעמדות וגדלים שונים(–זמינות

אוטוגרפט מסתם שמקורו בגופת אדם.

.תפקוד המודינמי מקסימלי יותר מאשר ביולוגי אבל פחות ממכאני.–עמידות T/E-ו AC – לא תרומבוגני כלל ולכן לא מצריך AC . אוטוגרפט הוא לא גוף זר, ולכן פחות רגיש לזיהומים. –זיהומים מוגבלת ביותר. ניתן לקחת רק מסתם אאורטי, ורק מצעירים )מסתם שעדיין לא הסתייד(.–זמינות

הומוגרפט של אותו החולה, והשתלת מסתם חלופי בעמדה פולמונית. pulmonic valve ע"י Aortic valveהחלפת

ניתוח מסובך מאד ודי נדיר. מבוצע בעיקר בצעירים. סיבוכים טרומבו-אמבוליים ואנטיקואגולציה

: T/E סוגים של סיבוכים 2מבחינים בין MAJOR thrombo-embolic phenomena.סיבוך שגורם לתחלואה קשה או מוות, ותמיד יצריך טיפול רפואי -

או הפרעת קצב קטלנית. MIלדוגמא: תסחיף למח המביא להמיפלגיה, תסחיף לקורונריים שגורם MINOR thrombo-embolic phenomenaתסחיפים שלא גורמים לנזק חסר השפעה לטווח הארוך. רוב -

ארועים אלו סמויים )למשל אוטם בטחול( שבמקרים רבים בכלל לא ידיעו לידיעת הרופא. קשה להעריך את ארועים מאלו אשר מזוהים ע"י דרגים2השכיחות המדוייקת של ארועים כאלו. כיום משערים כי קיימים פי

רפואיים. תלויה ב- :T/Eהשכיחות של

.סוג המסתם- כפי שפורט קודם יעילות טיפולAC .( הפרעות קצבAF )-העמדה שבה יושב המסתם: הסיכון בmitral מאשר 2-2.5 גבוה פי aorticסיכון זו מוגבר עוד יותר בנוכחות .

AF סה"כ שכיחות לארועי .major-שכיחות אירועים ל( שנות מסתם, תחת טיפול ב-100 AC:)o aortic – %0.5 oMitral 1%בסינוס oMitral + AF 1.5% .

סיבוכי אנטיקואגולציהMajor bleeding – דמם מוחי, דמם GI 1-2% משמעותי. שכיחות של.Minor bleeding – .אפיטקסיס, דמם בעקבות פרוצדורה כירורגית קטנה

וגם להמנע מדימום. השכיחות של הסיבוכים ההמורגיים תלויה רקT/E צריך להיות מושכל, כדי למנוע ACטיפול אחר:INR. כדי לגרום למינימום סיבוכים אפשרי, עבור כל מסתם נקבע יעד INRב-

aortic 2-2.5.mitral – 2.5-3mitral + AF – 3-3.5

תלוי בעיקר במודעות ובהענות של המטופל וםחות בטיב הטיפולINRהנסיון מראה שהאיזון האופטימלי של הרפואי בו. בשל כך נמצא חולים אשר סולים מסיבוכי הטיפול שוב ושוב בעוד חולים אחרים חיים שנים רבות ללא

כל סיבוך מהטיפול.מהזמן )ולפעמים רק 70%גם במצב האופטימלי, האיזון מושג רק ב- מהזמן(. כאשר עיקר הסיבוכים30-40%

אופטימלי, לאINR מגיעים לערכי קיצון. אולם יש לזכור שגם באיזון INRמתרחשים כאשר מסיבות שונות ערכי ה

17

Page 19: עינייםimg2.tapuz.co.il/CommunaFiles/39910353.doc · Web viewגם אם לאחר הפרוצדורה מתפתח PR, הלב הימני סובל את זה מצויין. נדיר

כירוגיית לב חזה

בשנה דמם1%ניתן למנוע את הסיבוכים )דמם או קרישים( לחלוטין. בכלליות טיפול אנטי-קואגולנטי מביא ל בשנה של אירועים תרומבואמבוליים. 1%מאג'ורי,

התחלת טיפול אנטי-קואגולנטי ( ובמקביל להתחילLMW Heparinלאחר השתלת מסתם מכאני צריך להתחיל טיפול בהפארין )או קלקסן, שהוא

תראפוייטי. INRחפיפה לקומאדין. אין להפסיק את ההפארין עד שהקומאדין נותן התחלת הטיפול בהפארין היא לרב ביממה השניה לאחר הניתוח )לא מיד לאחר הניתוח, כי הסיכון של דמם

ניתוחי הוא גבוה ביותר בשעות הראשונות(. העקרון: יותר קל לתקן סיבוכי דמם בבית החזה מאשר שבץ במח. Prosthetic valve endocarditisזיהומים -

סוגים: 2מבחינים בין Early – עד 2moהמחוללים לבי"ח. האופיינית עמידות בעלי אלימים יותר המחוללים הניתוח. לאחר

. לא ניתן להשתלט על30-40%מתיישבים על מסתם שנותח. המחלה מאד סוערת והתמותה מגיעה עד . זהו סיבוך נדיר.20%המחלה ע"י אנטיביוטיקה וחייבים להוציא את המסתם. התמותה בניתוח החוזר מגיעה ל

Late – מעל 2mo לאחר הניתוח. המחוללים דומים לאלו של native valve endocarditisהחיידקים לרב . ב טיפולי בנסיון נתחיל ולכן סוערת פחות הרבה והמחלה לאנטיביוטיקה, ישנןABxמגיבים זאת, עם .

ל-1-2%אינדיקציות להתערבות כירורגית )מיד(. השכיחות היא שנת מסתם, הסיכון לתמותה מהמצב100 .10%כולל משני לניתוח מגיעה ל

( :late-prosthetic )מסתם נאטיבי או SBEאינדיקציות לניתוח של חולה עם .ערעור המודינמי של המסתם כאשר החולה לא מגיב לטיפולAbx. )אבססים( הזיהום של חוץ-וולבולרית להתפשטות ויכולים–עדות הלב ברקמת מוגעים האבססים

לעשות שחזור של נוכל גדול שלא כ"כ יהיה נחכה, ההרס הולכה חדשות. אם בהפרעות להתבטא המסתם.

קיום של וגיטציות גדולות עם אמבוליות ספטיות מהווה אינדיקציה יחסית לניתוח–אמבוליות ספטיות )וגיטציות לבדן אינן אינדיקציה לניתוח(.

תמותה. בזיהום ע"י 100% בזיהום פטרייתי ללא ניתוח יש כמעט –סוג המחולל Staphהסף להתערבות ימים של2-3הוא יותר נמוך כי הוא נוטה לעשות אבססים. לכן, אם לא משתלטים על הסטאפ בתוך

אנטיביוטיקה, נלך לניתוח.איזה מסתם לאיזה חולה ?

<( שצריך החלפת מסתם, ההעדפה היא למסתם ביולוגי שכנראה לא יצטרך החלפה חוזרת65באדם מבוגר )ו- נוסף )עם הסיכוןACוחוסך את הסיבוכים של תרומבואמבוליות יותר הסבירות שיזדקו לניתוח . בגיל צעיר

הנלווה( היא מאד גבוהה ולכן בדר"כ נשתיל מסתם תותב. מאד בילד לכן, גבוה. מאד הללו המסתמים של ההסתיידות קצב כי ביולוגי מסתם לשים ניתן לא בילדים

משתדלים לתקן את המסתם הטבעי או לשים הומוגרפט/אוטוגרפט. - טיפול אנטי-קואגולנטי הוא מאד מסוכן בטרימסטר ראשון ושלישי להריון. לכן, באשה בגיל הפוריות נשים ביולוגי. כאשר הוא מתבלה האישה בדר"כ כבר סיימה את תוכניות שמתכוונת להרות בדר"כ נשתיל מסתם

הפוריות שלה ואז שמים מסתם מכאני ומתחילים בטיפול אנטי-קואגולנטי. לטיפול האדם של ההיענות את להעריך צריך לניתוח. המועמד באדם מאד תלויה המסתם בסוג הבחירה אנטי-קואגולנטי לעומת הרצון שלו לקחת סיכון של החלפת מסתם חוזרת. ככל שהאדם יותר מעורב בקבלת

ההחלטות, כך הוא יהיה יותר מודע למצבו יטפל בעצמו טוב יותר. , שצפוי לשפר את יכולת האיזון של החולים. עם זאת, גםINRבעתיד הלא-רחוק יכנס מכשור למדידה ביתית של

תראפוייטי יכולים להיות ארועים טרומבו-אמבוליים. INRב-

מחלות המסתם המיטרלי

מבנה

18

Page 20: עינייםimg2.tapuz.co.il/CommunaFiles/39910353.doc · Web viewגם אם לאחר הפרוצדורה מתפתח PR, הלב הימני סובל את זה מצויין. נדיר

כירוגיית לב חזה

המסתם בנוי מרטיקולום שכולל אנדותל אך לא כלי דם )הוא ניזון בדיפוזיה(. המצאות כלי דם היא תוצאה שלמחלה ראומטית בד"כ.

במסתם יש עלה אנטריורי ועלה פוסטריורי. במבט מהעליה, העלה האנטריורי הרבה יותר גדול ופונה לספטום, העלה הפוסטריורי פונה לקיר החופשי. העלה הקדמי תורם את מרבית הפתיחה של המסתם )אם העלה האחורי

סטנוטי זה לא יעשה היצרות משמעותית במסתם(. שבנוי מקולאגן וציפוי אנדותל, ושרירים. השריר הפפילרי בנוי מכמה פטמותchordaהאפרטוס התת-מסתמי כולל

)פפילות(, ולכל אחת מתחברים כמה כורדות. השריר הקדמי מחובר גם לעלה האחורי, ולהיפך. זה מתאפשר ע"י של המסתם. Obliqueמנח

תפקוד בזמן הסיסטולה המסתם מוחזק סגור על ידי האפרטוס. כיווץ השריר הפפילרי בזמן הסיסטולה הוא לא משמעותי מבחינה תפקודית. הסגירה היא פסיבית בהתאם למפל הלחצים. למעשה השריר משמש כאוגן לעלי המסתם.

papillary muscle קיצור המרחק בין המסתם לנקודת החיבור של ההמסתם נסגר הודות לכיווץ דופן החדר קצה המסתם יכול לנוע מעלה ולהפגש עם העלה השני.

מטעה. שכן במצב זה הבעיה אינה חוסר פעילות של השרירpapillary muscle dysfunctionבשל זאת המושג אין קיצור המרחק בין אוגן עלי המסתם לשורש המסתםהפפילרי אלא חוסר תפקוד של דופן החדר השמאלי

.העלה לא יכול להסגר במלואו בזמן דיאסטולה יש הרחבה של הטבעת, הרפיה של השריר )כחלק מההרפיה של החדר( ואז המיתרים הולכים

אחורה וגורמים לפתיחת המסתם.

Mitral Regurgitationפתוגנזה ומנגנונים

משטח העלה נמצא במגע עם העלה השני. לכך צריך שהעלים יהיו חופשיים ומסוגלים25%במצב התקין, כ- להגיע לחפיפה. בכל אי-ספיקה יש רסטריקציה של עלי המסתם על ידי השריר, כך שהם לא מגיעים למישור

החפיפה )מישור הקואפטציה(. ישנם מנגונים שונים שגורמים לכך.רסטריקציה של השריר

איסכמיה עלולה ליצור צלקת, והאיזור הצלקתי מונע את תנועת השריר לעבר העליה, וסגירת המסתם. זו לא כמו יתדות שלtetheringהגבלה של יכולת ההתכווצות )ההתכווצות מעורבת בפתיחה של המסתם(. זה מצב של

אוהל. יש רסטריקציה של תנועת המסתם. במצב כזה יתכן שנראה מסתם משוך לתוך החדר בזמן הסיסטולה. אותה הפרעה מתקיימת בעת הרחבה של החדר, כשיש הגדלה של המרחק בין מוצא השריר למסתם, ואז השריר

להלן(. –לא יכול להגיע לכיוון העליה )במצב כזה גם האנולוס מתרחב ולגרום לפיברוזיס. שוב, ההגבלה התת-מסתמית מונעת במחלה ראומטית יש דלקת שיכולה לפגוע במיתרים

מהמסתם להגיע למקום שלו, ולמעשה יש את אותה רסטריקציה שגורמת אי ספיקה. חוסר התפקוד של השריר יכול להיות סגמנטלי או כחלק מחוסר תפקוד גלובלי של החדר השמאלי. ולכן תיתכן אי

גורם בעצמו להרחבת החדר כלMRספיקה של שריר אחד באופן מבודד, בלי מעורבות של השני. מאחר שכל נוצר MRאטיולוגיה שיוצרת תביא להתפתחות מנגנון זה. לאחר יצירת המנגנון viscous circleשמחמיר את

המצב.הגדלה של האנולוס

כמעט בכל מצב שגורם להרחבה של החדר יש גם הגדלה של האנולוס המסתמי, שבעקבותיה נמנעת החפיפה ביןעלעלי המסתם ומתפתחת אי-ספיקה.

צניחה של העליםששכיח יותר בנשים. בהרבה מקרים ה-)Mitral Valve Prolapse )MVPקיים מצב של prolapseאינו בשטח

הקואפטציה ובמקרה כזה אין ספיקה. הפתולוגיה במקרה כזה היא שבעלה האחורי יש קטע שבולט לתוך חלל העליה בגלל שינויים מיקסומטואידיים או חולשה של הקולאגן. הקטע הזה מכה בדופן העליה, וזה מה שיוצר את

ללא אוושה של אי-ספיקה מיטרלית. Barlow syndrome=systolic click syndromeהקליק. מתקבל אי ספיקה מתפתחת במצב של מחלות רקמת חיבור, כששני העלים עוברים שינויים דגנרטיביים. לחילופין, יכול

להיקרע שריר או מסתם, וזה גם גורם לזה. ייתכן גם קרע של כמה מיתרים, ואז רק חלק מהעלה הוא צנוח.

19

Page 21: עינייםimg2.tapuz.co.il/CommunaFiles/39910353.doc · Web viewגם אם לאחר הפרוצדורה מתפתח PR, הלב הימני סובל את זה מצויין. נדיר

כירוגיית לב חזה

חור במסתם או דליפה מסביב למסתם כפי שיכול להתרחש לאחר ניתוח החלפת מסתם.SBEחור בעלים עקב

Acute MRמנגנון

בלב בריא, הגדלת נפח הדם בחדר)למשל במאמץ( גורמת לעליית לחץ וחוסר יכולת של החדר להתמודד. בשלב של הגעה לקצה היכולת, יש עליית לחץ בורידי הריאה וזה גורם לקוצר נשימה בגלל דלף נוזלים וגם בגלל הפעלה של בארורצפטורים בורידי הריאה שגורמים לקוצר נשימה רפלקסיבי. קוצר הנשימה יגרום לאדם להפסיק את

הפעילות הספורטיבית שגרמה לעליית הנפח.אקוטי, הנפח המוגבר לא נגרם ע"י טריגר שניתן להפסיקו, אלא ע"י הדלף של המסתם. החדר נמצאMRב-

בנקודה מאוד מתקדמת מבחינת הנפח ולכן יש לחץ מאוד גבוה וסימפטומים.

עקומת לחץ כביטוי של הנפח בחדרהכחול: מצב נורמלי.

Chronic MRהאדום:

קליניקה אבל ההיענות של החדר מוגבלת ומתפתח לחץvolume overload אקוטי, החדר השמאלי מקבל MRבמצב של

( דיאסטולית. הלחץ הגבוה משפיע לאחור בעיה יש לכן )ראה עקומת ההיענות(. גבוה סוף-דיאסטולי מאד backflow pressure.ויש עליית לחץ בקפילרות הריאתיות ובצקת ריאות )

ומתפתח שוק קרדיוגני. אם לחדרEffective COבנוסף, בסיסטולה חלק מהדם חוזר לעליה ולכן יש ירידה ב- והיה יכול לתתSV כרוני( הוא היה מסוגל לקבל את הנפח ולהגדיל את ה-MRהיתה היענות גדולה יותר )כמו ב-

דרוש(. COלספק infective אקוטי ייתכן שילוב של בצקת ריאות ושוק קרדיוגני. זה יכול לקרות לקרות למשל ב-MRלכן, במצב של

endocarditis .או כסיבוך של אוטם טיפול

כדי "למשוך"afterload עצום על החדר. חייבים להוריד את ה-volume overloadכאמור, הבעיה הבסיסית היא את הדם קדימה, מהחדר לאאורטה.

ה- ע"י afterloadהורדת היא האולטימטיבית intra-aortic baloon pumpהתנגודית את דרסטית שמוריד הורדת התנגודת הפריפרית תגרום לכך שלחולה–הסיסטמית. אם ניתן תרופות ואזודילטוריות נחמיר את המצב

ע"יafterloadלא יהיה ל"ד, והורדת התנגודת הריאתית תחמיר את הבצקת. )במצב לא מאד קריטי ניתן להוריד משתנים(.

אקוטי הוא ניתוח דחוף !!!MRבכל מקרה, הטיפול הדפנטיבי של Chronic MR

MR כרוני הוא תהליך הדרגתי. במצב של MRכרוני )כמו באימון גופני הדרגתי( כיווציות חדר שמאל נעשית טובה יותר מאשר של חדר רגיל ויש לו יכולת להתמודד עם עומס הנפח, בלי להעלות מאד את הלחץ הסוף-דיאסטולי.

COהתארכות סיבי השריר מאפשרת את הגברת ההתכווצות )עפ"י עקומת פרנק-סטרלינג( והחדר מצליח לתת הוא אסימפטומטי. למרות זאת, בשלב מסויים ההתארכות שלMRאפקטיבי. בשלב הזה, שיכול להמשך שנים, ה-

, והלחץ בחדר↓COהשריר תביא לירידה ביעילות של התכווצות. לכן נפח דם מסויים ישאר בחדר. משני לכך מתחיל לעלות.

(acute MRרק בשלב הזה יתפתחו סימפטומים - קוצר נשימה, עייפות במאמץ )במנגנון דומה לבצקת ריאות של (. תחושת קוצר הנשימה יכול להיות גם במנגנון רפלקטוגני משני לעליתCOוסימני איס"ק לב )בגלל הירידה ב-

יכול להרגיש קוצר נשימה למרות שהריאות שלו "יבשות".MRהלחץ בעליה שמאל. כלומר אדם עם בנוסף, ככל שנפח החדר עולה יש גם עליה במתח הדופן, ולכן יש התנגדות רבה יותר לזרימה הכלילית. שריר כזה

בביופסיה יהיה זרוע במיקרו-אינפרקטים )לרב ללא עדות קלינית לאוטמים( וללא עדות למחלה כלילית.הטיפול

20

P

V

Chronic MR

Acute MR

Page 22: עינייםimg2.tapuz.co.il/CommunaFiles/39910353.doc · Web viewגם אם לאחר הפרוצדורה מתפתח PR, הלב הימני סובל את זה מצויין. נדיר

כירוגיית לב חזה

כרוני הוא שבמשך זמן רב השינויים הם הפיכים ושקטים )א-סימפטומטיים(. ברגעMRהמהלך הטבעי של שמתחילה ההתדרדרות זו תוצאה של שינויים מבניים לא הפיכים.

לשנה, והבעיה היא שגם כשמתקנים או מחליפים את המסתם, זה עלול8% סימפטומטי היא MRהתמותה מ-להיות מאוחר מדי לתיקון הנזק לשריר, ובחלק מהמקרים גם לאחר ניתוח תוחלת החיים נמוכה. לעומת זאת, ב-

MRוגודל חדר תקינים, תוחלת החיים שמורה. לכן, רוצים לתקן את המסתם לפני אסימפטומטי עם תפקוד שהחולה נעשה סימפטומטי או שהחדר נפגע.

, ואז לעקוב אחר החולהbaseline צריך לבצע הערכה של גודל ותפקוד החדר השמאלי כ-MRבחולה שאובחן עם )עובי הדופן בסיסטולהfraction of shorteningמדי חצי שנה ע"י הערכה קלינית ובדיקת אקו - גודל החדר וה-

לזכור את האפשרות שהחולה לא הגיעMRלעומת הדיאסטולה(. אבל: בכל מעקב אחר חולה עם חייבים . חייבים לקחתbaseline. לכן, אסור להניח שהממצאים בהערכה הראשונה הם ה-MRלידיעתנו מיד כשנוצר ה-

בחשבון את האפשרות שאלו כבר ממצאים של התחלת התדרדרות.גורם להורדתMR( כי ה-hyperkinetic, התפקוד של חדר שמאל הוא מעל הנורמה )MRבמצב הבסיסי של

afterload-לכיוון העליה( ולחדר קל יותר להתכווץ. לכן, ב( MR יתקבל FoS=40-45% לעומת במצב30-35% (.MRנורמלי )ללא

מתחיל לרדת לכיוון הטווח הנורמלי, זה מעיד על פגיעה בחדר. בשלב הזה, גםFoSכאשר החדר מתרחב או שה- אם החולה אסימפטומטי, נשקול התערבות ניתוחית. במקביל, לא רוצים לנתח מוקדם מדי, בגלל שהניתוח חושף את החולה לסיכון לא מבוטל )תחלואה ותמותה סביב-ניתוחית( וגם בגלל שהמסתם החלופי בפני עצמו מקטין את

תוחלת החיים ! שיקול חשוב בבחירת עיתוי ההתערבות הוא סוג הניתוח: אם המסתם ניתן לתיקון, התחלואה הפוסט-ניתוחית היא הרבה יותר נמוכה )אין קרישיות, אנטי-קואגולציה, זיהומים( ואז אין שום הצדקה לעכב את הניתוח מעבר לשלב שבו תפקוד החדר מתחיל להפגע )אין הצדקה לנתח פרופילקטית קודם לפגיעה בחדר, כי הניתוח עצמו מסוכן(. לעומת זאת, אם ברור שלא נוכל לתקן את המסתם ונהיה חייבים להחליפו, יש טעם בדחיית הניתוח עד לתחילת הסימפטומים, והתמותה ללא ניתוח מצדיקה את הסיכון הניתוחי והפוסט-ניתוחי. החלפת מסתם מיטרלי לחולה

שהוא אסימפטומטי היא גרימת מחלה לא מוצדקת. ההכרעה האם אפשר לתקן את המסתם מבוססת על ההופעה האקוקרדיוגרפית. קרע של מיתרים, למשל, בטוח שאפשר לתקן ואילו מסתם פיברוטי בלי תנועה של העלים והמיתרים לא ניתן לתיקון. ישנם הרבה מצבי בינייםשבהם היכולת לתקן את המסתם בתלות באיכות הכירורג. ככלל הסיכוי לתיקון יותר טוב במצבים בהם הגורם ל

MR.)הוא עודף רקמה )מחלה דגנרטיבית( בניגוד לאתיולוגיות של חוסר רקמה )ראומטית, איסכמית במהלך התיקון יש חשיבות לשימור המנגנון התת מסתמי. שכן שימור זה משפר את תפקוד שריר החדר. נקודה זו

למשל עקב מחלה איסכמית.–חשובה במיוחד בחולים בהם תפקוד החדר פגוע הפרוגנוזה של תיקון המסתם היא מעולה. מלבד החסכון של סיבוכי המסתם התותב, יש גם יתרון תפקודי למסתם הטבעי, כי קיימים יחסי גומלין בין תפקוד החדר לבין מבנה המסתם: כיווץ השריר מחייב נקודת התייחסות נייחת,

וה- הפפילרי השריר עם המסתם פגיעהchordaוזהו ויש הטבעית הגיאומטריה נאבדת המסתם, בהחלפת . ביעילות של התכווצות השריר. לכן גם בהחלפה של מסתם משתדלים להשאיר לפחות את העלה האחורי, אבל זה

בתיקון של המסתם. 1-1.5%, לעומת 7%לא עוזר, והתמותה הניתוחית בהחלפה היא

ראומטי התיקון יותר קשה(. MR ניווני )ב-MR תיקון של 90%בעולם המערבי )ארה"ב, צרפת( יש כיום מעל תוך-ניתוחי לאחר הירידה ממכונתTEEכמובן שבתיקון מסתם חייבים לוודא שהפתולוגיה תוקנה. זה מבוצע ע"י

. כל תוצאה אחרת אינה מתקבלת על הדעת, ומחייבתmild MR, או לכל היות no MRלב-ריאה. שואפים להשיג תיקון נוסף או החלפה של המסתם.

על רקע איסכמי הם שונים. במצב איסכמי יודעים שנחרץ גורלו של החולה ברגע שישMRהניהול והפרוגנוזה של (. באטיולוגיה איסכמית, הפרוגנוזה של המחלהMR כי מלכתחילה התפקוד ירוד )זה מה שגרם ל-MRאבחנה של אחרי הניתוח.MR. גם לאחר ניתוח לתיקון המסתם, צפוי שתהיה חזרה של ה-5yS 20% –היא גרועה

Mitral Stenosisפתוגנזה

21

Page 23: עינייםimg2.tapuz.co.il/CommunaFiles/39910353.doc · Web viewגם אם לאחר הפרוצדורה מתפתח PR, הלב הימני סובל את זה מצויין. נדיר

כירוגיית לב חזה

האטיולוגיה ברב מוחלט של המקרים היא ראומטית. המנגנון הוא תהליך ריפוי של הדלקת, ופיברוזיס של המסתםשל הריפוי ע"י נפגע והמסתם השכבות, בכל פן-קרדיטיס יש ראומטית במחלה התת-מסתמי. והמנגנון

האנדוקרדיטיס. ובגלל איחוי זב"ז, בהיקף המסתם(, )נקודת המגע של שני העלים בשלב הראשון יש איחוי של הקומיסורות הקומיסורות, הפתח האפקטיבי הולך וקטן. אם ריפוי הדלקת נמשך, יש גם פיברוזיס של המיתרים שגורמת לעיבוי

יוצר גם בגלל עיוותMRוהתקצרות שלהם. במקרים מתקדמים, יש הדבקות של העלה לשריר הפפילרי. זה המסתם )פיית דג, שלא נפתח ולא נסגר(. בשלב המתקדם יש גם שינויים ניווניים והסתיידות של המסתם.

מהלך קליני backflow pressure מסתמן ע"י קוצר נשימה בגלל MS היא תשלובת של גורמים. MSהווצרות סימפטומים ב-

תלוי בשטח המסתם, בקצב הלב )משך הדיאסטולה( ובתפוקת הלב: LA לריאות. הלחץ ב-Lt atriumמה-עולה כדי ליצור מפל לחצים מוגבר שיוכל להזרים את הדם דרך ההיצרות,LAכאשר המסתם מוצר, הלחץ ב-

והלחץ המוגבר הזה מועבר אל הריאות. כאשר הדיאסטולה ארוכה יש יותר זמן למילוי החדר ומפל הלחצים יורד. הוא יכנס באופן פתאומי לבצקת ריאות, גם אם בקצב סינוס הוא א-סימפטומטי. AFלכן, אם החולה נכנס ל-

מוגבר משמעו יותר נפח שצריך לעבור דרך המסתם המוצר, ולכן יותר לחץ.COהלחץ תלוי גם בתפוקת הלב, כי במצבים שבהם עולה תפוקת הלב )הריון, תירוטוקסיקוזיס( יתפתחו סימפטומים.

(. MRקיימת קורלציה ישירה בין חומרת ההיצרות לבין הדרגה התפקודית )בניגוד למצב ב-ברמה אנטומית: אסימפטומטי )סימפטומים רק בשינויי קצב או נפח(MSהתקדמות עלייתLAP עליית

קפילרי וראקטיביתלחץ רפלקסיבית הריאתיות, הארטריולות של ואזוקונסטריקציה אנטומיים שינויים לחץ דם ריאתי מוגבר ולא הפיך. בדפנות הארטריולות: עיבוי של השריר החלק והסאב-אינטימה

יל"ד ריאתי = מפל הלחצים )באקו( + ואזוקונסטריקציה ראקטיבית + ואזוקונסטריקציה אנטומית לא הפיכה. ככל שהלחץ הריאתי יותר גבוה ופחות הפיך, הסיכון הניתוחי עולה. לכן, בהחלטה על הטיפול חשוב לברר מהו

.pulmonary HTNהמרכיב הבלתי-הפיך של ה-הואזוקונסטריקציה את בצנתור(. ישירה למדידה )או אקו ע"י להערכה ניתן המסתם על הלחצים מפל

מאד פוטנטי, ורואים את השפעתוpulmonary vasodilator חמצן, שהוא 100%הראקטיבית ניתן לבדוק ע"י מתן בצנטור ימני )או, לפעמים, באקו(. בהנתן הנתונים הללו, ניתן לחשב את המרכיב הבלתי-הפיך.

לחילופין, ניתן גם לעשות חישובים המודינמיים ישירים ע"י צנתור דו"צ: מודדים לחצים, מחשבים תנגודת ריאתית או לתנגודת הריאתית )מה מידת ההפיכות(.COואז נותנים חמצן או ניטרופרוסיד ורואים מה קורה ל-

MS דרגה פונקציונלית.– היא מחלה ממושכת והדרגתית. ניתן להעריך את התקדמות המחלה ע"י מעקב קליני קומות ? בכמה זמן ? כמה3חשוב לעשות אנמנזה ביקורתית ע"י שאלות מכוונות לחולה וגם למשפחתו )עולה

הפסקות ? החולה עצמו לא תמיד מודע למגבלות שלו(.הטיפול

טיפול שמרני ע"י האטת דופק,–בחירת הטיפול תלויה מאד בדרגה התפקודית. בדרגה פונקציונלית טובה-סבירה בצנתור או ניתוח. )מובן שתיקון עדיף בהרבה עלvalvuloplastyדיורטיקה. כאשר החולה מתחיל להתדרדר, נציע

החלפה(. : valvuloplastyתנאים להתאמה ל-

ותמותה עצומה(acute MRמסתם שאינו מסוייד )במסתם מסוייד יש סכנת .1, במיוחד בצנתור(valvuloplasty ע"י ה-MR )עלולים להחמיר MRללא .2שלילת קרישים בעליה )כדי שלא לגרום אמבוליזציה במהלך הפרוצדורה(.3שלמות המנגנון התת-מסתמי )אחרת תיקון המסתם עצמו לא מספיק(..4

Ballon valvuloplasty –בצנתור, מעבירים בלון דרך המסתם ומנפחים אותו, כך שאיחוי הקומיסורות יקרע שיחמירmild MS)בצורה מבוקרת( והמיפתח של המסתם ישתפר. התוצאות סבירות אך לא מושלמות )נותר

בהדרגה ובעתיד יגיע לניתוח(. היתרון העיקרי הוא בכך שחוסכים את הסיכון הניתוחי )למרות שתוצאותיו הןארוכות טווח(. לא ניתן לחזור על פרוצדורת הבלון פעם נוספת.

Closed mitral valvotomy – טכניקת תיקון מסתם ללא bypass,מכניסים אצבע דרך האוזנית של העליה - ממששים את המסתם ואז מכניסים צנתר דרך דופן החדר, שמרחיב פיזית את המסתם. הפרוצדורה עיוורת, אבל

22

Page 24: עינייםimg2.tapuz.co.il/CommunaFiles/39910353.doc · Web viewגם אם לאחר הפרוצדורה מתפתח PR, הלב הימני סובל את זה מצויין. נדיר

כירוגיית לב חזה

ההיצרות במסתם בהשוואה לצנתור. זו פרוצדורה טובה עם תמותה ניתוחית מעטהנותנת פתרון יותר טוב של מאד, אבל כיום כמעט לא מבוצעת )מאז שנכנס הצנתור(.

,10yS 30%כאשר לא ניתן לתקן את המסתם, צריך החלפת המסתם. התמותה עם מסתם מיטרלי תותב היא ולכן צריך לבחור בזהירות את עיתוי ההחלפה. צריך לשאוף למקסימום תועלת ומינימום נזק, ולזהות את נקודת

הזמן שבה הדרגה התפקודית כבר נפגעת, אבל התמותה ניתוחית לא גבוהה והשינויים הריאתיים הפיכים. הוא בשלבי המעבר בין דרגה תפקודית MVRמתברר שהזמן האידיאלי לניתוח לדרגה 2 )דרגה 3 2bבעיתוי .)

. 4%הזה, ובהיעדר גורמי סיכון נוספים, התמותה הניתוחית היא כ- אם נחכה יותר מדי, יתפתח יל"ד ריאתי בלתי-הפיך. זה מגביר מאד את הסיכון הניתוחי המיידי כי הגמילה ממכונתואזואקטיביים מהעץ יש שחרור של חומרים )בזמן השימוש במל"ר לב-ריאה מעלה את התנגודת הריאתית

ועד מוות. לכן, התמותה הניתוחיתCOהריאתי( וכאשר ממילא יש יל"ד ריאתי זה עלול להביא לירידה דרמטית ב- . מעבר לזה, כל הסיבוכים של יל"ד ריאתי שאינו הפיך ימשיכו להתקיים גם אחרי הניתוח,15-20%עלולה להיות

(. 10yS 30-40%והתועלת של הניתוח תהיה לא גבוהה )

מחלות המסתם האאורטי

עקרונותקלסיפיקציה של מסתמי הלב

עמדה: – 1#חלוקה Atrio-Ventricular )MV, TV( –פתיחה אקטיבית ע"י כיווץ השריר הפאפילרי, וסגירה פאסיבית )סגירת

.leaflet, sub-valvular aparatusיתר נמנעת ע"י המיתרים(. אנולוס, Ventriculo-arterial )AV, PV( – מסתם פאסיבי שכולל רק anulus-ו leaflet.

- מערכת לחצים: 2#חלוקה לחצים נמוכים–( לב ימין PV, TV) לחצים גבוהים–( לב שמאל MV, AV)

האטיולוגיה הנרכשת לפגיעה במסתםרשימה קבועה, הסדר משתנה עפ"י המסתם

)זיהומי )אנדוקרדיטיסRHD)מחלת לב ראומטית( שאינו( דלקתיRHD) דגנרטיבי

איסכמיטראומטיאיאטרוגני)מולד(

מנגנוני נזק למסתם: Regurgitation –העמסת נפח על המדור שקודם למסתם הרחבה של המדור פגיעה סיסטולית

(chamberבתפקוד אותו מדור )Stenosis –העמסת לחץ על המדור שקודם למסתם היפרטרופיה פגיעה דיאסטולית Combined –.)שילוב של איס"ק והיצרות, כשאחד מהם הוא הדומיננטי )לאו דווקא זה שהיה קיים קודם

ישנה פגיעה משולבת, סיסטולית ודיאסטולית.המסתם האאורטי

לבין האאורטה.LVהמסתמים האטריו-ונטריקולריים הם חלק מהאאורטה. המסתם האאורטי ממוקם במעבר בין עלעלים שנקראים בהתאם למוצא העורקים הקורונריים מהם:3המסתם כולל

Rt coronary cusp –העלל ה antLt coronary cusp –העלל ה Lt posNon coronary cusp – העלל ה Rt pos

פאוצ'ים מזנכימיאליים. מומי לב מולדים3 של rotation ו-septationהתפתחות הלב וכלי הדם הגדולים היא ע"י למשל( נוצריםTGA, מומי לב של כלי הדם הגדולים )septation למשל( נגרמים בשלב ה-VSDשל חללי הלב )

23

Page 25: עינייםimg2.tapuz.co.il/CommunaFiles/39910353.doc · Web viewגם אם לאחר הפרוצדורה מתפתח PR, הלב הימני סובל את זה מצויין. נדיר

כירוגיית לב חזה

גורם לכך שהלב השמאלי הוא אחורי )כשפותחים את הפריקרד רואיםrotation. תהליך ה-rotationבשלב ה- מולנו את לב ימין, שהוא קדמי(.

Aortic Stenosisאטיולוגיה

: ASאטיולוגיות אפשריות של senile )degenerative( ASRheumatic – קומפלקסי Ag-Ab.שוקעים בעלעלים ועוברים פיברוזיס שמקשה את תנועתם למעשה, אותו מנגנון כמו ב-–דלקתי לא ראומטי RHD .אבל הטריגר לדלקת שונה תהליך ההחלמה של האנדוקרדיטיס שפגע במסתם. –זיהומי congenital bicuspid valve

בחולה הנתון: ASאטיולוגיה של ה- 55-60מעל גיל = senile )90%( 55-60מתחת לגיל = RHD, congenital

.40, מולד בגילאי 20 היה מופיע בגילאי ה-RHD לבין מסתם מולד ביקוספידי כי RHDבעבר היתה הפרדה בין , הפגיעה המסתמית מופיעה יותר מאוחר. שאר האטיולוגיות הן נדירות ביותר. RHDכיום, עם השיפור בטיפול ב-

פתוגנזה פגיעה דיאסטולית בתפקוד החדר. היפרטרופיהLV מנגנון: העמסת לחץ על

AS( היא מחלה מתקדמת vicious cycle)(: גורם אטיולוגי )כלשהו זרימה מערבולתית פגיעה באנדותל של יצירת פיברוזיס. נסיון החלמה ע"י שיקוע מטריקס חלבוניעלי המסתם

קליניקהטריאדה של סימפטומים:

)תעוקת-חזה( 1) angina )–אנגינה פקטוריס היא ביטוי של פער בין דרישה לאספקה בכל אטיולוגיה. אנחנו המנגנון שונה: יש הגדלה בנפח השריר )עליה בתצרוכת( וזרימהASרגילים לאנגינה בגלל חסימה קורונרית. ב-

שנשארת )לכל הפחות( אותו הדבר. במקרים יותר קשים גם הזרימה נפגעת: הזרימה בקורונריים נקבעת ע"י שעולה(. לכן זרימתLVEDPפרנטי )eפרנטי )ל"ד דיאסטולי סיסטמי, שלא משתנה( לבין לחץ aהפער בין לחץ

(. זוהי אותהV5-V6 בלידים ST-depression )נראה strainהדם בקורונריים מופחתת. באק"ג ניתן לראות סימני כי האנדוקרד הוא האזור הכי נלחץ. במקרה כזה, מתן ניטרו יוריד ל"דsub-endocardial ischemiaתמונה של

דיאסטולי סיסטמי ולכן יקטין עוד יותר את מפל הלחצים והאיסכמיה תוחמר. זה עלול להרוג את החולה. לפני מתןניטרו חייבים )!( לשים סטטוסקופ על החזה כדי לוודא, לכל הפחות, שאין אוושה סיטולית.

שמסוגל לעבור דרך המסתם המוצר. לכן, במאמץ הלב לאCO של upper limit יש ASב-סינקופה במאמץ: ( 2) לפי צרכי הגוף. בנוסף, במאמץ גם יש חימום של שרירי השלד והרחבה של המיטהCOמסוגל להעלות את ה-

הקפילרית כך שגם הדם שיוצא לאאורטה פוגש כלי דם מורחבים ולחץ הדם צונח. האדם מתעלף. יש משמעות ? יש קורלציה למידת ההיצרות(. COלמידת המאמץ הדרוש כדי לגרום להתעלפות )עד כמה הלב מסוגל להעלות

מוגבר( משפיע על הלחץ בעליהLVEDP )Pressure overloadקליניקה של קוצר-נשימה. איס"ק שמאלית: ( 3) אל מערכת כלי הדם הריאתיים. יש עליהbackflow pressureהשמאלית )שהיא בדר"כ שווה ללחץ בחדר( ולכן

)קוים אופקיים בבסיסי הריאה(. זהCurley B linesבלחץ ההידרוסטטי בכלי הדם ודלף של נוזל. בצל"ח רואים מעיד על בריחת נוזל מהקפילרות שמתנקזות אל ורידי הריאה )שנעים מההיקף למרכז הריאה(.

בניגוד לcurly A –קוים מרכזיים שאופיניים לpulmonary HTNבשל מריחת נוזל מה arterial capilaries. (4 )Sudden Death( נגרם ע"י הפרעות קצב חדריות :VT/VFהן נגרמות ע"י איסכמיה וגם ע"י היפרטרופיה לא .)

. re-entryאחידה של השריר שמעוררת לסימפטומים יש חשיבות פרוגנוסטית:

שנים5 תמותה ב-50% –הסתמנות ראשונה של סינקופה / אנגינה שנים. 2 תמותה ב-50% –הסתמנות ראשונה של איס"ק לב

ASדרגת החומרה של דרוג לפי מפל לחצים:

24

Page 26: עינייםimg2.tapuz.co.il/CommunaFiles/39910353.doc · Web viewגם אם לאחר הפרוצדורה מתפתח PR, הלב הימני סובל את זה מצויין. נדיר

כירוגיית לב חזה

0-25 mmHgקל - 25-50 mmHgבינוני - 50-75 mmHgקשה - <75 mmHg - critical AS

תקין.LVמפל לחצים יותר גבוה = מחלה יותר קשה, בהנחה שתפקוד דרוג לפי אינדקס - שטח המסתם חלקי שטח הפנים:

<2תקין - 1.5-2 - ASקל 1-1.5בינוני - 0.8-1קשה - >0.8 - critical AS

הטיפול CHF! חשוב להזהר עם מתן תרופות שמורידות ל.ד. כאשר יש כבר קליניקה של ASאין טיפול תרופתי מתאים ל-

.β blockerנותנים בעיקר הרחבה מילעורית - זמני, קצר טווח. רק במצב מסכן חיים או חולה שאינו מתאים לניתוחתיקון כירורגי.החלפה כירורגית

מתי ננתח חולה ? כשהרווח עולה על הסיכון: + חולה אסימפטומטי + היצרות קשה-קריטיתgood LV function + חולה סימפטומטיgood LV functionהחל מ modarate AS. ירידה בתפקוד החדרי שלEF<50% עושה downgradingשל ההיצרות שמהווה סף לניתוח, כלומר ננתח

גם חולה בהיצרות יותר קלה. הסיבה - עברנו את שלב ההיפרטרופיה של החדר והתחילה דילטציה לאהפיכה.

לחולה שצריך לעבור גםCABGנחליף מסתם החל מ modarate AS. יש ירידה בשטח מפתחsenileניתן לחזות את קצב ההצרות בקרב האתיולוגיות השונות. כך למשל באתיולוגיה

AS ע"פ קצב זה נשאר מתי נגיע לסף הדורש ניתוח ונחליט האם לנתח או לא. לעונת זאת ב0.1cm/yהמסתם של קצב ההתקדמות איטי יותר.RHDעל רקע

כשלעצמו אינו בר ניתוח שכן הניתוח מביא לשחרור בתפקוד הלב.ASמהצד הנגדי אין כמעט מצב בו

Aortic Regurgitationפתופזיולוגיה

volume overload על חדר שמאל הרחבת LV .פגיעה סיסטולית אטיולוגיה

AR :ראשוני - פגיעה ישירה בעלי המסתם RHD זיהומי, ניווני, דלקתי ,AR .שניוני - הרחבה של האאורטה )ולכן הרחבת טבעת המסתם( עקב דיסקציה או אנוירזמה של האאורטה

סימפטומיםסימפטום אחד עיקרי: קוצר נשימה בגלל פגיעה דו-פאזית:

גודש הפרת משוואת פרנק-סטרלינג LVEDP עליית LV פגיעה דיאסטולית - הרחבת (1) חלה ירידה בתפקוד הסיסטולי.ARפגיעה סיסטולית - בשלב מתקדם של (2)ARהיא מחלה פרוגרסיבית לאורך שנים רבות. בדר"כ החולה אסימפטומטי זמן ממושך. הסיבה טמונה בעקומת

פרנק-סטרלינג: עליה בנפח גורמת לעליה בהתכווצות, כך שאין סימפטומים. אבל - כאשר עומס הנפח הוא מאד מאד קשה( אנחנו מגיעים לזרוע היורדת של העקומה, שבה הלב לא יכול להגביר את ההתכווצות כדיARגדול )

לדחוף את הנפח המוגבר )הסיבה - אין חפיפה בין אקטין למיוזין(. רק כשמגיעים לשלב הזה מופיעים סימפטומים. ARשל

ARהטיפול ב-

25

Page 27: עינייםimg2.tapuz.co.il/CommunaFiles/39910353.doc · Web viewגם אם לאחר הפרוצדורה מתפתח PR, הלב הימני סובל את זה מצויין. נדיר

כירוגיית לב חזה

( פועלות כנגד המנגנון של המחלה - הן "מושכות"ACE-i )בעיקר afterload reductionתרופות ל-טיפול תרופתי: . לכן, הטיפול יעיל למדי בשלבים מוקדמים.regurgitation fractionאת הדם קדימה לאאורטה, ומפחיתות את ה-

.Ca channel blocker, β blockerתרופות נוספות הן טיפול כירוגי

האינדיקציות לטיפול כירורגי: ( חולה אסימפטומטי עם עליה פרוגרסיבית בגודל החדרLVEDd>6cm) .חולה סימפטומטי תחת טיפול תרופתי

הטיפול הכירורגי - תיקון המסתם או החלפתו. קשה לתקן מסתם אאורטי דולף )בהיצרות אאורטלית סיכויי הצלחת התיקון יותר גבוהים(. ככלל, קשה יותר לתקן

מסתם אאורטי בהשוואה למסתם מיטרלי, ויש יותר החלפות מסתם. כאשר מחליטים על החלפה, צריך לבחון בזהירות את סוג המסתם שיושתל - מכאני מול ביולוגי. מסתם אאורטלי

שנים(. קצב הבליה יותר מהיר כשיש שחלוף סידן מוגבר10 שנים )במודל חיה הוא מחזיק כ-12ביולוגי מחזיק כ- 3-4)גדילה, אוסטאופרוזיס(, ומושפע גם מה"שימוש" במסתם. לכן, בגיל צעיר המסתם יחזיק הרבה פחות זמן )

שנים בלבד(. לכן, יש סיכוי סביר שיהיה צורך בהחלפה נוספת )בעיקר בצעירים(. מסתם מכאני, לעומת זאת,מחזיק כל החיים אבל דורש אנטי-קואגולציה ויוצר סכנת דמם.

ניתוחי האאורטה העולה

נגדיר קודם אנטומית את חלקי אבי העורקים:Ascending Aorta-מגובה המסתם האאורטלי ועד ליציאת ה - inominate artery .Aortic Arch –מיציאת ה inominate .a. עד ה Rt subclavian .a.Descending Aorta –דיסטלית ליציאת ה Rt subclavian .a.במסגרת שיעור זה המושג יתייחס לרב לאבי .

.החזיהעורקים היורד מצבי חירום:

Intra-mural Hematomaבין ה ופריצה של דם אל לintima: קרע בשכבות הפנימיות של האאורטה media כמו דיסקציה חריפה.–. הדיסקציה לא ממשיכה דיסטלית אלא נוצר פאוץ' של דם. ההתייחסות

אם מאבחנים, ניתוח מיידי. Acute aortic dissectionקרע בשכבות הפנימיות של העורק וחדירת דם בין ה :adventitia ל media.rupture of aneurysm –אאורטה שהיא בבסיסה מורחבת, לרב א-סימפטומטית. לפעמים הרחבה נוספת

עומד להתפוצץ.–גורמת לכאב שמעורר את הבירור. ההנחה קרע של האינטימה והמדיה, כך שדם פורץ אל חלל כוזב שתחום ע"י האדבנטיציה של האאורטה.–דיסקציה

מסוגלת להחזיק את כל לחץ הדם )בעיקרadventitia מכח האאורטה. אך לא תמיד ה80%האדבנטיציה מהווה אם הוא מוגבר(, ולכן יש סכנה לפקיעה של הדיסקציה ודימום למוות.

הקלסיפיקציה של סוגי הדיסקציה השונים מתבססת על מיקום הקרע הראשוני וכן על אורך התעלה בתוך אביהעורקים. משני לסוגי הדיסקציה השונים יתכנו ביטויים קליניים שונים.

קלסיפיקציהIקרע ב - ascending aortaאשר ממשיך דיסטלית וכולל את ה descending aortaהקרע יכול לחתור לאורך קו .

ההפרדה ולעבור לדופן כלי דם קטנים יותר. שם חדירת דם אל דפנות העורק יכולה להביא להצרות הקוטר הפנימי של כלי הדם. משני לכך תיווצר איסכמיה דיסטלית. בנוסף במקומות פיצול של עורקים הדיסקציה יכולה לתלוש את העורקים ממקום חיבורם ולכן שוב ביטי משני לאי זרימת דם באברים דיסטליים באזור אספקת הדם של

העורק. דוגמאות: כלי דם צווארייםסימנים נוירולוגיים adamkevitch .a. ant spinal syndrome עורקים כליליים Acute coronary syndrome

26

Page 28: עינייםimg2.tapuz.co.il/CommunaFiles/39910353.doc · Web viewגם אם לאחר הפרוצדורה מתפתח PR, הלב הימני סובל את זה מצויין. נדיר

כירוגיית לב חזה

בנוסף אם הדיסקציה מגיעה עד חיבור אבי העורקים ללב היא יכולה לחתור תחת המסתם האאורטלי ולגרום לAR.

II –.קרע באבי העורקים העולה אשר אינו ממשיך לאבי העורקים היורד .B בעוד שהשתיים הבאות מסווגות כ Aע"פ קלסיפיקתחה שונה שני הדיסקציות שהוזכרו מסווגות כסוג

IIIa קרע ב - descending aorta.כאשר הדיסקציה חותרת לכל אורכו III קרע ב - descending aorta.מוגבל

גורמי סיכון ואתיולוגיות . בין מחלות רקמתHTN + atherosclerosis + connective tissue diseaseשלושת האתיולוגיות העיקריות הן

. כמו כן קיימים מצביםmaarfan syndrome, Ehler Danlos syndromeהחיבור ניתן לכלול סינדרומים כדוגמת turner syndrome, Iatrogenic. גורמים נוספפים כוללים takayasu arthritis כדוגמת aortaהפוגעים ישירות רק ב

בשל פציעת אבי העורקים ע"י קטטר שמועבר דרכו.קליניקה

מהאנשים הביטוי הוא מוות פתאומי. 40%ב הקליניקה קלאסית היא כאב פתאומי בחזה. החולה יודע בדיוק מתי הכאב החל, הכאב מאד-מאד חזק, עם או בלינגרם ע"י הרחבה אקוטית של האאורטה. עקב כך סיבים סימפטטים סביב בין השכמות. הכאב הקרנה לגב

האאורטה מגורים, והמח מפרש זאת בצורת כאב. Type Aהרבה יותר שכיח מ Type B,הדיסקציה יכולה לכלול את הקשת והאאורטה היורדת וכל עורק אחר .

שמצטבר בחלל הכוזב יכול להיות בלחץ גבוה ולחסום את מוצא העורק. לכן, כל איבר בגוף )חוץ מהריאות( והדםמזנטרית, חסימה בגלל אבדומינלית פרזנטציה להיות יכולה מגוונת: להיות יכולה והקליניקה להפגע, יכול

בגלל עירוב של הקרוטיד, פרזנטציה דמויתCVAפרזנטציה של גפיים בגלל חסימה של הפמורלים, פרזנטציה של MI בגלל עירוב של הקורונריים, איס"ק כליות בשל עירוב renal artery.'וכו

בהתאם לעורקים שנזוקו יכנו ביטויים שונים בבדיקה פיזיקלית. למשל:.דפקים פריפריים אנורמליים בשל הצרות של עורקי הגפייםJVP↑ בשל pericarditis.במקרי דיסקציה פרוקסימלית במיוחד Pulsus paradoxus כאשר נוצר ARUnilateral loss of breading sounds בשל hemothorax

אבחנה - החולים בדר"כ נמצאים בגיל המתאים, והסיפור דומה. MIהאבחנה המבדלת העיקרית היא

אק"ג - יכולים להיות סימני איסכמיה בגלל דיסקציה קורונרית. אק"ג פתולוגי לא שולל דיסקציה. ולא ברורה צל"ח יכול לתמוך, אם רואים הרחבה של האאורטה, אבל הממצא הוא לרב של הרחבה יחסית

מאליה. לכן לצורך האבחנה יש צורך בצילום חזה קודם אשר לרב לא זמין.Angiographyהיא בדיקה מעולה אשר יכולה להדגים בצורה הטובה ביותר כל פגיעה. אולם הבדיקה מלווה באחוז

סיבוכים כולל תמותה גבוהה יחסית ולכן לרב לא בשימוש. , החלקaortic arch לא מדגים טוב את ה US. בנוסף TEEאקו הוא בדיקה מעולה, אבל להדגמת האאורטה צריך

של אבי העורקים שיותר נמוך מגובה הקיבה. חללים באבי העורקים. החלל האמיתי לרב מלא2 עם חומר ניגוד, שבו רואים CTלכן, בדיקת הבחירה כיום היא

בחומר ניגוד. לעומתו בחלל הפתולוגי לרב הזרימה לא טובה ולכן צביעת חומר הניגוד בו חלשה יותר. כך ניתןדרך החלל ואז הזרימה יש מס' קרעים לאורך מסלול הדיסקציה לזהות את החלל הפתולוגי. אולם לעיתים

הפתולוגי מהירה. במצב שכזה לא יהיה ניתן להבחין בין החלל הפתולוגי לאנטומי. )כי העלעלים צמודיםAR רוצים לראות האם יש – TEE, חייבים לעשות גם CTלאחר שעשינו את האבחנה ב-

החלפה או תיקון. רוצים–למדיה, והיא התנתקה מהאדבנטיציה וקרסה פנימה(. זה חשוב לגבי החלטה ניתוחית (. אסור להניח שאםCTגם לראות זרימות, כדי להעריך איפה מתחילה הדיסקציה )לא ניתן לראות את זה ב-

האאורטה העולה מעורבת היא גם המקור. השאלה היכן הקרע הראשוני חושבה ביותר לגבי הגישה הניתוחית כפישיוסבר בהמשך.

טיפול type A

27

Page 29: עינייםimg2.tapuz.co.il/CommunaFiles/39910353.doc · Web viewגם אם לאחר הפרוצדורה מתפתח PR, הלב הימני סובל את זה מצויין. נדיר

כירוגיית לב חזה

לשעה1%הטיפול כאן הוא החלפת הקטע הפגוע בשתל, עם או בלי החלפת המסתם. ללא טיפול, יש תמותה של - תוך חודש. במחצית מהחולים, אחרי תיקון נקודת המוצא של הדיסקציה,25% הראשונות. עוד 48hבמהלך ה

ההמטומה בחלל הכוזב נספגת. במקרה כזה הסיכון לחזרה של הדיסקציה הוא מאד נמוך. בגלל אירועים10yS 50%. בחולים הללו יש רק persistant false lumenבמחצית השניה של החולים נשאר

שנה כדי לוודא שלא1/2 כל CTחוזרים של נזק לאאורטה. כדי למנוע את הסיכון )העצום(, עושים מעקב ע"י שמחמיר. אם החולה יציב, מגדילים מרווחים לפעם בשנה. persistant false lumenקיים

באופן קפדני. תרופת הקו הראשון היא חוסמי ל"ד להוריד β. חוסמי βבמקביל לטיפול הכירוגי חשוב מאד מקטינים גם את קצב עליית הלחץ בכל פעימה ופעימה. קצב העליה ולא רק הלחץ הסופי משפיעים על הסיכון

. 100mmHgהורדת הל"ד בצורה תרופתית היא לפחות אל מתחת ללקרע נוסף ולכן. Type B dissection

לשים ניתן אגרסיבי יותר טיפול רוצים אם ל"ד. הורדת ע"י תרופתי, בעיקר הוא הטיפול stent-graftכאן -2באנגיוגרפיה. הסטנט מצמיד את החלל הפנימי לחלל הכוזב, וחונק את הדיסקציה. אבל: יש סכנה של פרפלגיה )

. העורק יוצא איפשהו בטווח הגובה של.ant. Spinal .a שמספק את ה-.adamkevitch .a( בגלל חסימה של 3%D7-L1אספקה ואין בעורק, יוצא שממנו בסגמנט סטנט שמים אם איפה. בדיוק מראש לדעת ניתן ולא ,

. 9%אלטרנטיבית, תגרם פרפלגיה. הטיפול ה"שמרני" במצב זה נובע מהשיעור התמותה הנמוך במצב זה- ב ניתוחי גדילת הדיסקציה עם הזמן, סימני איסכמיה של אברי מטרה, כאשרtype Bהתוויות לטיפול כולל:

הדיסקציה ממשיכה גם אל אבי העורקים הביטני.מפרצת של האאורטהניתוח אלקטיבי של

צריך לקזז את הסיכון הניתוחי עם הסיכון של המחלה. התברר שיש קשר בין קוטר האאורטה לבין הסיכון לפיצוץ 5 מעל – ס"מ. )באאורטה בטנית 4.5האנוריזמה ותוחלת החיים. כיום ערך הסף לניתוח הוא קוטר האאורטה מעל

ס"מ החלפה פרופילקטית(. ( ואותם ננתח מוקדם יותר. אםCOPDישנן קבוצות חולים שנמצאים בסיכון מוגבר )תסמונת מרפן, גיל מתקדם

, מסתם ביקוספידלי( יותר קל להחליט על החלפת האאורטה.יש חשיבותARיש עוד אינדיקציה לניתוח )למשל לנסיון של הכירורג והמערכת להתמודד עם ניתוחי-לב גדולים. אם המרכז לא מנוסה, כדאי לדחות קצת את

הניתוח כי הסיכון הניתוחי הוא מוגבר.סוגי ומהלכי ניתוח

להחליף את הקטע הפגוע בשתל עם/בלי החלפת המסתם.–המטרה , מעלים על מכונת לב ריאה וחותכיםclampהניתוח נעשה תוך שימוש במכונת לב-ריאה. בשלב הראשון שמים

את האאורטה בחלקה הפרוקסימלי )מעל המסתם האאורטי, אלא אם מחליטים להחליפו(, ובמקומה תופרים את. Dacron graftה-

, להלן( ואז לפתוח אותה דיסטלית עד שמוצאיםDHTCAבשלב השני צריך להפסיק את זרימת הדם באאורטה ) את קצה הדיסקציה הדיסטלית. בשלב זה חושפים חלקים באבי העורקים מהם מתפצלים כלי דם שונים ולכן

. DHTCAהאזורים דיסטליים להם יהיו במהלך הפרוצדורה באיסכמיה מלאה. לצורך כך עובדים בשיטת כעת חותכים את האאורטה לאחר סוף הדיסקציה ותופרים את קצה השתל. כאשר הקשת מעורבת, חייבים להמשיך את הניתוח דיסטלית, לבודד ולתקן את הדיסקציה באזור יציאת עורקי הקשת )לתפור את השכבות זל"ז(

ואז לחבר את מקטע המוצא של עורקי הקשת אל השתל. ולחסוךinominate artery מיד לפני יציאת ה-clampלכאורה, אם הקשת לא מעורבת בתהליך, ניתן לשים את ה- כדי לאפשר ויזואליזציה דיסטליתCirculatory arrestאת הצורך בהפסקת זרימת הדם. אבל: בכל דיסקציה עושים

ולוודא שאין נזק נוסף גם אם נדמה שהפתולוגיה היא רק באאורטה העולה. )בנוסף, התפירה של השתל לאיזור שלהיא פחות טובה(. עושים את המקטע הפרוקסימלי על מכונת לב-ריאה כרגיל, בינתיים מקררים, ואזclampה-

ופותחים את הקצה הדיסטלי. DHTCAעושים בחלק מהמקרים מחליפים את המסתם האאורטי, מה שמחייב קומאדין. זה בעייתי כי מחצית מהחולים שנותחו

יעשו דיסקציה נוספת, ואז הם יגיעו לניתוח חירום כשהם על קומאדין. לכן מעדיפים לשמרacute dissectionבשל ניתן לשים הומוגרפט של מסתם + אאורטה עולה )בעייתי, כי נלקח זו תרומת ולתקן את המסתם. לחילופין

איברים מגופה(. אם ההחלפה כוללת את המסתם האאורטי, צריך לתפור את העורקים הקורונריים אל השתל.

28

Page 30: עינייםimg2.tapuz.co.il/CommunaFiles/39910353.doc · Web viewגם אם לאחר הפרוצדורה מתפתח PR, הלב הימני סובל את זה מצויין. נדיר

כירוגיית לב חזה

צריך להחליף בעיקר את אזור הקרע הראשוני ולוודא שאין אזורי קרע נוספים. אזורי דיסקציה דיסטליים יותר אינם מחייבים ניתוח שכן הדם בדפנות אבי העורקים יספג בכוחות עצמו. זאת כאמור בתלות בכך שכרגע אין איסכמיה בשל לחץ על עורקים משניים שיוצאים מאבי העורקים. במקרה שכזה יש לתקן גם את אבי העורקים

באזור בו יוצאים כלי הדם הסימפטומתיים. . ובנוסף המטופל נותר עם נזק איסכמי לאברי מטרה10-15%סה"כ התמותה בניתוחים מסוג זה גבוהה ומגיעה עד

עקב מיעוט אספקת הדם אליהם במהלך הניתוח.DHTCA - Deep Hypothermia & Total Circulatory Arrest

דקות.30-45 מעלות )מדידה טימפאנית או רקטאלית( מאפשר עצירה של זרימת הדם עד 18קירור מח החולה ל- מעלות ואז נוצר הפרש טמפרטורה בין בגוף לדם והמח מתקרר. הסיכון הוא4טכנית, זה נעשה ע"י קירור הדם ל-

פגיעה במח בגלל שהוא לא מקבל פרפוזיה. גם הכליות נפגעות. שיטות הגנה:

קיצור זמן האיסכמיה - קיים קשר חזק ביותר בין משך האיסכמיה לבין הנזק המוחי, והעקומה עולה באופן.1 דקות. לכן, קיצור זמן האיסכמיה הוא הגורם החזק ביותר שמפחית נזקים מוחיים.30חד סביב

קירור חיצוני של המח - ע"י הרטבתו בתמיסה קרה ו/או ע"י מתן דם מקורר אנטגרדית/רטרוגרדית. .2ברביטורטים - הפחתת הפעילות המוחית ולכן הפחתת התצרוכת..34.Antegrade cerebral perfusion –פרפוזיה סלקטיבית למח ע"י קנולציה של העורקים המספקים את

( פותחהdirectהמח לכן הניתוחי(. בשדה )נמצאות הניתוח את היא שהקנולות מסרבלות הבעיה .) ו- על clampהאפשרות של קנולצית העורק האקסילרי inominateכך שהדם ממשיך אל הקרוטידים

(.indirectומגיע למח דרך מעגל ויליס )5.Retrograde cerebral perfusion –-קנולציה של ה SVCכאשר הוא חסום בכניסה לעליה(. הבעיה היא(

מהדם הניתן מגיע למח, והמתן מוגבל ע"י לחץ )לא8%שהדם הולך רטרוגרדית דווקא ליד ולקרקפת. רק . הדםneuro-protection(. התועלת היא בעיקר בתוספת של קירור וגם במתן תרופות mmHg 23מעל

חוזר אל השדה הניתוחי דרך העורקים. הדם שחוזר זולג החוצה לשדה הניתוחי ונשאב ע"י סקשן אל תוך סקשן ולא במס' רב של קנולות לכל–מכונת הלב ריאה. יתרון גישה שכזו היא שיש צורך במכשיר בודד

עורק ועורק. השימוש בטכניקה הזו היא כאשר הדיסקציה מערבת את קשת אבי העורקים ואז אנו צריכים להפריד

ולא בצורה נפרדת. בצורה זו ניתן להכפיל אתDHTCAולחבר אותם מחדש. השיטה נעשית במקביל ל משך האיסכמיה.

ניטור קפדני של הפעילות המוחית ע"י המרדימים. .6

חזה-אויר (פנאומטורקס)

חזה אויר = אויר בחלל הפלאורלי. מרבית הפנאומטורקס הם ממקור הריאה ודרכי האויר, ולא ממקור חיצוני לגוף.

אטיולוגיה:ספונטני

o .ראשוני-אידיופטי: על-רקע ריאה תקינהo( שניוני: מחלת ריאה ברקעSOL ,זיהום ,COPD).

( נרכשaquired:)o טראומה חודרת או קהה.–טראומטי o למשל עקב –איאטרוגני . כאשר דוקרים יותר מדי לעומק ומחוררים אתcentral lineהכנסת

מהצלעות. גבוה יותר במישור אבל לקלביקולה להיות מתחת אמורים בתקין סיבות הריאה, .ביופסית שדסיבוך של סיבוך של ניקור פלאורלי, נוספות:

obaro-trauma – כאשר חולה מונשם בלחצי PEEP גבוהים פתופזיולוגיה

29

Page 31: עינייםimg2.tapuz.co.il/CommunaFiles/39910353.doc · Web viewגם אם לאחר הפרוצדורה מתפתח PR, הלב הימני סובל את זה מצויין. נדיר

כירוגיית לב חזה

Simple –חור ברקמת הריאה שממנו דולף אויר שמצטבר בחלל הפלאורלי. בשלב מסויים הריאה עוברת תמט ואז החור שממנו דלף האויר נסגר והדלף נפסק. יש תמט של חלק מהריאה או כולה. זהו מצב יציב. אין שינויים המודינמיים )לכל היותר יהיה שאנט בגלל התמט, אבל אז יש גם ואזוקונסטריקציה ולכן החולה בדר"כ לא יהיה

.simpleהיפוקסי(. יכולים להיות מעט שינויים נשימתיים. לא מצב מסכן חיים. מרבית הפנאומטורקס הם Tension –גדול, גם לאחר התמט של הריאה עדיין יש דלף. מצטבר המון אויר בחלל כאשר החור בריאה

הפלאורלי, תחת לחץ. הסרעפת תהיה מושטחת ודופן בית החזה מעט מורמת, אבל כל אלו זניחים לעומת סטיהלירידה ולכן הורידי גורמת לפגיעה בהחזר זו הנגדי. סטיה וכלי הדם הגדולים( לצד של המדיאסטינום )הלב

בתפוקת הלב ולהתפתחות שוק שיכול להיות קטלני. מהירות ההתקדמות היא משתנה. Open – תמיד נרכש, בדר"כ טראומטי. נקרא גם sucking woundישנו מעבר פתוח בין החלל הפלאורלי לעולם .

החיצון )פתח בדופן בית החזה(, ובעת האינספיריום נוצר לחץ שלילי ששואב אויר לתוך החלל הפלאורלי. ככל שהפתח בדופן בית החזה יותר גדול, כך יש יותר "שאיבה". חזה אויר פתוח לא יהיה בלחץ, כי אויר יכול להכנס ולצאת. המצב יכול להתרחש במנגנון איאטרוגני כאשר יש צינור שמתנתק מנקז חזה. הטיפול הוא לסגור את

הפצע החיצוני וגם להכניס מנקז פלאורלי. חזה אויר ספונטני

Spontaneous Pneumathorax 25-15 - מחלה שכיחה למדי. גילאי ,M:F=8:1מנגנון

primary spontanous pneumathorax נגרם ע"י בקיעה של blebsבפסגות הריאה )על רקע של ריאה בריאה סגירה )מכאנית( של החור תמט של הריאה הצטברות אויר בחלל הפלאורלי דלף של אוירלחלוטין(

ריפוי של החור ע"י רקמת חיבור כעבור מספר ימים. , כי התמט של הריאה מונע הצטברות כמויותtension pneumathoraxחזה-אויר ספונטני הוא בדר"כ פשוט ולא

.tensionגדולות של נוזל שייצור לחץ מוגבר. אם החור מאד גדול, יכול להיות קליניקה

גירוי של הפלאורה הויסצרלית בגלל קרע של –כאב חד ופתאומי blebהחמרה של הכאב בנשימה . .full expirationעמוקה )בגלל תנועת הפלאורה( לא יכול לעשות

)סטורציה שמורה )חולים צעירים ובריאים, עם ריאות תקינות.)קוצר נשימה קל )לא תמיד

אבחנה הדיות בניקוש, פרמיטוס מופחת. אם יש↑פנאומטורקס קשה מסתמן ע"י ירידה בכניסת אויר לאזור התמט,

יהיו גם ממצאים המודינמיים של שוק, סטיית מדיאסטינום. –פנאומטורקס בלחץ , בו רואים ריאה שאינה מתפשטת עד לגבול כלוב הצלעות )ישצל"ח אקספירטוריהאבחנה הדפנטיבית היא ע"י

אזור שחור ללא שום ציור ריאתי, שהוא האויר הכלוא בחלל הפלאורלי(. מקובל לומר ש"חזה אויר בלחץ הוא אבחנה קלינית, שלא מצריכה צילום חזה". זה נכון, אבל כשנמצאים בבי"ח

)ולא בשטח( בהחלט מוצדק לצלם את החולה )בפרט בהסתמנות לא קלאסית(.טיפול

חזה אויר = ניקוז !!!נקז בין צלעי

בפרוצדורה תחת הרדמה4-5ישנן טכניקות שונות להחדרה של נקז בין-צלעי. הנקז מוחדר למרווח בין-צלעי המשיכה. כח עפ"י וניקוז למים מתחת הנקז קצה הכנסת ע"י חד-כיווני, לשסתום מחובר הנקז מקומית. באינספיריום, המים עולים בצינור בגלל לחץ שלילי תוך-חזי. באקספיריום, המים יוצאים מהצינור בגלל הלחץ

10חיובי. אם יש עדיין דלף של אויר, נראה בועות בזמן האקספיריום. חשוב להקפיד שתהיה כמות מים בינונית )יוכל ס"מ, לא יותר מדי או פחות מדי( כדי שבאינספיריום לא ישאבו מים לתוך החזה, ובאקספיריום החולה

להתגבר על עמוד הנוזלים ואויר שדלף יוכל לצאת החוצה. בנוסף לכח המשיכה, במכשירי בי"ח ישנה שאיבה אקטיבית בלחץ שלילי נמוך, שמטרתה למשוך החוצה אויר או

, כדי שלא להזיק למנגנוןcmH2O 25נוזל שנמצאים בחלל החזה. ישנה בקרה שהלחץ השלילי לא יעלה על הנשימה או לגרום לסטיה של המדיאסטינום.

30

Page 32: עינייםimg2.tapuz.co.il/CommunaFiles/39910353.doc · Web viewגם אם לאחר הפרוצדורה מתפתח PR, הלב הימני סובל את זה מצויין. נדיר

כירוגיית לב חזה

לאחר הכנסת המנקז, הריאה מתנפחת מחדש, ולכן יש סיכון מסויים שהחור )שקרס( יפתח מחדש. לכן, המנקז שעות, אם אין דלף, מוציאים את הנקז.24 שעות לפחות, כדי לוודא שאין דלף. כעבור 24מושאר במקומו למשך

. כעבור מספר שעות עושים צל"חunder-water אך משאירים את הנקז suctionבשלב הראשון מנתקים את ה- מוציאים את הנקז. –כדי לוודא שאין הצטברות מחודשת של אויר. אם הצילום תקין

ימים ואם אחריהם עדיין יש דלף,5-7 מהחולים הדלף לא יסגר מעצמו. במקרה כזה משאירים את נקז 10%אצל צריך לסגור את החור בחדר-ניתוח.

Needle aplication.tension pneumothoraxטכניקה מקדימה לשחרור מיידי של לחץ במקרה של

המטרה - לשפר מצב המודינמי, עד שמגיעים למתקן רפואי שבו ניתן להכניס נקז בין-צלעי. ב-)ונפלון( הכנסת מחט הטכניקה: שני, צלעי בין mid-clavicularבמרווח lineלמיקום נוספות ציון נקודות .

שממוקם בחיבור בין המנובריום לסטרנום.angle of louisהפרוצדורה: לטראלית ל- זוהי פרוצדורה שיכולה לציל חיים, אבל ניתן גם לגרום לסיבוכים קשים אם אין אינדיקציה או מיומנות. לכן אם

נמצאים בבי"ח, ולא במצב של החייאה, רצוי לקרוא לכירורג מיומן ולהכניס ישר נקז בין-צלעי.מתי לא ננקז?

כאשר יש חזה-אויר מאד קטן, ללא קליניקה, בחולה מאושפז, ניתן לעקוב אחריו. .1אם החולה נשאר יציב ניתן לחזור על הצילום, לעקוב קלינית. .2

במקרים אלו הסיכון עולה על התועלת האפשרית. הנחיות בשחרור

מומלץ להמנע מפעילות גופנית למשך שבועיים, כדי לאפשר החלמה מלאה של החור. אח"כ ניתן לחזור לחייםתקינים ומלאים ללא הגבלות.

מומלץ להמנע משינויי לחץ גדולים )כמו צלילה או טיסת מסוקים( למשך מספר שבועות. אין הגבלה על טיסות סילון מסחריות כי לחץ האויר במטוס הוא מאוזן. למרות זאת, חברות הטיסה בדר"כ ידרשו להתרחק מהאירוע

לפחות שבועיים לפני הטיסה. פרוגנוזה

שנים. אם יש חזרה אחת, הסיכון לחזרה נוספת מאד3 סיכון לחזרה בתוך 20-30%בפנאומטורקס ספונטני יש גבוה.

, וממליציםblebs-חזה עם חתכים דקים בפסגות, כדי לראות את היקף ה-CTכאשר יש אירועים חוזרים, עושים של פסגת הריאה, באזור שבו רואים את ה-wedgeעל ניתוח לכריתת האזור הבעייתי )טורקוטומיה שבה כורתים

blebs שפשוף מכאני של הפלאורה שגורם לפיברוזיס והדבקות שכבות–(. באותה הזדמנות גם עושים פלאורודזיס הפלאורה זו לזו.

מחלת לב איסכמית

יורדת בגלל מניעה יותר טובה. אולם האבחון והזמינות של הטיפול יותר גבוהות, ולכן ישIHDהשכיחות של אשליה של עליה בשכיחות.

גורמי הסיכון מגביר סיכון למחלה קורונרית ושבץ. איזון ל"ד מפחית את הסיכון ל-–יל"ד IHD.)לפי מחקר פרמינגהם(

חזק. EBM. יש לכך mild HTNמוצדק לטפל אפילו ב- מגביר אתרוסקלרוזיס, גורם ל-–עישון IHDאין עדויות טובות שהפסקת העישון מתקנת את הנזק שכבר :

נגרם, אבל יש עדויות טובות שהתקדמות המחלה נעצרת. .סיפור משפחתי סוכרת דיסליפידמיה

גיל מין השמנה

סימפטומים כאב בחזה– angina pectoris :

31

Page 33: עינייםimg2.tapuz.co.il/CommunaFiles/39910353.doc · Web viewגם אם לאחר הפרוצדורה מתפתח PR, הלב הימני סובל את זה מצויין. נדיר

כירוגיית לב חזה

o)קשור במאמץ )אבל יכול להיות משני לסטרס נפשי או להופיע במנוחהo הקרנה לגב היא קלאסית(.–מקרין ליד שמאל, ללוע, לאפיגסטריום. )בנשים oכאב לוחץ, רטרו-סטרנלי

קוצר נשימה: מלווה את הכאבים בחזה. לחלופין יכול להיות מצב שנקראAngina equivalent –קוצר ירוד. LVנשימה ללא כאבים בחזה, שאופייני לחולים עם תפקוד

Silent MI – ארועי MI .ללא סימפטומים. אופייני לחולי סוכרת, חולים ממוצא אסיאתיבדיקה פיזיקלית

יכולה להיות –לא מאד מועילה )בניגוד לאנמנזה( IHDקשה ביותר, ובדיקה פיזיקלית תקינה לחלוטין. אולם , משקעי שומן, יל"ד.PVD אוושה קרוטידית, סימני –לעיתים כן ניתן להפיק ממצאים תומכים

בדיקות עזרבדיקות דם:

CBC –אנמיה קשה יכולה לגרום לסימפטומים קורונריים עקב הגברת ה CO. בעיקר –אלקטרוליטים( הפרעות אלקטרוליטיות K.גורמות לאנגינה ) כחלק מהערכת גורמי סיכון. )הערכה אמבולטורית, לא בחדר מיון(. הבדיקה נעשית–פרופיל שומנים

שעות. החולה יכול לשתות נוזלים צלולים )מים, תה ללא סוכר או חלב( )לא מיץ12בצום-אכילה של (.cyt P-450אשכוליות כי הוא מעכב

בדיקת –גלוקוז בצום screening..לסוכרת כחלק מהערכת גורמי סיכון ,טרופוניןCPK – מועילים רק בחולה שעובר MI.בטווח של שעות

. יש לשים לב לפרטים הבאים:lead ECG-12 במנוחה, – אק"גRate –-אין ממצא ייחודי ב IHD .חשוב לשים לב לבראדיקרדיות / טאכיקרדיות .Rhythm: AF נגרם בין השאר ע"י IHD . מורפולוגיה: גליQ שמעידים על MI ישן או MI בהתהוות. שינויי STבמנוחה בדר"כ יהיו כרוניים ולא

מחודדים מעידים על העבר. Tבעיה בהתהוות. גלי אק"ג טוב מאד להערכת העבר ועוזר בהערכת ההווה, לא העתיד.

CXR –( לאיתור ממצאים ריאתיים COPDסרטן ריאה, גרורות(, הגדלת לב כעדות ליתר ל"ד כרוני. בדיקה , חשובה ברמה הבסיסית, לפני בדיקות עזר מורכבות.

- מבחן מאמץ ומד ל"ד. עושים רישום במנוחה, ואז החולה מתחיל לרוץ עלlead ECG-12החולה מחובר ל- trademill לפי פרוטוקול Bruce,הפרוטוקול מותאם למצב הקליני של החולה. במהלך הריצה בודקים סימפטומים .

, אריתמיות(. הספציפיות של הבדיקה היא טובה, אך הרגישות פחותה.STתגובת ל"ד למאמץ ושינויי אק"ג )שינויי (. FPהמבחן לחלוטין לא אמין בצעירים ללא גורמי סיכון )הרבה

baseline של חומר רדיואקטיבי, צילום במצלמת גמא מיד לאחר ההזרקה שמשמש כ-IVהזרקה - מיפוי תאליוםומכאן שפגמי מילוי4וצילום חוזר כעבור שעות. קליטת החומר בשריר הלב מושפעת מאספקת הדם אליו,

שעות ואם פגם הקליטה נעלם ניתן להניח כי24מעידים על בעיה באספקת הדם. ניתן לחזור על הצילום כעבור האזור המדובר הוא אזור איסכמי שיהנה מרפרפוזיה. ניתן לשנות את הבדיקה ע"י הוספת מבחן מאמץ או מתן

דובוטמין. כלומר משווים את הקליטה בטריטוריות–עם זאת יש לזכור את חסרונות הבדיקה. הבדיקה היא השוואתית

שונות של העורקים. אם יש איסכמיה באיזור אחד ולא באיזור אחר, ניתן יהיה לראות פגם מילוי באזור היותר חסום. אולם אם כל האספקה פגועה באותה מידה, התאליום יכול להראות תקין. תליום יכול להיות מאד מועיל

.PCTAאחרי שיודעים את האנטומיה כלומר ניטור לאורך זמן לאחר . ניתן גם לבדוק תפקוד גלובאלי, מסתמים. ניתןregional wall motion abnormalitiesבעיקר מתעניינים ב- - אקו

בחולים שלא יכולים לבצעstress-testלשכלל את הבדיקה ע"י מתן דובוטאמין וכך לייצור בדיקה אשר מחליפה hibernation )הערכת very poor LV functionאת מבחן המאמץ מסיבה פיזיתפיזית לעשות מאמץ, בחולים עם

של המיוקרד(, ובחולים שאצלם מבחן מאמץ לא אמין )נשים(. להערכת מחלה איסכמית. )במקרים של חשד קליני משמעותי, נלך לצנתור בלי לעבור אתgold standard - צנתור

שלבי הבירור הקודמים(.

32

Page 34: עינייםimg2.tapuz.co.il/CommunaFiles/39910353.doc · Web viewגם אם לאחר הפרוצדורה מתפתח PR, הלב הימני סובל את זה מצויין. נדיר

כירוגיית לב חזה

Coronary CT - ( הזרקת חומר ניגוד פריפריתCT-angioוצילום סדרת תמונות של הלב. הדרישות: חתכים דקים ) יש )כיום כך שמצלמים תמיד בשלב של ECG-gating(. 64וריבוי מצלמות mid-systoleויחסית )הלב מלא

כפולה לפחות בהשוואה לצנתור, אבל כמות חומרCT. הקרינה הניתנת ב-βסטאטי(, האטת הקצב ע"י חוסמי- אולם קצת פחות טובה ללזיותsoft atheromaהניגוד פחותה. בדיקה זו טובה במחלה פרוקסימלית ובמקרה של

מרוחקות ולא כ"כ טוב לעורקים מסויידים. צריך הרבה מיומנות לפרשנות של הבדיקה. בעתיד ככל הנראה יהפוך.gold standardל-

PET-scan בדיקה טובה מאד להערכת - hibernation.של שריר לב acute MIתמותה וסיבוכי

חלוקת גורמי התמותה היא ע"פ זמן הופעתם.מיידיהפרעת קצב קטלנית. זהו גורם המוות המיידי השכיח ביותר. הביטוי הוא "דום לב". עיקר ארועים אלו

.RCAהנם משניים לחסימת הCardiogenic shock – עקב אי תפקוד סיסטולי של חדר שמאל. אופייני לאוטם של proximal LAD.

earlyבשלב הקרוב של מס' שעות אחרי האוטם שכיחים בעיקר סיבוכים משניים לטיפול בארוע:

דימומים. משני לטרומבוליזהReperfusion injuryבעת החזרת זרימת הדם לאזורים האיסכמיים נוצר שינוי אלקטרוליטרי מהיר, כמו .

כן יש מעבר של רעלנים מהאזור הנמקי לאזורים סמוכים. משני לכך יופיעו הפרעות קצב קטלניות.בטווח של שעות עד שבוע לאחר הארוע:

Free wall rupture –בעיקר נפגעות הדפנות ה lat + ant.הטיפול ניתוחי .acute MR – עקב קריעת pos papillary muscle אופייני לחסימת .RCAהביטוי הוא של בצקת ריאות .

מיידית. Acute VSDל גורם .acute pulmonary edema ע"י הוא המצב אבחון .USומדידת ימני צנטור /

סטורציות. יחד עם הנשמה בלחץ חיובי. ההנשמה בלחץIABPהטיפול בשני האחרונים עד לתיקון הניתוחי הוא הכנסת

יוצרIABPחיובי מקטינה את מפל הלחצים על פני העץ הריאתי ולכן פחות דם יכנס למחזור הריאתי. במקביל ה תת לחץ בשלב הסיסטולי של מחזור הלב ובכך מביא לתנגודת פריפרית נמוכה. מאחר שהדם מהחדר השמאלי ילך אל החלל בו התנגודת נמוכה יותר, ע"י שתי פעולות אלו הגדלנו את נפח הדם שילך למחזור הסיסטמי ובכך

הקטנו את עומס הנוזלים על הריאות..3d תוך 80%שלושת הסיבוכים מחייבים ניתוח בהקדם שכן ללא ניתוח התמותה בבהם מגיעה ל

Late פגיעה בתפקוד סיסטולי CHF.Late ischemic MR – לאחר האוטם דופן החדר נהיית דקה יותר והחלל מתרחב. משני לכך annulus

המסתם מתרחב ונוצרת דליפה דרכו.הפרעות קצב סביב הצלקת. טיפול ע"י אבלציה אנוריזמה של דופן החדר. מאופיין בגליQ, ST elevation ב V1-V6קליניקה של קשיי נשימה. הטיפול .

כריתה ניתוחית של האזור הפגוע.

CABGניתוח מעקפים -

CABGטיפול שמרני מול מחקרים להשוואה בין טיפול רפואי3 נעשו 70. בשנות ה-60ניתוחי המעקפים הראשונים בוצעו בסוף שנות ה-

.CASS – coronary artery surgery study. המחקר הקלאסי הוא CABG)בעיקר חוסמי בטא( ל-

33

Page 35: עינייםimg2.tapuz.co.il/CommunaFiles/39910353.doc · Web viewגם אם לאחר הפרוצדורה מתפתח PR, הלב הימני סובל את זה מצויין. נדיר

כירוגיית לב חזה

)כיום זה בחיים לא יתקבל!(, רק 15%במחקר השתתפו מרכזים שבהם התמותה ניתוחית היתה פחות מ- 8% )כיום LIMAמהמנותחים קיבלו שתל של כמעט כולם(, כמעט כל החולים היו גברים, ולכמעט כולם היתה–

stable angina ולא( UAP או post-MI )– זו אוכלוסיה שהיא מאד pre-selected המחקר היה .prospective RCT , ולכן היתה בעיה קשה בניתוח סטטיסטי של הנתונים. 24% בשנה הראשונה היתה cross overאבל שיעור ה-

: CABG מצבים שבהם היתה אינדיקציה ברורה ל-3למרות כל המגבלות, היו מחלה תלת-כלית בנוכחות פגיעה בתפקודLV מעורבותleft mainprox LAD.)בעצמו או בקומבינציה(

ופחות שימוש בטיפול תרופתי. הנתונים הללו נצפה שיפור בתוחלת החיים, בסימפטומים באינדיקציות הללו שנה אח"כ, כדי להגדיר את המושג של 30רלונטיים עד היום, prognostic diseaseבמצבים הללו, גם ללא .

סימפטומים כלל, הניתוח מועיל לחולים והוא מוצדק!PCI מול CABG

פורסם ב-RITERמחקר )עם 1992 follow-up יותר מאוחרים(. שוב, מחקר על אוכלוסיה שהיא מאד pre-selected.)הקריטריונים לא תקפים יותר(

אף פרוצדורה לא יותר בטוחה מהאחרת לטווח ארוך. – הקבוצות 2לא היה הבדל בתמותה בין נוספת(, בקבוצת ה- )הצורך בהתערבות נוספת בתוך שנתיים.CABG, 11%לגבי תחלואה נזקקו להתערבות

מהחולים נזקקו לטיפול אנטי-אנגינוטי )הבעיה היתה80%, מעל PCI. בזרוע ה-40% השיעור היה PCIבקבוצת ה-re-stenosis-בזרוע ה PCI .)

Stent מול CABG.CABG בזרוע ה-11% בזרוע הסטנט מול 30%התמותה בשתי הקבוצות דומה ; התחלואה -

לגבי סטנטים מצופי-תרופה עדיין אין מחקרים ארוכי טווח. מהתוצאות הראשוניות, שמבוססות על תקופת מעקבש- נראה ל-DESקצרה, בהשוואה התמותה את מכפילים CABGהשוויון רגילים, סטנטים על )במחקרים .

בתמותה מתייחס לכל תקופת המחקר ולא לטווח קצר(. שניהם טיפול בטוח ומוצלח, בבחירה נכונה של החולים. PCI with stent וגם CABGמסקנה: CABGל- ותאינדיקציסיכום

, אבל במציאות, מגיעים לניתוח בעיקר חולים שבהםAHA/ACC guidelinesע"פ המחקרים שהוזכרו נקבעו (.PTCAהאנטומיה לא מאפשרת פתיחה צינטורית של החסימה )בכל שאר המקרים, הקרדיולוגים יתאמצו לבצע

אינדיקציות מקובלות: 1)L.main > 50% כאן .CABG .הוכח כמאריך חיים PCI -יעשה רק במסגרת התערבות חירום ב MI.2)Proximal LAD + Circumflex מצב זה מכונה .LMA equivalent3)Proximal LADהסיבה שכאן יש עדיפות לניתוח הוא החשש שמא ה .stentיבלוט אל ה LMAואז טיפול

באזורי המחלה כאשר זה מצב לעומת ראשי. עורק של למחלה יביא משני שאינםLADבעורק .PCI על פני CABGפרוקסימלים )כלומר ע"פ הגדרה לאחר הענף הספטלי הראשון( לא הוכח יתרון ל

. prox. LAD כאשר אין עירוב של CABGמחלה דו-כלית עם איסכמיה נרחבת. אולם לא הוכח יתרון ל(4מחלה תלת-כלית. בחולים אלו חייבים לעשות רה-וסקולריזציה. אולם רק בחולי סכרת הודגם עדיפות ל(5

CABG על פני PCI.6)Unstable AP או MI שלא מתאימים לביצוע PTCA - ניתוח מעקפים דחוף.PTCAחולים שמסתבכים במהלך (7

וגם לניתוחCABG משולב בניתוחי חזה אחרים. במצבים אלו ההתוויה לניתוח הCABGבנוסף ניתן לבצע ניתוח החזי השני יכולים להיות יותר רכים מאשר ההתויות לכל אחד מהם בנפרד. דוגמאות:

MR + IHD במצב בו .IHD -הביאה לנזק איסכמי לשריר הפפילרי ולכן ל MRכאשר הדליפה מוגדרת כ . moderate to severה ידי הקטנת על נחליפו( )ולא נתקן את המסתם annulusועיצוב של המסתם

.MR כדי לטפל בגורם לCABGהמסתם מחדש. במקביל נעשה גם AS + IHD נחליף את המסתם כבר בהצרות .moderate שטח מתחת ל( 1.5cm2ניתוח ה ( יחד עם

CABGשכן ל .AS.יש נטיה לעבור התקדמות עם הגיל שיקולים בביצוע הניתוח

34

Page 36: עינייםimg2.tapuz.co.il/CommunaFiles/39910353.doc · Web viewגם אם לאחר הפרוצדורה מתפתח PR, הלב הימני סובל את זה מצויין. נדיר

כירוגיית לב חזה

<(70% ; עורק אחר LM>50%היכן קיימת היצרות משמעותית ? )היצרות משמעותית מוגדרת כ.12.graftable artery מ"מ הוא בר-השקה. )הערכה לפי צנתור, ההחלטה בניתוח(.1 ? קוטר מעל באיזה שתלים נשתמש ? .34.LV function –ככל ש LV↓.כך החולה יותר יהנה מהניתוח מבחינת שרידות, תפקוד סיסטולי ככל שהקליניקה קשה יותר כך החולה ירוויח יותר מהפרוצדורה.–קליניקה .5

:viableסימנים לכך שדיסטלית לחסימה יש שריר .הופעת כאב במאמץ - מלמד על רקמה ויאבילית הסובלת מאיסכמיה ולא עברה נמק מלא מבחןthaliumשריר עבה )ולא דק ופיברוטי( בecho

מקרים לדוגמא ): 1מקרה (. מבחן מאמץ לא חד-משמעי. בצנתור:SAPאיש עם היסטוריה שמתאימה מחלת לב איסכמית

. ההתלבטות הטיפולית היא בין "כלום" לבין "תרופות" )אספירין,RCA ב-prox LAD, 20% ב-30%חסימה של מניעה שניונית לגורמי סיכון(.

. RCA של LM, 100% של 98% קלאסית. בצנתור: stable angina, 55בן : 2מקרה בעורקים גדולים )ללא70-80% –, ריבוי היצרויות classic stable angina, מופיע עם IDDM, 55בן : 3מקרה

LM היצרות בעורקים דיסטליים.99%(, עד : אין החלטה ברורה בין "כלום" לבין מניעה שניונית.1מקרה ניפוח בלון–, יש עדויות שהניתוח מועיל. )יש גם בעיה טכנית בצנתור PCI: אין מידע שתומך בביצוע של 2מקרה

מלא(.MIבעורק אחד יוצר : חולים עם סוכרת מרוויחים יותר מניתוח.3מקרה

בחירת השתליםהשתלים האפשריים:

ורידיים: בדר"כ greatשתלים saphenous veinורידים אחרים אבל הם פחות טובים(. לקיחת )ניסו . שתלים ורידיים אינה פוגעת בתפקוד האתר התורם )הרגל(. יתכן סטזיס ורידי.

:שתלים עורקייםניתן לקחת מעקף עורקי רק אם יש קולטרלים, או שהעורק לא משמעותי במיקומו המקורי.

oRt / Lt Internal Mammary Artery )IMA( חופשי או( in-situ)oRadial artery אפשרי רק בתנאי שיש קולטרלים לאספקת הדם ליד. כלומר .allen test.תקין oInferior Epigastric arteryעורק שמזין את שריר הרקטוס אבדומיניס. מוגבל באורכו )מאפשר -

דמוי ורידים ולכן השימוש הופסק.patencyרק מעקף קצר(. oGastro-Epiploic arteryהעורק על העקומה הגדולה של הקיבה. השימוש בו מחייב פתיחת -

(. יכול לספק את הדופן התחתון של הלב. התחלואהin-situהבטן והעברת העורק דרך הסרעפת )של לקיחת העורק הזה לא מצדיקה את הפוטנטיות, ולכן השימוש בו הופסק.

.אין שתלים סינטיים - כל הנסיונות נכשלוLIMA

ל-LIMA. הפוטנטיות של gold standard שנים, וזה ה-10 פוטנטיות ל-90% יש מעל LIMA to LADלמעקף marginal דומה לזו של - LIMA to LAD-לא מחברים אל ה .circumflex-עצמו, כי הוא נמצא ב AV grove.

LIMA-לא יכול להגיע ל RCA כשתל in-situ. RIMAכי יש רק השקה כירוגית אחת. השימוש של free graft יותר טוב ומחזיק יותר שנים מ graft in situככלל

כנראה מועיל בעיקר בצעירים )עדיין לא הוכרע(. LIMAבנוסף ל-: LIMAסיבות לא להשתמש ב-

)מחלה טרשתית של העורק )נדיר, לא ברור מדוע-היצרויות ברמת המוצא מהsubclavian-שחוסמות את זרימת הדם ל LIMA פגיעה טכנית בעורק ע"י המנתח-היצרות של הLIMA-לאחר המוצא מה subclavian הפרעה בריפוי פצע

35

Page 37: עינייםimg2.tapuz.co.il/CommunaFiles/39910353.doc · Web viewגם אם לאחר הפרוצדורה מתפתח PR, הלב הימני סובל את זה מצויין. נדיר

כירוגיית לב חזה

-הLIMA-מספק את ה vertebral artery-ולכן שימוש ב LIMA כקורונרי יוצר subclavian stealשפוגע במח.

RIMA .marginal(, לא יגיע ל-ramus )או ל-LAD ו-RCA כמעקף ל-in-situיכול לשמש

,LAD פחות פוטנטי בהשוואה למעקף ל-RCA שנים. מעקף ל-10 ל-80-90% היא RIMAהפוטנטיות של מעקף עם הזמן מתפתחת חסימה דיסטלית למעקף ואז האנסטומוזה עובדת מול לחץ מאד גבוה והיאLADכי במעקף ל

שם לחץ הזרימה )מול הלחת התוך חדרי של חדר ימין( נמוך יותר.RCAנסתמת. מצב זה יותר נדיר במעקף ל-Radial Artery

לאחר ההשקה, ולכן לבעית זרימה דרך המעקף. reactive vasoconstriction יש נטיה ל-IMAלעורקים שאינם למרות זאת: דיסקציה נכונה של העורק + שימור נכון עד ההשקה + תרופות לדילטציה עורקית = פוטנטיות טובה. לכן, לאחר לקיחת העורק משרים אותו בתמיסת חוסמי סידן או ניטרטים עד ההשקה. ובנוסף לאחר הניתוח

.POנותנים לחולה את אותם סוגי תרופות ( Rt>Lt. לא ברורה משמעות מיקום ההשקה )כנראה 80-85% שנים היא 5הפוטנטיות ל-

ורידים מהאנסטומוזות נסתמות10%. כאשר 10y לאחר 30% שנים, 5 מהמעקפים הורידיים פתוחים כעבור 50%רק תוך שנה.10% שבועות )קרישים(, עוד 6-4תוך

הגברת פוטנטיות הפוטנטיות של השתלים מאד תלויה בטיפול אנטי טרשתי מתאים: כל חולה לאחר ניתוח מעקפים מקבל אספירין

.IHDוסטטינים. בנוסף - מניעה שניונית של כל גורמי הסיכון ההפיכים של )בחסימה מוקדמת זה לא תמיד אפשרי, כי מנגנוןPTCAבמקרים רבים, שתלים שנסגרים יוכלו להפתח ע"י (.PCIהחסימה הוא טרומבוס ולא פלאק שניתן לטפל בו ע"י

אז אילו מעקפים עושים למעשה? 40% הוא העורק הכי חשוב שעובר רה-וסקולריזציה, והוא מספק LAD: ה-LIMA to LADתמיד יבוצע מעקף

(. לכן, שם נשים את המעקף הכיLAD כשהוא לא צריך רה-וסקולריזציה של CABGמהחדר )נדיר שחולה יגיע ל-פוטנטי שניתן לעשות.

נעשה LIMA to LADהשימוש ב- gold standart-השתלים הנוספים, באותו שלב, היו ורידיים(.80 בשנות ה( . זה משמעותי בעיקרLAD לעומת וריד ל-LAD ל-LIMA עדיף ב-long term survivalבהמשך התברר גם שה-

חייבים לתת הסבר מפורט לסיבה.LIMA שנים, כאשר הורידים נכשלים. כיום, אם לא שמים 10מעבר ל-Complete Arterial Re-Vascularization .-ש התברר עלתהLIMAכאשר טווח, ארוכת השרדות משפר

לעשות הצדקה יש האם דבר. אותו יעשו נוספים עורקים שתלים האם Completeהשאלה arterial revascularization אין עבודות ? RCA-שדנות בנושא, אבל יש מידע רטרוספקטיבי נרחב )החל מאמצע שנות ה

(. העבודות לא הוכיחו כי שימוש בשתלים עורקיים נוספים משפר את ההשרדות )קשה להוכיח זאת, כי ה-90LIMA של בבדיקה אבל משמעותי(, כ"כ endpoints-ב צורך התעוקה, )חזרת אחרים PTCAטיפול ,

אנטי-אנגינוטי( כן הוכח שיפור ע"י ביצוע מעקף עורקי מלא. עם זאת, למעקף עורקי מלא יש מחיר:

בביצועdouble mammary( LIMA-ו RIMA .יש סיכון של זיהום הסטרנום, בעיקר בסכרתיים ) בלקיחת עורק רדיאלי יש סיבוכים מקומיים של היד

לעומת הסיבוכים ההבדלים בתמותה ניתוחית ותחלואה אינם משמעותיים. בנוסף, במעקף עורקי מלא הניתוח נעשה יותר מורכב. כיום: ככל שהחולה יותר צעיר, זה יותר מוצדק לעשות רה-וסקולריזציה עורקית מלאה. מתוך

תקווה שעורקים יחזיקו יותר זמן מורידים.לסיכוםבכל חולה יהיה שימוש בLIMA.בחולים צעירים תהיה העדפה לשימוש גם בRIMAאלא אם כן מדובר בחולה סכרתי. במצבים אלו יש

. .radial .aעדיפות מסויימת לשימוש ב.בחולים מבוגרים המעקפים המשניים יעשו ממעקפים ורידיים

גורמי סיכון קדם-ניתוחיים לתמותה ניתוחית

36

Page 38: עינייםimg2.tapuz.co.il/CommunaFiles/39910353.doc · Web viewגם אם לאחר הפרוצדורה מתפתח PR, הלב הימני סובל את זה מצויין. נדיר

כירוגיית לב חזה

independent risk factors-לפי שיטות ה scoring :המקובלות .גיל נשים מתות יותר אחרי –מין CABG.)הסיבה כנראה טכנית, לב יותר פריך ועורקים יותר קטנים( LV function ,מייצג גם דרגה תפקודית( MIs.)בעבר PVD – .מעורבות קרוטיד מעל( או תחת טיפול ל-140/90יתר לחץ-דם HTN.) גורם סיכון חלש לתמותה, גורם סיכון משמעותי לתחלואה. –סוכרת או צורך בדיאליזה(.160, 130איס"ק כליות )הגשרות שונות: קראטינין מעל CVA.בעבר COPD.שמחייב טיפול ממושך בברונכודילטורים או סטרואידים ניתוח דחוף.השמנה( ניתוחי-לב קודמיםre-do.)יותר מסוכנים

כ-very poor LV function , שמנה, סכרתית, מגיעה לניתוח חרום, 80לדוגמא בת תמותה ניתוחית.30% תמותה ניתוחית. 0.5% טוב, ללא מחלות רקע LV, תפקוד SAP, 50לעומתה בן

תחלואה ניתוחיתperi-operative infarct:

o ,מנותח: קרישיות יתרMI-שהחל טרם הניתוח בגלל ה stress.o :מרדיםcoronary steal .בגלל חלק מגזי-ההרדמה o.מנתח: חוסר מיומנות, שתל לא אופטימלי

:דמםoגורמים כללים: הפרעה המטולוגית )טרומבוציטופניה(, אספירין/פלביקס/קומאדין, שימוש במכונת

לב-ריאה שמחייב הפריניזציה מלאה. o .טכני: דמם מאזור אנסטומוזה, המוסטזיס לא מספק

:נוירולוגיo -major)אירוע מוחי קשה )סימני צד, דיסארתריהominorאובדן זכרון, שינויי אישיות -

:זיהומיםomediastenitis תמותה. גורמי סיכון:50%. זיהום עמוק בפצע הניתוח בחזה אשר מביאה לעד

)בעיקר דו"צ(.internal mamaryסוכרת, השמנה, סטרואידים, לקיחת o זיהום בפצע ניתוח ברגלo.פנאומוניה

:הפרעות קצבoAF –-מהחולים אחרי ניתוח לב. סימפטומטי. 30% שכיח מאד, כ o זמניות בדר"כ, נדיר שצריך קוצב קבוע בניתוחי –הפרעות הולכה CABG.

איס"ק כליות: גורמי סיכון כולליםCre>160 .יצטרכו דיאליזה.1:6 ושימוש במכונת לב-ריאה יום מהניתוח, מכל סיבה שלא תהיה.30תמותה ניתוחית = תמותה בתוך

Off-Bypass CABGורק תוך שימוש במכונת לב-ריאה, כך שהאנסטומוזות נעשו ניתוחי מעקפים נעשו אך עד לפני מספר שנים

קמה בשאיפה לצמצם אתoff-bypass. הנטיה לניתוח blodless & arrestedבתנאים אידיאליים, כאשר הלב הוא הנזקים המוחיים והכלייתיים של מכונת לב-ריאה.

: Off-bypass CABGהאתגר של ( שהם מעין מנוף שמחובר לרטרקטור של הסטרנום,stabilizersהעורק זז )פועם( - פותחו מייצבים )(1

וממנו יוצאת זרוע דמוית "ח" שמוצמדת לאזור העורק אותו רוצים להשיק. הלחץ המקומי גורם היפוקינזיהשל אזור ההשקה )קיבוע של העורק(.

37

Page 39: עינייםimg2.tapuz.co.il/CommunaFiles/39910353.doc · Web viewגם אם לאחר הפרוצדורה מתפתח PR, הלב הימני סובל את זה מצויין. נדיר

כירוגיית לב חזה

דמם בפתיחת העורק - כדי להשיק חייבים לחסום את העורק הקורונרי פרוקסימלית ודיסטלית למקום(2ההשקה, וזה יוצר איסכמיה מקומית.

מרבית הלבבות מסתדרים עם ההיפוקינזיה והאיסכמיה המקומית. הוא bypassמבחינה טכנית, ביצוע מעקפים ללא אפשרי, אבל מותנה בקיום כישורים טכניים טובים, שהושגו

לפני שהוא מנוסה מאד בניתוח מעקפים "רגיל" והטכניקהoff-bypassבניתוח רגיל - מנתח לא יוכל לבצע ניתוח היא מאדoff-bypassהכירורגית שלו לביצוע אנסטומוזות היא מושלמת. גם אז, עקומת הלמידה לביצוע ניתוח

איטית )צריך נסיון של מאות ניתוחים עד להגעה לרמת ביצוע טובה(. וה-LADה- RCA-גלויים בעת פתיחת הפריקרד, אבל ה PDA-וה marginals.נמצאים בדופן האחורי של הלב

בניתוח "רגיל" אין בעיה להרים את הלב )ממילא היציבות ההמודינמית נשלטת ע"י המכונה( אבל כדי לבצע ניתוחoff-bypass.פותחה טכנולוגיה שתאפשר לסובב את הלב כאוות נפשנו, בלי לגרום נזק המודינמי משמעותי

: off-bypassההגבלות של ניתוח החולה והלב - לא כל החולים יכולים לעמוד בשינויים המודינמיים כי הם ממילאcompromised . אנטומיה - צריךtarget.נוח לעבודה, לא עורק שהוא טרשתי לכל אורכו

, תמיד חייבים את האפשרות לעלות מהר על מכונת לב-ריאה אם יש בעיה המודינמית. ולכןoff-bypassכל ניתוח תמיד יהיו בחדר הניתוח מכונת לב ריאה מוכנה לעבודה וטכנאי.

? off-bypassהאם יש הצדקה לניתוח ניתוחית הן בסה"כ דומות )אין עדיפות לניתוח ללא מכונת לב-ריאה(. לגבי התחלואה התוצאות לגבי תמותה

לגבי סיבוכים נוירולוגיים וכלייתיים, שחלקם מוכיחים שיפור בצורתRCTהפוסט-ניתוחית עדיין אין הכרעה: נעשו ירידה בתחלואה ואחרים לא.

בנוסף, זוהי טכניקה חדשה, ולכן אין מידע ארוך טווח לגבי הפוטנטיות של האנסטומוזות. מכיוון שאלו אנסטומוזותשנעשות "בתנאי קרב", יתכן ולאורך זמן הן יהיו פחות טובות ויסגרו מוקדם יותר.

הוא פתרון אפשרי לחולים עם טרשת מאד קשה באאורטה העולה. אצלם, עליה על מכונתoff-bypassניתוח ניתן לעשות )לכל הפחות( השקה שלoff-bypassלב-ריאה מלווה בסיכון משמעותי לסיבוכים מוחיים. בטכניקת

LIMA-ל LAD והשקת RIMA-ל RCAעם זאת, לפעמים החולים יעברו רה-וסקולריזציה לא מלאה כי לא ניתן . לעשות השקה פרוקסימלית לאאורטה או שהחולה לא סובל את הרמת הלב, ואין אופציה של שינוי לניתוח עם

. מול חסרונותcomplete re-vascularizationשימוש במכונת לב-ריאה. לכן, יש יותר חולים שמסיימים ניתוח ללא והגבלות אלו ישנו היתרון של הקטנת סיכון נוירולוגי.

נוספים לגבי התמותה הניתוחית,off-bypass CABGעל-מנת שטכניקת תתקבל באופן גורף, דרושים נתונים לאורך זמן.patency וה-incomplete re-vascularizationהתחלואה, שיעור ה-

CABGדחיפות דרגות דחיפות של ניתוח מעקפים: 3ישנן

Elective Stableחולה עם AP מגיע לניתוח מהבית, יציב לחלוטין. לרב עבר ,PTCAאבחנתי אשר הדגים מחלה שבה

CABG משפר פרוגנוזה על פני PCI .Urgent

Unstableחולה עם AP אבל יציב. החולה ינותח במהלך האשפוז. מצב נוסף הוא חולה לאחר MIאשר עבר החולה עובר את הסיבוכים הקשים כדוגמת הפרעות קצב6-72h. שכן בין 72hתקופת "צינון". תקופה זו היא לרב

מסכנות חיים. הכנסה לניתוח במצב זה מגדילה את הסיכון לסיבוכים קטלניים. לכן נבצע את הניתוח בהקדםלאחר תקופת הצינון.

RV. חלק משריר הRV. מצב זה מוגדר כאוטם שפגע בתפקוד 3-4w מחכים עד RV infractיש לציין כי לאחר מסופק ע"י כלי דם שיוצאים מחלל החדר ולכן לאחר תקופת החלמה השריר באזור המתאים יותר יציב. במקביל

.RV במועד מוקדם יותר לא מביא לשיפור תפקוד CABGביצוע Emergency

דחוף בחולה במהלך של CABGניתן לבצע MI במידה ונכנסים לניתוח תוך 6hמתחילת הארוע. לאחר מכן הניתוח מלווה בסיכונים וסיבוכים אשר מחייבים שיקולי כדאיות לגבי ביצוע הפרוצדורה.

שהסתבך, היצרות קריטית שמתגלה בצנטור.PTCA דחוף כוללים: CABGמצבים נוספים בהם יש לעבור

38

Page 40: עינייםimg2.tapuz.co.il/CommunaFiles/39910353.doc · Web viewגם אם לאחר הפרוצדורה מתפתח PR, הלב הימני סובל את זה מצויין. נדיר

כירוגיית לב חזה

היצרות קריטית - חולה שיושב על "פצצת זמן" והניתוח חייב להתבצע באופן מיידי. )לדוגמא - עורק קורונרי יחיד . עורק כזה יכול להחסם בכל שניה וחסימתו תביא מיידית למות החולה כי אין שום90%פתוח שבו יש היצרות של

לניתוח, מכניסים מניעתית טרם הניתוח, כדי לשפר את הפרפוזיהIABPעורק כלילי פוטנטי אחר(. כהכנה induction או צניחת ל"ד במהלך ה-MIהקורונרית. הבלון חשוב במהלך ההתארגנות לניתוח וחשוב גם כדי למנוע

של ההרדמה )שמללוה במקרים רבים בשינויים המודינמיים(. לגמול את החולה מה- ניתן הניתוח )ממילא הוא הוכנסIABPלאחר יותר אינו ממלא שום תפקיד כי הוא ,

( וע"י הורדת מידת הניפוח של1:1 פעימות במקום 1:4למניעה(. הגמילה נעשית ע"י הורדה הדרגתית בתמיכה ) היא מסוכנת. IABP(. גמילה מהירה מ-afterload reductionהבלון )כך שהוא יוצר פחות

Rescue CABGניתוח מעקפים בחירום, בגלל פרפורציה של עורק קורונרי במהלך צנתור. במקרה כזה דם - ניתוח ונוצרת טמפונדה, ובמקביל יש איסכמיה של המיוקרד. זה מחייב ריצה מהירה לחדר מצטבר בפריקרד

ניתןbypassוהעלאת החולה על מכונת לב-ריאה במהירות שיא, כי בלעדיה החולה ימות ! לאחר העליה על לנשום עמוק, להרגע ולהתחיל לתקן את הנזק.

שנים, כאשר טכניקות הצנתור היו10-15כיום ניתוח מעקפים חירום לתיקון נזק צנתורי הוא נדיר, אבל לפני חירום היה מתרחש פעם-פעמיים בשבוע. באותה תקופה היה מסוכן לבצע צנתוריםCABGחדשות, אירוע של

: למעשה, זוהיper-cutaneus bypassבבי"ח שאין בו ניתוחי לב. פתרון זמני שהציל את החולים )לפעמים( היה מכונת לב-ריאה קטנה שמחוברת אל העורק והוריד הפמורלי )בכניסה עיוורת, ללא דיסקציה(, ומאפשרת להשאיר

את החולה בחיים עד הבאתו לניתוח דחוף.CABGסיבוכי

1%סה"כ תמותה ממוצעת של.Mediastenitisזיהום עמוק של חלל ה .mediastenumהמקור הוא בעיקר זיהום בעצם ה .sternumמצב .

דבר המביא לראספקת דם לקוייה ביותר לעצם ולכן נטיהRIMA + LIMAזה מופיע בעיקר עקב הורדת בצורתLIMAלהזדהמות. הסיכון מוגבר עוד יותר בחולים סכרתיים. יש טענה שכאשר מוציאים את ה

skeletonizedכלומר ללא הרקמה הצמודה אליו( ולא כ( pedicleהסיכון לסיבוך זה יורד. לעומת יתרון זה היא משך ניתוח ארוך יותר וזיכון מוגבר לפגיעה בעורק בעת בידודו. לכןskeletonizedחסרון שיטת ה

נשקול שימוש בשיטה זו בעיקר בחולים צעירים )בהם יש עדיפות למעקפים עורקיים( סכרתיים )בהםהסיכון לסיבוךל גבוה(.

.pectoralis flap וכיסוי החלל שנוצר ע"י sternotomy –במידה והסיבוך מופיע יש לבצע הטריה CVA –עקב אמבוליוץ. משני לקנולה אשר מוחדרת לאבי העורקים ושיחרור רבדים טרשטיים. כאמור

.off – bypassהסיכון למצב יורד בצורה ניכרת בניתוח Dressler syndrome –חום ותפליטים ב pleura, pericardהמופיעים מס' ימים לאחר ניתוח לבבי. מטופל

.NSAID'sע"י Acute Renal Failure – ריאה נותנת זרימה קבועה בעוד שהכליה צריכה זרימה– מאחר שמכונת לב

פולסטיבית. סיבוך זה מופיע בעיקר באנשים מועדים. כלומר אנשים עם מחלות רקע שפוגעות בכליה על הצד הגבוה של הנורמה.Creכדוגמת סכרת או אנשים שהגיעו עם ערכי

Re-Do CABG ישנם מקרים שבהם אין מנוס מביצוע ניתוח מעקפים חוזר. ניתוח מעקפים חוזר הוא ניתוח יותר מסובך כי בית החזה מלא בהדבקויות. הבעיה הגדולה ביותר היא שהסטרנום דבוק ישירות אל הלב )החדר הימני(. ולכן בניסור הסטרנום יש סכנה משמעותית של פגיעה בלב או בעורקים קורונריים. לכן, פתיחת הסטרנום נעשית בצורה שונה

מהרגיל ובזהירות רבה. היא שהאנטומיה הקורונרית פחות ברורה.Re-Do CABGבעיה נוספת של

טיפול כירורגי בלב הכושל

End Stage Heart Disease (ESHD)

39

Page 41: עינייםimg2.tapuz.co.il/CommunaFiles/39910353.doc · Web viewגם אם לאחר הפרוצדורה מתפתח PR, הלב הימני סובל את זה מצויין. נדיר

כירוגיית לב חזה

ESHD– חולים עם severe CHF-לשנה, ולא ניתן לשפרם בטיפול תרופתי או50%, בעלי תוחלת חיים פחותה מ כו'.ESRDכירורגי קונבנציונלי. מחלה יותר קטלנית מרב הממאירויות,

סובלים מצורה כלשהי של קרדיומיופתיה. קרדיומיופתיה היא במקור הגדרה פתולוגית,ESHDכל החולים עם .DCM>HCM>RCMאבל היא משמשת גם כמושג קליני. סוגי קרדיומיופתיה לפי שכיחות:

Dilated CMO: 60%מהגורמים ל ESHD.פגיעה סיטולית ,האטיולוגיה ע"פ השכיחות:

ככל הנראה זיהומי שלא הוכח.–אידיופטית .12.Ischemic induced – אוטם אחד גדול או multiple infractions-משטח שריר הלב60%. כאשר נהרס כ

בלבד.LADנוצרת קרדיומיופתיה מורחבת. זה יכול לקרות ע"י סגירה של 3.Infectious –הכי שכיח זיהום ויראלי )אקו, קוקסקי(. יתכן גם בקטריאלי, פטרייתי, פרזיטי. בהגדרה, ניתן

להדגים עליה של טיטר נוגדנים. 4.Toxic –בעיקר אלכוהול, שיכול לעשות אפקט אקוטי על הלב ולעצור את ההתכווצות, ע"י הרעלה של

פחיות בירה ליום. הפיך עד שלב מסויים. יכול להיות גם נזק6 –המיטוכונדריות במיוציטים. המינון הדרוש acute,)במינוני אלכהול גבוהים במיוחד. צריך לזכור גם את האפשרות של ציטוטוקסיקה )אדריאמיצין

מתכות כבדות. .MR volume loaded valvular disease או AR –מסתמי .5 כל מום שיש בו שאנט. –מולד .6

Hypertrophic CMO: 30%פגיעה דיאסטולית , ראשוני– HOCMלמין גם תאחיזה וכנראה יש תאחיזה משפחתית הגן. זוהה לא אך גנטי לסוגיו.

)משפחות עם נשים חולות ולעומתן משפחות עם גברים חולים(.שניוני - יל"ד מוזנח ארוך-טווחASל מוביל לא hyoertrophy של low cardiac outputל גורם מסויים משלב אלא dilated

cardiomyopathyאשר היא מובילה ל ESHD.Restrictive CMO: 10%פגיעה משולבת ,

הסננה: עמילואיד, וילסון, המוכרומטוזיס–ראשוני - שניוניconstrictive pericarditisשנדבק ל myocard

Low Cardiac Output Syndrome.low CO syndrome, מכל סוג של קרדיומיופטיה, יש ESHDבכל המקרים של

יל"ד ריאתי וגודש backflow pressure and flow (forward flowהפתופזיולוגיה: כשלון שמאלי )היעדר . ESHD כשל ימניפגיעה ימנית

סימנים בבדיקה: ירידה קוגניטיבית :צווארJVP)מוגבר )עדות לאיס"ק ימנית חרחורים, צפצופים, נוזל פלאורלי )הנוזל - גודש ריאתי של איס"ק שמאלית ו/או גודש–ריאות: רטובות

לימפתי של איס"ק ימנית(.-לב: הזזה של הPMIדופק סדיר או לא סדיר, הפרעות קצב עלייתיות )הרחבת החדר והגדלת העליה ואז ,

של נוזל פריקרדיאלי. אוושה בשל הרחבת אנולוס המסתם אשרfriction rub, 3-4(, קול MSהתפתחות .MRמביאה ל

:בטןcardiac cachexia( shiftשל דם מהמעי לאיברים חיוניים, קיבה אטונית שלא סופגת כלום ומקיאים משני לכשל הימני(,TRהכל, חוסר תאבון בגלל חולשה כללית(, מיימת, כבד מוגדל )פולסטילי אם יש

cardiac cirrhosis .)מיימת אבל הכבד לא נמוש, פרוגנוזה גרועה ביותר( ,סימנים פריפריים: עור חיוור, הזעה קרהfiliform pulse ,צבע אפור ,cutis marmurata.

בדיקות עזר :אק"ג

oאריתמיות עלייתיות

40

Page 42: עינייםimg2.tapuz.co.il/CommunaFiles/39910353.doc · Web viewגם אם לאחר הפרוצדורה מתפתח PR, הלב הימני סובל את זה מצויין. נדיר

כירוגיית לב חזה

oVPBs פגיעה בארכיטקטורה של סיבי השריר גורם - VPBsלאורך זמן, השכיחות של ההפרעות . . כאשר ישCMO שהוא סיבת המוות מספר אחת ב-VT/VFהחדריות הולכת ועולה עד הופעת

systolic dysfunction( וגם הקוטר הסוף דיאסטולי Edd הוא מעל )ס"מ, חובה להגן על החדר7 מניעתיים. amiodorone או ICDע"י

מיפוי תליום - מדגים הגעת דם לאזורי הלב. בדיקהintra-vascularבדיקה פתולוגית מעידה על היעדר . זרימת דם )כלי הדם חסום לחלוטין או שאין קולטרלים( אבל לא ברור מה מצב התאים. אם יש פגם מילוי, לא נדע אם מדובר בזרימת דם באזור מת או בחוסר זרימת דם באזור בהיברינציה. לרב יש קורולציה

בין זרימת הדם לחיות התאים אך לא תמיד.PET scan –,שלב אינטרה-וסקולרי של אמוניה ושלב של קליטת גלוקוז שמעיד על קיום תאים חיים

שיכולים להנות מרה-וסקולריזציה. בדיקה אמינה יותר לאבחנה בין צלקת להיברנציה לעומת מיפוי תליום. צל"ח– cardiomegaly - cardio-thoracic index 0.5 מציין יחס לב:חזה. כאן הוא מעל. לא נותן מידע על העץ הקורונרי, לא מחשב במדויק –אקו CO .והמודינמיקה מערכת קורונרית, תפקוד לב שמאל. –צנתור שמאלי )נותן מדדים המודינמיים.–צנתור ימני )שוואן-גנץ

מדדים המודינמיים: מדידתCO שיטת :thermodilution .)או נוסחת פיק )דם ורידי מעורב

CO/body surface לשטח גוף תקין. שמוגדר ע"פ CO - יחס Cardiac Index תקין, יש COאין הגדרה של area.

CI >3lit/min/m2( .הוא תקין low CO: CI<2.) :)מדידת לחצים )ערכים תקינים

CVP 0-8mmHg )m( PA 25-30/10-20RA 0-8 mmHg )m( PWP )pulmonary wedge pressure( 8-12 )m(RV 25-30/0-10

PWP )pulmonary wedge pressure( שווה LAP )Lt Atrial Pressure(רק בהנחה שכל גל ההדף של העליה השמאלית מועבר, ללא שום הפרעה, לארטריולות הריאתיות.

חישובSysVR-ו PulmVR:)לפי חוק אוהם( SVR = MAP-CVP / CO * 80 )N=800-1200dynn(PVR = MPP-LAP / CO * 80 )N~200 dynn(

.dynn היא מקדם כדי לעבור ליחידות של 80ההכפלה בCOדרכים לשיפור של

הורדת תנגודת– afterload reduction: o.טווח מיידי: ניטרופרוסיד )ניטור קפדני, ט"נ(, ניטרוגליצריןo שעות עד ימים: מעכביACE.oIABP

הגברתcontratility – :תרופות יונוטרופיות של החולה.HR ו-SVRצריך לבחור בתרופה המתאימה לחולה, עפ"י

HRSVRעוצמה+++++++אדרנלין+++++++דופאמין

++↓+++דובוטאמין+++++++נוראדרנלין

Isoprotanol+++↓RR+++ ↓++אמרינון, מילרינון

(SVR, לא משנה HR ישנה, טובה לשימור מצב. מורידה –)דיגוקסין R.פועל בעיקר על צד ימין =

.POיש לשים לב שאין תרופות יונוטרופיות שניתן לקחת

41

Page 43: עינייםimg2.tapuz.co.il/CommunaFiles/39910353.doc · Web viewגם אם לאחר הפרוצדורה מתפתח PR, הלב הימני סובל את זה מצויין. נדיר

כירוגיית לב חזה

-ראה סקירה מעולה בUpToDate .לגבי תרופות ואזופרסוריות ויונוטרופיות השתלת לב

Inclusion criteria:ESHD :(. 70 החלטה של החברה, משתנה בלתי תלוי רק מעל גיל – שנים. )הסיבה 0-65גילStrong & Stable Psychosocial support –מובן שלא נשתיל חולה פסיכוטי, ולעומת זאת כן נשתיל

OCD הבעיה הגדולה היא .abusive disorders –חולה שלא מסוגל להכניס את עצמו למשטר של חצי עישון, אלכוהול, )סמים, מהתמכרות לגמילה morbidשנה obesityהפרעה בין ישיר קשר יש .)

מהמועמדים. 20-25% ולכן חיי השתל קצרים. לכאורה, פוסל low complianceהתמכרותית לבין Exclusion Criteria :

מחלות זיהומיות כרוניות כגוןHIV, HBV, HCV – חשש מפני flare תחת immunosupressionאיידס . מוחלטת, בהפטיטיס כן עושים השתלות כבד, לא השתלות לב.CIהוא

.)מחלה סופנית של איברים אחרים )כליות, ריאותמחלה מקצרת חיים, אבל בעיקר חשש מהתלקחות המחלה תחת הדיכוי–ממאירות בהווה או בעבר

אונקולוגי כן יושתל, אבל רמיסיה מלאה - לא.cureהחיסוני. לכן חולה שנמצא בסטטוס של :תרומת לב התנאים ל

brain death –.)מת במוחו וחי בגופו )עדיין, ימות בתוך שעות-ימים !!! הסכמה לתרומת איברים עפ"י בדיקת אקו-לב.–לב עם תפקוד תקין, ללא מחלה מסתמית עם גורמי סיכון.40 או מעל גיל 45צנתור קורונרי - מעל גיל ללא מחלה ממארת( ללא זיהום כרוניHIV.ברקע )הפטיטיס ,

התאמת תרומה-מקבל: :גודלbody surface area +/- 20%.)לב קטן לא יתמוך בגוף גדול, לב גדול לא יכנס בחזה קטן( התאמתABO ע"פ כללי קבלת דם. )התאמת HLAלא משמעותית בטווח קצר ובינוני, וממילא לא ריאלי

להתאים(. זה הזמן המקסימלי האפשרי של 8 עד –מרחק גאוגרפי( שעות הגעה cold ischemic time.)

מהלך ההשתלה: .קצירת האיבר לתרומה: חיתוך ברמת כלי הדם הגדולים .הגנה על השתל ע"י קירור ושיתוק, העברה למרכז ההשתלות .הכנת המושתל: מכונת לב-ריאה, הוצאת הלב ברמת כלי הדם הגדולים .חיבור הלב המושתל ברמת כלי הדם הגדולים!!!אימונוסופרסיה מוחלטת עוד במהלך הניתוח.חידוש זרימת הדם ללב

דחיה: Hyperacute rejection – מנגנון הומורלי ע"י pre-existing Abמתבטא על שולחן הניתוח. ניתן למנוע .

)בדיקת צימות בין סרום של המושתל מול לימפוציטים של התורם(. הרגישות שלcross-matchזאת ע"י ,PRAהבדיקה מושלמת, אבל היא אורכת זמן רב. לכן עושים קודם להשתלה בדיקה של עירור אימונוגני )

panel reactive antigensשבה חושפים את הסרום של המושתל למדגם של דם באוכלוסיה ורואים אם ) לפני ההשתלה. החולים שיהיו מעוררים חיסונית הם מיcross matchמתעוררת תגובה. אם כן, חייבים

שקיבלו דם רב, אשה שילדה פעמים רבות, אדם בעל ריבוי פרטנרים מיניים.Acute rejection –מנגנון תאי. מתחילה ימים אחדים לאחר הניתוח, ולעולם לא מסתיימת. חייבים טיפול

אימונוסופרסיבי. הפרוטוקול הקלאסי: סטרואידים + אימוראן + ציקלוספורין. יש פרוטוקולים אחרים. לכל עומס נוזלים, האצה–התרופות הללו יש המון תופעות לוואי. סטרואידים בעייתיים במיוחד במושתלי הלב

של אתרוסקלרוזיס, הפרעות באיזון סוכר עד סוכרת, פגיעה בפרופיל שומנים. בדר"כ החולה חייב תרופות

42

Page 44: עינייםimg2.tapuz.co.il/CommunaFiles/39910353.doc · Web viewגם אם לאחר הפרוצדורה מתפתח PR, הלב הימני סובל את זה מצויין. נדיר

כירוגיית לב חזה

היענות: של קשה בעיה מתעוררת לוואי. בתופעות לטיפול תרופות המוני ועוד אימונוסופרסיביות .transplant disease ב-ESHDמחליפים

Chronic rejection –.מתחיל להתפתח חודשים עד שנים לאחר ההשתלה. מנגנון משולב הומורלי ותאי . HLAככל הנראה קשור בחוסר התאמה של

פרוגנוזה: 1m – 95% עקב תמותה השרדות: hyperacute rejectionבעיות השתל, של טוב לא שימור ,

טכניות-כירורגיות. 1y – 90% דחיה בחצי שנה הראשונה, ואח"כ יחס הפוך. 1/3 זיהומים ו-2/3 השרדות: תמותה עקב 3y – 75% .דחיה .5y – 68% . שבו זה קורה תוך שנה(. ESHD תמותה )לעומת 50% שנים תתקבל 11רק ב-

מעל בגלל 3התמותה בעיקר אבל ממאירויות, עקב היא שנים small vessel disease המכונה graft vasculopathy קומפלקס שקיעת עקב נגרמת המחלה .Ab-Ag התאמת חוסר בגלל כנראה ,HLAגם יש .

משמעות לטיפול בסטרואידים. התופעה הזו קיימת בכל סוגי ההשתלות, וכיום לא יודעים לטפל בחסימת כלי הדם שנים אחרי ההשתלה. 22הקטנים, והחולים מתים. המושתל ששרד הכי הרבה חי

איכות החיים של המושתלים טובה למדי. בהשתלת לב לא מחברים את הואגוס. לכן, לחולה אין יכולת לעשות טאכיקרדיה רפלקסיבית והוא מאד מוגבל

ע"י טאכיקרדיה. הפתרון הוא מתן משאף של ונטולין לפני תחילתCOבהתחלת מאמץ כי אינו יכול להעלות המאמץ. זה עושה טאכיקרדיה לזמן קצר, עד שהאדרנלין האנדוגני יכנס לפעילות.

מה צופן העתיד ? Cardiomyoplasty –עטיפה חיצונית של שריר הלב ע"י שריר משורטט אחר, קיצובו והפעלתו כמשאבה חיצונית

שתומכת בלב. נהוג להשתמש בשריר הלטיסמוס דורסי )ניתן להעבירו לכיוון הלב בלי ניתוק אספקת הדם שלו(. . התברר שניתןtetanyהבעיה בשימוש בשריר סקלטלי היא שהוא, בשונה משריר הלב, מגיב למאמץ ממושך ע"י

,1:1, ובהדרגה עולים לקיצוב של 1:4 בתחילה מקצבים אותו ביחס של –להתגבר על הבעיה ע"י אימון של השריר כך שהשריר תומך בכל פעימות הלב. למרות זאת, השריר עובר פיברוזיס אחרי שנתיים.

הטכניקה אינה בשימוש כגשר להשתלה, כי הוצאת המבנה הזה בעת ההשתלה גורמת לדמם נוראי. זהו פתרון שמסיבות שונות לא ניתן להשתיל. סה"כ כיום כמעט לא בשימוש.ESHDלהארכת חיים בשנתיים במקרים של

Heart assistance device –משאבה קטנטנה שתומכת בפעילות של הלב. המשאבה יכולה לתמוך בצד אחד של הלב, או בשניהם )שזה בעצם לב מלאכותי(. בשנים האחרונות חלו שכלולים רבים במזעור של המשאבה, מציאתשל הבעיה נטרול לזיהומים(, מקור שמהוים חוטים צריך שלא )כך הסוללות של חיצונית לטעינה פתרון אנטי-קואגולציה ע"י אפיתליזציה של המשאבה ועוד. המשאבה הזו הומצאה במקור כגשר להשתלה, משמשת

כיום כגשר לריפוי של מחלות כמו מיוקרדיטיס ובעתיד צפוי שזה יהיה הפתרון הדפנטיבי )במקום השתלה(. . 17 – משאבות כאלו, ובישראל 5000עד היום הושתלו בעולם

מכונת לב-ריאה, ניטור תוך ניתוחי

)מל"ר(. משתמשים במל"דcardiopulmonary bypass –ניתוח לב פתוח = ניתוח תוך שימוש במכונת לב-ריאה בניתוחים הבאים

מעקפיםתיקון מסתמיםניתוחי עורקים גדוליםמומים מולדים

ישנם ניתוחי לב שנעשים ללא שימוש במכונת לב-ריאה : במקרים מיוחדים–מעקפים תיקוןMS )comisurotomy(

43

Page 45: עינייםimg2.tapuz.co.il/CommunaFiles/39910353.doc · Web viewגם אם לאחר הפרוצדורה מתפתח PR, הלב הימני סובל את זה מצויין. נדיר

כירוגיית לב חזה

סגירתPDA. bloodless ו-arrestedהשימוש במל"ר מאפשר להשיג שדה כירורגי עם לב שהוא

, בתיקון מומים מולדים. השימוש במל"ר לתיקון בעיות נרכשות50השימוש הקליני במל"ר החל בראשית שנות ה-.60)מעקפים, מחלות מסתמים( החל בשנות ה-

"לקיחת" הדם (. לצורך כך עושים תפר טבעתיbloodless, כי אז הלב לא PA )לא RAבמרבית המקרים, הוצאת הדם היא מה-

. בקנולה יש חורים בקצה וחורי צד. הקצה יושבRA הכנסת קנולה סטרילית לחלל ה-– חתך –באוזנית של העליה . אל הקנולה מתחבר צינור )לא סטרילי, מחוץ לשדה הניתוחי( שמעביר את הדם אלRA ושאר הקנולה ב-IVCב-

למכונה הוא פאסיבי תודות לגרביטציה )החולה יותר גבוה מהמכונה(. לאחר הכנסתRAהמכונה. מעבר הדם מה- עד להרכבת המערכת השלמה.clampהקנולה היא חסומה ע"י

שכן בעליה יש לחץ שלילי שמושך את המסתם. בנוסף התנגודת דרךtricyspidדם לא עובר דרך המסתם ה המכונה נמוכה הרבה יותר מהתנגודת דרך הלב והריאה.

–, מיקסומה(, קנולציה ימנית אינה טובה TV, תיקון ASD בניתוח )למשל סגירת RAכאשר צריך לפתוח את ה-ברגע שנפתח את העליה, הדם יברח לשדה הניתוח, והחולה ימות.

. סביב הקנולה שמים קשר שמכוון את כל הדם לקנולה,SVC וה-IVCבמקרה כזה עושים קנולציה כפולה, של ה- לא משמעותי כי פתיחת coronary sinusוכך כך הההחזר הורידי יכנס אל שתי הקנולות )ההחזר הורידי של ה-

(.bloodless & arrestedהעליה היא על לב שהוא )Cardio-Pulmonary Bypassכונת לב-ריאה (מ

מחמצן reservoir דם מהחולה משאבההחזרת הדם אל החולה. המכונה יכולה להחזיר את הדם לחולה בצורה פולסטילית או רציפה-למינרית. הזרימה הלמינרית היא לא פזיולוגית, אך למשך הזמן שבו מחובר החולה למכונה זה לא משפיע על הפרפוזיה לאיברים )זרימה למינרית תעשה בעייתית בחולה שמחובר למל"ר

למשך ימים(. הרצוי )נוסחתflow היה נמוך(. ה-COקצב החזרת הדם הוא בתפוקת הלב האופטימלית )גם אם לפני הניתוח ה-

. lit/min/m2 2-2.5אצבע( הוא או מגדיליםflow מורידים –. אם זה לא מתקיים lit/min 5, צריך לקבל מהחולה lit/min 5 כדי להחזיר –המגבלה

ע"י דם, תמיסה קריסטלואידית. reservoirאת הנפח ב- - החזרת הדם3שלב

קנולהחתך ascendig aorta . עושים תפר טבעתי ב-ascending aortaבמרבית המקרים נחזיר את הדם ל- צינור )לא סטרילי( אל המכונה. סטרילית

על הקנולה, עד להפעלת המערכת.clampגם כאן, שמים AV( וגם אנטגרדית לפריפריה. כאשר יש AVכשהמל"ר מופעל, הדם המחומצן זורם מהקנולה רטרוגרדית )לכיוון

clamp צריך להוסיף ARקומפטנטי - הדם הרטרוגרדי לא יכנס אל החדר, אבל כן יספק את הקורונריים. אם יש על האאורטה מיד לאחר הפעלת המכונה )יהיה דלף קצרצר(.

יש מצבים בהם לא ניתן להחזיר את הדם אל האאורטה העולה: ,דיסקציה של האאורטהtype A .)קרע באאורטה העולה או בקשת( .אנוריזמה של האאורטה העולה טכנית אין בעיה, אבל בהכנסת הקנולה אנחנו נשחרר–טרשת קשה )הסתיידויות( באאורטה העולה

רבדים טרשתיים שיכולים לגרום לאוטמים מוחיים או פריפריים. לכן, אם ההסתיידויות מאד קשות, ולאניתן למצוא מקום לקנולה, צריך לחשוב על פתרון אחר.

)מחלה טרשתית מחלה של אחרים אתרים או הקרוטיד על אוושה כשיש מתקיים הקליני החשד ללא חומר ניגוד. ניתן גם לעשות אקוCT(. האבחנה היא ע"י צל"ח, אקו, צנתור, CVA, PVDקורונרית,

תוך ניתוחי פרי-אאורטי, אבל זה בדר"כ נעדיף לדעת על הבעיה לפני הניתוח. נצטרך לשקול את ביטולהניתוח )אם אפשר( או שינוי הטכניקה הכירורגית.

פתרונות אפשריים :

44

Page 46: עינייםimg2.tapuz.co.il/CommunaFiles/39910353.doc · Web viewגם אם לאחר הפרוצדורה מתפתח PR, הלב הימני סובל את זה מצויין. נדיר

כירוגיית לב חזה

-קנולציה של הInnominate-או ה Subclavianאם אינם מעורבים בדיסקציה / אנוריזמה(. ניתן להכניס( היא לפיsubclavian ל-innominateלתוכם קנולה ואז לנתח את האאורטה הפרוקסימלית. הבחירה בין

מידת המעורבות הדיסטלית של הדיסקציה / אנוריזמה. עושים חתך נוסף במפשעה, מבודדים את העורק הפמורלי ואם הוא נמוש תקין )ואינו–עורק פמורלי

מעורב בדיסקציה( ניתן להכניס לתוכו קנולה. החזר הדם דרך הקנולה ילך רטרוגרדית לכיוון הלב ולעורק הפמורלי הנגדי ובמקביל אנטגרדית אל הפריפריה )גם כאן הפתרון הוא לא פזיולוגי, אבל עובד למשך זמן

הניתוח(. graft בניתוח דיסקציה או אנוריזמה יש שלב שבו פותחים את האאורטה כדי להחליף אותה ב-–הבעיה

18סינתתי. בשלב הזה חייבים לעקוף את האאורטה לחלוטין )היא לא קיימת( - עושים קירור עמוק ) 30מעלות( ובסיוע הגנות נוספות )פרמקולוגיות ואחרות( ניתן לעצור לחלוטין את פעולת המכונה למשך

דקות, ובזמן הזה ליצור אאורטה סינתתית. Priming של המכונה

ליטרים, ובתחילה היא מלאה באויר )שיגרום 3-2נפח הצנרת של המל"ר הוא air emboliאם יוכנס למחזור שהוא נוזל קריסטלואידי, והוא מחליף אתprimeהדם(. לכן, לפני ההפעלה של המערכת, ממלאים את הצנרת ב-

כל האויר שבמערכת. 8-9( יש מהילה משמעותית של הדם וצניחה בהמוגלובין, שיורד לרמה של clampsבנקודת ההפעלה )פתיחת ה-

gr%זה דווקא רצוי מכיוון שמיהול הדם מפחית את הנטיה לקרישיות בזמן שהחולה על מל"ר, והפגיעה בהובלת . החמצן היא לא מאד קריטית לזמן הניתוח.

של המכונה הוא ע"י דם שתואםprimeמיהול הדם בנפח גדול של נוזל הוא מאד בעייתי בתינוקות, ולכן אצלם ה-את דם התינוק. ישנן נוסחאות לחישוב כמות הדם ומיהולו שמתאימים לתינוק, עפ"י משקלו.

ע"י הפארין במנות עצומות, שהואfull anticoagulationאנטי-קואגולציה: כדי להעלות חולה על מל"ר חייבים חיוני כדי למנוע התפתחות של קרישים במערכת של צינורות סינתטיים.

ניטור בעת הניתוחלחץ-דם כולל ל.ד עורקי אינבסיבי)אק"ג )מוניטור)סטורציה )לא קצב נשימות, כי החולה מונשםטמפרטורהתפוקת שתןCVP

ניטור לחץ דם ( אינו רציף ואינו מספק בניתוח שבו יש שינויים המודינמיים גדולים, כמו ניתוחי לב.cuffניטור לא חודרני )ע"י

. cuffבנוסף, על מל"ר יש זרימה למינרית שגם היא אינה מאפשרת מדידת ל"ד ע"י . הליין יוכנס לכל עורק נגיש, שיש לו קולטרלים. בדר"כ זה נעשהarterial lineבניתוח לב יש צורך בניטור חודרני -

שהכנסתend artey לטראלית(. לא נכנסים לעורק הברכיאלי כי זהו VANבעורק הרדיאלי או בעורק הפמורלי )ליין לתוכו לגרום איסכמיה ליד הדיסטלית.

עקומת הלחץ-דם שמתקבלת היא פולסטילית, בקצב הדופק של החולה. ערך המינימום בעורק פריפרי הוא לא.dicrotic notchאפס. יש בדר"כ

CVP )Central Venous Pressure) מייצג גם אתCVP(. בלב בריא, ה-cmH2O 2-10 מודד את לחצי המילוי של לב ימין )הלחץ הנורמלי הוא CVPה-

נמוך מעיד על היפוולמיה. CVP(. LVEDPלחצי המילוי של לב שמאל )אין שום ערך בעת השימוש במל"ר, אבל הוא מאד חשוב לפני ובעיקר אחרי הירידה מהמכונה, כדיCVPל-

לעשות אופטימליזציה של תפוקת הלב. , ומשם מקדמים אותו אל.subclavian .v או ל .internal jugular .v או ל-.subclavian .v מוכנס ל-CVPצנתר ה-

. הצנתר משמש למדידת לחצי מילוי )כנ"ל( וגם למתן נוזלים, דם, תרופות ועוד.RA או ה-SVCה-הפעלת מכונת לב-ריאה

45

Page 47: עינייםimg2.tapuz.co.il/CommunaFiles/39910353.doc · Web viewגם אם לאחר הפרוצדורה מתפתח PR, הלב הימני סובל את זה מצויין. נדיר

כירוגיית לב חזה

, כך שהדם יזרום דרך המכונה.clampsלאחר חיבור כל מערכת הצינורות, מפעילים את המכונה ומסירים את ה- לב שעובד על ריק. – empty beating heartבשלב זה יש לנו

מה קורה למדדי הניטור ?CVP –-אין דם שנכנס ל( מתאפס RA.).)אק"ג - אינו משתנה כלל )מייצג את הפעילות החשמלית ממ"כ. ניתן להשפיע50-100 נוצר קו לחץ ישר ורציף. שואפים ללחצי פרפוזיה ממוצעים של –רישום ל"ד

או התנגודת )ואזודילטציה או ואזוקונסטריקציה(flowעליו ע"י שינוי ה-הפסקת פעילות הלב

על האאורטה שממוקם ביןclamp צריך להפסיק זרימת דם לעורקים הכליליים, ע"י arrestedכדי להשיג לב שהוא של שריר הלב, בגלל שאיןglobal ischemiaהקנולה לבין מוצא הקורונריים. עצירת הזרימה הקורונרית גורמת

של השריר. demandאספקת דם ועדיין יש :myocardial protection –חייבים לנקוט במספר אמצעים כדי להקטין את את הנזק האיסכמי

את תצרוכת החמצן ע"י השריר. השיתוק נעשה ע"י תמיסת "קרדיופלגיה"90%מפחית ב-–שיתוק הלב שמוזלפת אנטגרדית או רטרוגרדית.

ע"י הזלפה של תמיסה קרה על פני הלב או ע"י מתן תמיסת קרדיופלגיה מקוררת. משיגים–קירור הלב מתוך5% )כלומר מוריד 50% מעלות. פעולה זו מקטינה את תצרוכת החמצן בעוד 4-3קירור של הלב עד

תצרוכת החמצן שקיימת לאחר השיתוק(.10%Cardioplegia - קרדיופלגיה

לעורקים הכלייליים )clampמזליפים את תמיסת הקרדיופלגיה לשורש האאורטה, במקטע שבין ה- antegrade cardioplegia כך שרק הלב מושפע ממנה. התמיסה ניתנת לזמן קצר ותוך דקה הלב משתתק, ומתקבל ,)diastolic

arrest -2-3 דקות, ולא באופן רציף כל פעם ל15-10 עם לב רפוי. בהמשך, התמיסה ניתנת אחת לmin. +5%(. תמיסת הקרדיופלגיה הראשונה היתה גלוקוז mEq/lit 40-50תמיסת הקרדיופלגיה מאד עשירה באשלגן )

KClבהמשך פותחו תמיסות יותר עשירות שגם מגנות על הלב באופן אקטיבי )ולא רק ע"י הפסקת הפעילות . היא כזו תמיסה bloodהמכאנית(. cardiopegia-מ שמורכבת ו-75% החולה של מחומצן דם תמיסה25%

קריסטלואידית עשירה באשלגן. כך התמיסה גם מאוששת את הלב אחת לכמה דקות ומאפשרת לו לפנות את אל המכונה )ומוחזרת אל החולה(, ולכאורה עלולה להיווצרcoronary sinusהפסולת. התמיסה מתנקזת דרך ה-

היפרקלמיה לקראת סוף הניתוח. בפועל, בחולה שנותן שתן, משך הניתוח וכמות התמיסה שניתנת לרב לא יוצריםבעיה במאזן האשלגן.

retrograde cardioplegiaבחלק מהחולים יש מחלת לב כלילית משמעותית ולכן מתן קרדיופלגיה דרך העורקים - coronary sinusהכליליים מונעת ממנה להגיע כראוי לכל השריר. הוצע לתת קרדיופלגיה רטרוגרדית דרך ה-

)מחייב לעשות חתך וקנולציה נוספת בדופן העליה הימנית(. הסתבר שרב הלב עובר פרפוזיה ע"י קרדיופלגיה כזו (. בסה"כcoronary sinus)יש בעיה היא רק באזורים קטנים של חדר ימין שמתנקזים ישירות אל החדר לא דרך ה-

השיטה הזו, למרות שאינה פזיולוגית, עובדת לא רע. כיום זהו "מוצר מדף" שמאפשר לתכנן את הניתוח להשגתולכן הזרימה גבוהים בה בלחצים ניתן להשתמש זו היא שלא נוסף של שיטה תוצאות אופטימליות. חסרון

האפשרית כאן נמוכה יותר. הביחרה בשיטת ההגנה על הלב תלויה במצב הבסיסי של הלב והיקף הניתוח. המנתח יכול לשלב בין קרדיופלגיה

,blood cardioplegia או להשתמש ב-KClאנטגרדית או להוסיף קרדיופלגיה רטרוגרדית, לבחור בין תמיסת גלוקוז-לקבוע את תדירות ההזלפה, ולהחליט האם לקרר את הלב.

שעות. משך זמן שכזה חשוב4 עד cross clamp )ischemic( timeשילוב אופטימלי של ההגנות מאפשר להגיע ל-בהשתלת לב . בניתוחי לב סטנדרטיים זמן האיסכמיה לא עולה על שעתיים.

Reperfusion + שפעול הלב צריךclamp על האאורטה ומחדשים את הפרפוזיה ללב. )לפני הסרת ה-clampלאחר סיום הניתוח מסירים את ה-

, כדי למנוע תסחיפי אויר לקורונריים(. כעבור זמן מה )מס' דקות עד חצי שעה( מתחדשתde-airingלעשות פעימת הלב. משך תקופת הרפרפוזיה תלוי בזמן האיסכמיה. ניתן לראות את התכווצות הלב בעיניים, וגם עוקבים

,VT/VFאחרי האק"ג לראות את צורת הקומפלקס. במקרים רבים יש בשלב הזה הפרעות קצב קשות, כולל .empty beating heartשמחייבות מכת חשמל תוך-ניתוחית. לבסוף מושג מצב של

46

Page 48: עינייםimg2.tapuz.co.il/CommunaFiles/39910353.doc · Web viewגם אם לאחר הפרוצדורה מתפתח PR, הלב הימני סובל את זה מצויין. נדיר

כירוגיית לב חזה

ירידה ממכונת לב-ריאהניטור לפני הניתוק מ-מל"ר:

בעין, האם כל הלב מתכווץ ? סינכרונית ? –התכווצות הלב קצב סינוס ? הפרעת הולכה ? עליות –תרשים האק"ג ST ? המערכת יכולה לחמם את החולה ההיפותרמי.–טמפרטורת גוף החולה דופלר על כלי הדם שחובר אח\ליהם מעקףהפרעות אלקטרוליטריות, אסטרופ!!! המוסטזיס הכנסתLAP

LAP)Lt Atrial Pressure) – צנטר שמודד LVEDPבאופן ישיר ומסייע מאד לניטור פוסט-ניתוחי. זהו צנתר דמוי שמכניסים בזמן הניתוח לתוך העליה השמאלית, מוציאים דרך העור ומחברים ל-CVPצנתר tranducerהוא .

נשאר בלב בסוף הניתוח ומוצא רק כעבור מספר ימים. ה- בריא, ל-LAPבלב מאד דומה CVPובתקופה המדדים, שני בין פער צפוי חולה הלב כאשר אבל ,

כדי לעשות ניטור אמין של תפקוד החדר השמאלי רוצים ביממה הראשונה( אנחנו )בעיקר הפוסט-ניתוחית יותרLVEDPאופטימיזציה של תפוקת הלב עפ"י עקומת פרנק-סטרלינג )חולה עם תפקוד לבבי פחות טוב יצטרך

לא ישתנה במקרה של אירוע חריף בחדר שמאל )למשל איסכמיה( ואילוCVP(. בנוסף, COגבוה כדי להשיג אותו יעלה מיד.LAPה-

וגם מאפשר למדוד )LAP )wedge pressureצנתר שוואן-גנץ יכול לתת את אותם נתונים כמו ה- CVPאבל , השוואן-גנץ הוא צנתר יקר עם הרבה סיבוכים, ובנוסף המדידה ע"י שוואן-גנץ היא עקיפה ומוגבלת ע"י לב שמאלי

שאינו תקין. ניתוק ממל"ר:

חסימת קנולה ורידית כך שכל הדם הסיסטמי יכנס ישירות ללב. .1החזרת הדם מהרזרבואר לחולה באמצעות הקנולה העורקית..2נותנים פרוטמין סולפאט כדי לסתור את ההפארין..3עושים דה-קנולציה עורקית .4המוסטאזיס .5סגירת החזה..6

ניטור המודינמי לאחר ניתוח - מהלך תקין

ראשונית ההערכ אתה מגיע לחולה שעבר אתמול ניתוח לב גדול תוך שימוש במכונת לב-ריאה. מה אתה רוצה לבדוק ? מה אתה

רוצה לעשות ? מה התוכנית ליממה הבאה ? הערכה נוירולוגית

? האם החולה בהכרה ? האם מגיב לפקודה ורבלית ? מבקשים מהחולה להניע כל גפה בנפרד, ניתן לתת גירוי תחושתי–האם יש חסרים מוטוריים גסים

שיעורר את החולה להניע את הגפיים. אין טעם לבדוק תחושה, תפקודים גבוהים.הערכה נשימתית

כחלון ?–צבע העור ? )טאכיפנאה ? דיספנאה )סובייקטיבית? שימוש בשרירי עזר עם חמצן, בלי חמצן. אין יעד ברור מבחינת החמצון של חולה ביום שלאחר הניתוח. עם זאת,–סטורציה

)שהיא O2Sat>90%שואפים להשיג לפחות PO2>60mmHgבפרט בניתוחי לב. רב החולים מסוגלים ,)

47

Page 49: עינייםimg2.tapuz.co.il/CommunaFiles/39910353.doc · Web viewגם אם לאחר הפרוצדורה מתפתח PR, הלב הימני סובל את זה מצויין. נדיר

כירוגיית לב חזה

לשמור סטורציות כאלו, אבל ביום שלאחר הניתוח שכיח שיהיו שאנטים )אזורים שמקבלים פרפוזיה אך.95%לא אוורור( בשל גודש או תמטים ולכן תמיד נותנים לחולה בחמצן כדי לשמור על סטורציה באזור

לנסות–האזנה ניתן בפסגות(. אנטריורית, להאזין )ניתן היעדרה או אויר כניסת מחפשים בעיקר להתרשם מגודש, אבל לא יותר מכך.

מותר, כדאי וצריך להושיב את החולה בזהירות ולגרום לו לנשום עמוק ולהשתעל.–פזיותרפיה נשימתית ניתן גם להאזין לשדות הריאה האחוריים בחיפוש קולות זו פותחים אלבאולי. באותה הזדמנות בדרך

נשימה.הערכה קרדיו-וסקולרית

תקין ?COהמטרה: האם הפרפוזיה של החולה תקינה ? האם ה-פרפוזיה פריפרית

הכלי הכי חשוב! להערכת פרפוזיה פריפרית. אם הפריפריה חמה, אפילו אם לא–טמפרטורת הגפיים מיששנו דופק, יש פרפוזיה טובה.

חוסר חמצון( ? חוורון )–צבע( כחלון CO? )נמוך גורם ואזוקונסטריקציה מילוי קפילרי דפקים. נוחכות דופק קרוטידי מלמד עלsystolic BP>60 דופק רדיאלי ,– systolic BP >80ניתן להעזר .

במכשיר דופלר לזיהוי דופק פריפרי שאינו נמוש. מיקום דפקים:oהDorsalis pedis 1 נמצא במרווחst-2nd metatarsal .o -הTibialis posterior-נמצא מאחורי ה medial maleolus.o-לצורך מישוש הדופק פופליטאלי צריל לעשות פלקציה של הברך, פיסוק ראשי הhamstrings

לצדדים ומישוש. הדופק נמצא בקו האמצע. מחייב הפעלת לחץ בשתי הידיים. אוושה על הקרוטידים: המקום הנכון להאזנה הוא מתחת לזוית המנדיבולה, כי שם נמצאת הביפורקציה

)שם נמצאות ההיצרויות שיגרמו לזרימה הטורבולנטית שנשמע(. למרות זאת, אוושה משמעותית תשמע שתקרין לשני צידי הצוואר, אבל לאASגם במקומות אחרים. האוושה בדר"כ חד-צדדית )בניגוד לאוושת

תמיד קל להבדיל(. אין קורלציה בין עוצמת האוושה לעוצמת ההיצרות. פרפוזיה כלייתית

.0.5-1cc/kg/hrהמדד לפרפוזיה של הכליות הוא תפוקת השתן. תפוקת שתן נורמלית היא ליטר(6לאחר ניתוח גדול שאינו ניתוח לב, תפוקת השתן לא שונה מהרגיל. בניתוח לב יש העמסת נפח ניכרת )כ-

במהלך הניתוח. החולה הרבה פעמים יצא מהניתוחIV של מכונת לב ריאה ובגלל נוזלים שניתנו primingבגלל ה- בצקתי, כי השימוש במכונת לב-ריאה גם מעורר תגובה דלקתית )משפשוף הדם בצנרת סינטטית( ושחרור פקטורי

לחללים חוץ-וסקולריים. leakדלקת גורם לכן, בשעות הראשונות לאחר ניתוח לב החולה ישתין יותר מהרגיל )בתנאי שהלב והכליות תקינים(. אם בשעות

נמוך ומחייב לחפש סימנים נוספים לתפוקת לבCO זה מעורר חשד ל-1cc/kg/hrהראשונות תפוקת הלב היא רק לא מספקת, ומצריך מעקב קפדני.

ימים. מנטרים את התהליך ע"י שקילה יומית. ניתן גם3-2החולה בדר"כ ישתין את עודף הנוזלים בהדרגה, תוך לעזור לחולים ע"י מתן משתנים.

מדדים המודינמייםלחץ-דם

ל"ד אינו מדד טוב לפרפוזיה כי יכולה להיות תפוקת לב מאד נמוכה ועדיין לחץ הדם יהיה תקין תודות למנגנוני כ"כ ירוד עד שמנגנוני הפיצוי כשלו, וזה מצב חירום. COהפיצוי. עם זאת, ל"ד נמוך מעיד על

ביממה שלאחר הניתוח אנחנו רוצים לחצי-דם שהם בגבול התקין. ל"ד גבוה יכול להיות מסוכן, בגלל שיש תפרים על כלי-דם )כולל האאורטה( ואנחנו לא רוצים שהם יקרעו. לכן, נוריד ל"ד גבוה מדי. עם זאת, חשוב להשוות את

שלו, כי אם החולה בדר"כ בלחצי דם גבוהים, הרי שלחץ-דםbaselineלחץ הדם של החולה לאחר הניתוח ל- "תקין" הוא נמוך בשביל אותו חולה. במקרה כזה נוריד את לחץ הדם בשעות הראשונות ע"י תרופות, אבל אח"כ

נסכים לקבל ל"ד מעט גבוה כדי להמנע מהיפופרפוזיה יחסית של האברים )בעיקר המח, הכליות והלב עצמו(. דופק

(. rhythm(, ועל מקור הקצב )HRמסתכלים על קצב הדופק )

48

Page 50: עינייםimg2.tapuz.co.il/CommunaFiles/39910353.doc · Web viewגם אם לאחר הפרוצדורה מתפתח PR, הלב הימני סובל את זה מצויין. נדיר

כירוגיית לב חזה

אנו לא רוצים דופק מהיר כדי שלא להעמיס על הלב. טאכיקרדיה יכולה להעיד על סטרס או קצב הלב : נמוכה, שהסיבה לה היא בדר"כ היפוולמיה )העמסת גם להיות פיצוי על תפוקת לב יכול כאב, אבל כדי )להלן( ניתן להעזר בלחצי המילוי חוץ-וסקולרית(. במקרה הזה ברובה הניתוח היא ע"י הנוזלים

להחליט על תוספת נפח. HR( נמוך לדקה( אינו מטריד אם הפרפוזיה הפריפרית תקינה. רק במידה שהפרפוזיה נפגעת כדאי60

להעלות את הדופק. צריך לזכור גם שחולים רבים מקבלים חוסמי בטא. מקור הקצב: חולה שנכנס לניתוח בסינוס גם ייצא בסינוס, אבל חולה עםchronic AFלא ייצא פתאום

( וזה בסדר. למרות זאת, חולים שאצלם היענותMAZEבקצב סינוס לאחר הניתוח )בהנחה שלא עשינו כדי להגדיל אתatrial kick, יל"ד ריאתי, אוטמים מרובים( זקוקים נואשות ל-ASחדר שמאל מאד נמוכה )

נפח הפעימה, ובהם נתעקש על סינוס.לחצי מילוי (ניטור חודרני)

CVP )Central venous Pressure( = RVEDP-בלב תקין, ה :CVP-נמוך מה יותר מעט LAPבגלל , שהיענות חדר ימין קצת יותר טובה. בדר"כ ההבדל הזה חסר משמעות.

LAP )Lt Atrial Pressure( = LVEDP ערך :LAPנמוך עם פרפוזיה פריפרית תקינה זו לא בהכרח בעיה ( הרבה יותר מטריד - הוא מעיד על פגיעה בהיענותcmH2O 15> גבוה )LAPולא חייבים להחזיר נפח.

החדר, למשל ע"י איסכמיה.צנתר שוואן-גנץ )אם הוכנס( מאפשר:

o מדידה של תפוקת הלב ע"יthermodilution.o מדידתLVEDP ע"י( wedge pressure בהיעדר )LAP.o-מדידת רוויון חמצן בעורק הריאה: הדם בPA הוא mixed venous bloodוהסטורציה התקינה

. כאשר תפוקת הלב נמוכה הדם נשאר בפריפריה זמן ממושך יותר, ולכן רוויון70-75%שלו היא לאחר אוטם יביא לתפוקת לב אפסית )שוקacute VSDהחממצן יורד. יוצא-דופן לכלל הזה:

קרדיוגני( אבל רוויון חמצן תקין בגלל הדלף. . כאמור, בלב בריא ההבדל בין שני המדדים הוא קטן וחסרCVP למרות קיום LAPנשאלת השאלה מתי נכניס

. אם יש חשדLAP וכן יתבטא בעליית CVPמשמעות. עם זאת, כאשר יש פגיעה בחדר שמאל זה לא יתבטא ב- לפגיעה בתפקוד חדר שמאל )בגלל מצב הלב טרם הניתוח או בגלל מהלך ניתוחי לא מושלם( רצוי להכניס גם

LAP כדי שנוכל לזהות מיידית בעיה בתפקוד החדר. אמנם, אם החולה מציג בעיה חריפה ולא הושאר LAPבעת הואLAP, אבל הסיכון הטמון בהכנסת wedge pressureהניתוח, ניתן במהירות להכניס קטטר שוואן-גנץ ולמדוד

נמוך והמידע שהוא מספק יכול להיות קריטי במצבים חריפים. טרם הניתוח היה גבוה, ולאחר הניתוח אנחנו רוציםLVEDP היא בחולים שאצלם ה-LAPסיבה נוספת להכנסת

, וצריכים ניטור הדוק. COלשמור עליו גבוה כדי לאפשר אופטימליזציה של ה-בדיקת גזים בדם

חולה עם תפוקת לב לא מספקת יכנס לחמצת מטבולית. חמצת מטבולית היא חמצת לקטית עד שיוכח אחרת, כי( היא הרבה יותר נדירה, בפרט בחולה לאחר ניתוח גדול. DKAכל אטיולוגיה אחרת )למשל

הבוקר שלאחר הניתוח מעקב חום

ניתוח חזה הוא ניתוח "נקי" שבו אל אמור להיות זיהום )בניגוד לניתוחי בטן עם פתיחת המעי(. למרות זאת, ניתוחילב הם ניתוחים ארוכים וזה מעלה את הסיכון לזיהום ניתוחי.

שעות ראשונות לאחר הניתוח הוא שכיח מאד, ובדר"כ חסר משמעות. המקור הוא בדר"כ24-48חום במהלך אטלקטזיס ריאתי או תגובה דלקתית לשימוש במכונת לב-ריאה. זיהומים שנרכשו בניתוח )זיהום פצע ניתוח( יופעיו רק בטווח זמן של ימים ולא ביממה הראשונה. לכן, כאשר לחולה יש חום צריך לבדוק אותו וניתן לבקש ס"ד

ש- ומתןWBC)למרות דם תרביות לקיחת עם להתאפק כדאי בדר"כ סטרס(. בגלל להיות יכולים גבוהים אנטיביוטיקה.

, או בחולה שהיו לו אבססים טרםSBEהגישה שונה לחלוטין כאשר מדובר בחולה שנותח להחלפת מסתם בשל הניתוח.

הפרשה מנקזים

49

Page 51: עינייםimg2.tapuz.co.il/CommunaFiles/39910353.doc · Web viewגם אם לאחר הפרוצדורה מתפתח PR, הלב הימני סובל את זה מצויין. נדיר

כירוגיית לב חזה

בסוף הניתוח משאירים נקזים במדיאסטינום קדמי ונקזים פלאורליים דו"צ )בניתוח פותחים את הפריקרד ולאלזכור הפלאורליים(. חשוב החללים של לפתיחה גם גורמת הפריקרד ופתיחת הניתוח, בסיום אותו סוגרים

שמיקום יציאת הנקז מהעור אינו אומר דבר על מיקום הנקז עצמו. . כאשר הניקוז100cc/hrחשוב לנטר את קצב הניקוז. הקצב התקין בשעות הראשונות שלאחר הניתוח הוא עד

יותר מהיר מכך נכנסים לסוגיה וטיפול ב"חולה המדמם".מעקב כאב ומתן אנלגזיה עפ"י הצורך

תרשים אק"ג פעימות מוקדמות הן שכיחות ולא מטרידות. –הפרעות קצב פחות שכיחות, יותר בניתוחי מסתמים. –הפרעות הולכה שכיח, הלב זז ממקומו התקין במהלך הניתוח. –שינוי ציר שינויST-T – .לא ספציפיים, לאו דווקא מעידים על איסכמיה גליQ אסורים !!! מעידים על – חדשים MI!

או איסכמיה סאב-ניתוחית היא בעייתית :MIהאבחנה של החולה במקרים רבים מורדם ומונשם, או סתם כאוב בגלל הניתוח. לכן, לא ניתן לסמוך על–כאבים

.MIהתלונות של החולה כאמצעי לאבחנת אנזימים. –אנזימים מכך משתחררים וכתוצאה ללב איאטרוגני נזק גורמים אנחנו הניתוח במהלך

CPK – כי נמצא CPK<50u .התייחסות מחייב לא איסכמיה. על מעיד ולא משמעותי אינו בדיקה חדשה, ולכן לא ברור מהו הערך ה"תקין" לאחר ניתוח. אין תועלת במדידת טרופונין–טרופונין

לחולה לאחר ניתוח, כי ממילא לא נדע איך להתייחס לתוצאה. אמצעי יותר אמין. גלי –שינויי אק"ג Q .חדשים מחייבים התייחסות

גדול הוא יצהיר על כך ע"י התדרדרות המודינמית. לכן, לא חייביםMIהנחת העבודה היא שאם החולה יעשה ליייחס חשיבות גדולה לכל שינוי אק"ג לא ספציפי. למרות זאת, אין לזלזל באיסכמיה סביב ניתוחית כי היא נמצאת

בקורלציה לתחלואה ותמותה פוסט-ניתוחית.בדיקות דם

, כימיה קצרה )אלקטרוליטים ותפקודי קרישה(. CBCבדר"כ נבדוק בדר"כ ההמוגלובין נמוך )–המוגלובין 8-9 gr%בגלל מיהול הדם ואובדן דם. לא ממהרים לתת דם ,)

לחולה אם הוא אין קליניקה. כמובן שחולה עם המוגלובין מאד נמוך יהיה סימפטומטי ויקבל דם. WBC – .מאפשרים לויקוציטוזיס מסויים, שהוא משני לסטרס של הניתוח תיקון הפרעות אלקטרוליטריות לפי הצורך. –אלקטרוליטים עדות לפרפוזיה כלייתית.–תפקודי כליה

צילום חזה מצלמים את החולה מיד לאחר הניתוח ובבוקר שאחרי הניתוח. בודקים:

טובוס, נקזים, –מיקום "גופים זרים" בחזה CVPזונדה , יכול להיות מורחב אם יש קרישים, המטומה.–רוחב מדיאסטינום האם קיים נוזל פלאורלי שלא נוקז ? האם יש אויר ?–חלל הפלאורה סימנים של תמט או גודש ?–שדות הריאה

פקודות ליממה השניהמיקום גאוגרפי של החולה

החולה נשאר ביחידת טיפול נמרץ רק למשך הזמן שבו הוא זקוק לטיפול נמרץ וניטור הדוק. ללא סיבוכים, ניתן 1:3 או 1:2(. ביחידת ביניים יש יחש אחיות של intermediate care unitבד"רכ להעביר את החולה ליחידת ביניים )

וניתן לעשות ניטור הדוק אבל לא ניטור חודרני. השארות עודפת ביחידה לטיפול נמרץ מעכבת את ההחלמה,למשל בגלל שהחולה לא יוצא מהמיטה.

ניטור החולהדופק, ל"ד, קצב נשימה+סטורציה, חום, תפוקת שתן, נקזים. )תת-כירורגיות מיוחדות–ניטור סימנים חיוניים

בנוירוכירורגיה, דופק היקפי בכיר' כלי-דם(.ICPמוסיפות ניטור:

50

Page 52: עינייםimg2.tapuz.co.il/CommunaFiles/39910353.doc · Web viewגם אם לאחר הפרוצדורה מתפתח PR, הלב הימני סובל את זה מצויין. נדיר

כירוגיית לב חזה

תדירות הניטור נתונה לשיקול דעת של הרופא: הניטור של דופק, ל"ד וסטורציה ביחדית הביניים עדיין יבוצע כל שעות. בכל מקרה, ביחידת ביניים הניטור הוא6שעה, אבל ניתן להסתפק בניטור חום או הפרשה מנקזים אחת ל-

(. port of entry ליממה נוספת כ-CVP, משאירים LAP ו-A-lineלא חודרני )מוציאים נוזלים והזנה

בעיקר היא הסיבה יקיאו. והם הניתוח הראשונה שלאחר ביממה ומזון נוזלים לשאת יוכלו לא החולים רב paralytic ileus אפילו בניתוח שאינו בטני( אך גם( stress ulcersבקיבה. עם זאת, החולה יוכל לשתות קצת מים

או לאכול מעט גבינה. )אין קונטרה-אינדיקציה להזנת חולה שרוצה לשתות או לאכול(. הוא אפס, ולכן צריך להחזיר לחולה נוזלים כדי שלא יתייבש. intakeהנחת העבודה ביממה הראשונה היא שה-

מ"ל של500 מ"ל שתן, 1500 מ"ל נוזלים ביממה )2000החזר הנוזלים הוא עפ"י אובדנים. החולה הרגיל מאבד כ-insensible loss-3 בהזעה ונשימה, אובדן זניח בנקזים(. הrd spacingשל החולה ביממה השניה לאחר ניתוח לב

הוא לא משמעותי )בניגוד לניתוח בטן(. הרופא צריך להחליט האם להחזיר בדיוק את האובדנים או שהחולהמיובש וזקוק ליותר נוזלים או גדוש וזקוק לפחות נוזלים. בכל מקרה הפקודה לנוזלים היא דינמית ומשתנה עפ"י ה-

intake.של החולה 1/2 או NSהתמיסה שאותה ניתו תלויה באובדן. חולה שאינו מדמם מאבד בעיקר נוזלים ומלחים. לכן, ניתן לו

NS קק"ל והן זניחות ביחס לתצרוכת. אין צורך להחזיר400, למרות שזה מספק רק 5%. בדר"כ נוסיף גלוקוז גדול עם אנסטומוזות ומחר החולה כבר יאכל.GIקלוריות כי זה לא ניתוח

תרופות מניעת דמם ממערכת העיכול בשלstress ulcers: zantac, losec בהתחלה מתן( IV.),אנטיביוטיקה מניעתית: בחירת האנטיביוטיקה היא עפ"י המזהמים הצפויים במהלך הניתוח. בניתוח חזה

שהוא ניתוח נקי, אלו בעיקר חיידקים של העור )בחולה שאינו אלקטיבי צריך להתחשב בשינוי הפלורה מנות במהלך היממה3של העור(. מקובל לתת מנה אחת של אנטיביוטיקה לפני החתך על העור, ועוד

הבאה, כדי למנוע זיהום בפצע הניתוח. אנטי-קואגולציה למניעת אירועים טרומבו-אמבוליים: כל החולים מקבליםclexane( במינון מניעתי 40

. הקלקסאן מופסק בשחרור. בניתוחי מסתמים כמובןimmobilisation מ"ג( בגלל 60 –מ"ג ליום, בשמנים שמתחילים גם חפיפה לקומאדין.

תרופות קבועות מהבית: ביממה הראשונה הספיגה עדיין בעייתית, ולכן עדיין לא מחדשים תרופות שאינן oralחיוניות. לדוגמא, לא נחדש סטטינים, hypoglycemicsזמנית באינסולין(, תרופות כמו )נשתמש

אלטרוקסין. כל אלו יחודשו ביממה הבאה. תרופות אנטי-אנגינוטיות שנלקחו לפני הניתוח לא יחודשואחריו, למעט חוסמי בטא.

בחולים שעברו ניתוח –אספירין CABGלא ניתוח מסתמים(. האספירין מטפל במחלה הכלילית שקדמה( )בעיקר בשתל ורידי(.graft occlusionלניתוח וגם מגן על השתלים ומונע

משככי כאבים: ניתן לתתNSAIDs( כמו וולטרן, אופטלגין, אקאמול. ניתן גם לתת אופייטים כמו מורפין 0.1mg/kg IV-ו )שילוב של אופייט חלש ב-NSAID( או טרמדקס נהוג להשתמש לא .)PCAאחרי

CABG .כי לרב לא צריך אנלגזיה כ"כ חזקה אחרי ניתוח חזה פזיותרפיה

חשוב לשים דגש על פזיותרפיה נשימתית )פתיחת תמטים, הקטנת סיכון לפנאומוניה(. זמינות הפזיותרפיסיטיות מוגבלת, ולכן גם הרופא והאחיות יכולים לעשות קצת פזיוטרפיה. ניתן גם להשתמש במכשירי נשימה ולעודד את

החולה להשתמש בהם.לוח זמנים לאחר הניתוח

התעוררות מהרדמה–( החולה יוצא מהניתוח מונשם ומורדם high dose Fentanyl .ומתעורר לאט ) שעות אחרי סיום הניתוח. 8-10אקסטובציה - לרב תבוצע הוצאתLAP בבוקר שלאחר הניתוח.–, הוצאת זונדה הוצאתa-line – .לפני המעבר ליחידת ביניים שעות. לרב ביממה השניה. 8 מ"ל ב-100 כאשר מפרישים פחות מ-–הוצאת נקזים CVP מושארים עד ליממה השניה שלאחר הניתוח, ואז מוצאים. –, קטטר פולי שעות ראשונות לאחר הניתוח24 –יחידת ט"נ

51

Page 53: עינייםimg2.tapuz.co.il/CommunaFiles/39910353.doc · Web viewגם אם לאחר הפרוצדורה מתפתח PR, הלב הימני סובל את זה מצויין. נדיר

כירוגיית לב חזה

שעות נוספות24 –יחידת מעבר ימים אחרי הניתוח 6 עד –אשפוז מחלקה .שחרור הביתה עם הנחיות מתאימות

הנחיות ובדיקות לקראת שחרור בדיקות והערכה לקראת שחרור הביתה:

-הערכה כללית: הרגשת החולה, כאבים, תאבון, תלונות נשימתיות, מערכת תמיכה, עצמאי בADL ? הערכה קרדיו-וסקולרית: האם החולהwell perfused ? .הערכה נשימתית: סטורציה באויר חדר, כניסת אויר טובה. צילום חזה לפני השחרור שעות ללא חום.48מעקב חום - חולה לא משתחרר עד שעברו ? מצב הפצעים: האם הצלקת תקינה ? יש אודם מקומי ? הפרשה אנטי-קואגולציה: האם הושגINR ? מטרה לאחר החלפת מסתם ? תרופות: אילו תרופות החולה קיבל באשפוז ? אילו תרופות צריך להמשיך בבית ? מחלות רקע: האם חודש טיפול תרופתי ? האם החולה מאוזן טרם שחרורו)האם החולה חזר למשקל טרום-ניתוחי )איבד את הבצקות סביב הניתוח.לעבור על גליון האשפוז, לוודא שלא פוספסו אירועים חריגים

הנחיות והסברים לחולה: פעילות גופנית - אין הגבלה רפואית על פעילות, ניתן לעשות לפי ההרגשה. להנחות את החולה שלא

דקות ביום, עליה הדרגתית עד שעה ביום. 10-15לשכב במיטה או להשאר בבית. הליכה .תזונה - דיאטה מאוזנת. הגבלת שומן, דיאטה נטולת סוכר בחולה סכרתי שבועות. הסיבה: חלק מהחולים מקבלים אנלגזיה שכוללת אופייטים ולכם3-4נהיגה - אסור לנהוג במשך

עלולים להיות מטושטשים ; יש סכנה משמעותית בתאונת דרכים עם פגיעה בסטרנום כאשר הוא עדייןלא החלים במלואו )דבר שלוקח מספר שבועות(.

CABG הרגל ממנה נלקח השתל - ויחלוף )לפחות חלקית(.תותרנ נפוחה מהרגל הנגדית. זה נורמלי .בינתיים: חבישה אלסטית על הרגל במשך היום, הרמת הרגל

צריך לספק למטופל הסבר מדוקדק לגבי אנטי-קואגולציה. צריך להגדיר את ה-–מסתמים INRמטרה ולהנחות את החולה כיצד עליו לנטר את מינון הקומאדין )פעם בכמה זמן בדיקות דם, קשר עם רופא

משפחה וכו'(. .משככי כאבים: החולה משתחרר עם משככי כאבים בהתאם לתרופות שקיבל בימים האחרונים לאשפוז

ניתן לתת פרקוסט למספר ימים, ואח"כ לרדת לאספירין ואז להפסיק. ביקורת במרפאה של כירורגית לב-חזה כעבור - )בעיקר4מעקב שבועות מהניתוח, באופן חד-פעמי

4-6ביקורת של החלמת פצע הניתוח(. המשך המעקב הוא ע"י הקרדיולוג המטפל - פגישה ראשונה שבועות אחרי הניתוח, ואח"כ לפי שיקול הדעת של הרופא ולפי מצב החולה.

.הנחיה להתקשר למחלקה בכל מקרה של בעיות לאחר השחרור - כאבים, פצע מפריש, חום

ניטור המודינמי: מהלך פוסט-ניתוחי לא תקין

צריך לחפש סיבה: –חולה שעדיין מורדם ומונשם בבוקר השני לאחר הניתוח o ? ו/אופגיעה נוירולוגיתo? ו/אופגיעה נשימתיתo ? פגיעה קרדיו-וסקולרית

סוג הניתוח שהחולה עבר, סיבוכים–המהלך הפוסט-ניתוחי של החולה קשור בקשר הדוק למהלך הניתוחי שלו במהלך הניתוח. בחולה עם מהלך ניתוח קשה, צפוי שיהיו סיבוכים בהחלמה.

הערכת הסביבה הגאוגרפית

52

Page 54: עינייםimg2.tapuz.co.il/CommunaFiles/39910353.doc · Web viewגם אם לאחר הפרוצדורה מתפתח PR, הלב הימני סובל את זה מצויין. נדיר

כירוגיית לב חזה

הסביבה הגאוגרפית של החולה יכולה לרמוז על מצבו והבעיות הפעילות שלו, ולכוון את הערכת מצבו. יש לשים של תרופות הואdrips, שוואן-גנץ, קטטר? איזה CVP נכנסים ויוצאים מהחולה - הנשמה, קוצב, linesלב: איזה

מקבל - יונוטרופיות, סדציה, דם? סדציה או תמיכה המודינמית יוצרות הטיה בהערכת הקלינית.

הערכה נוירולוגית ( בדר"כ לא יגיב להוראות ורבליות. ההערכה הנוירולוגית מוגבלת מאד ע"י הסדציה. מותרsedatedחולה מורדם )

וצריך להפסיק את הסדציה זמנית. ממקם ? תגובה פלקסורית? –ניתן להעריך מצב הכרה ע"י תגובה לכאב

טונוס שרירים, רפלקסים, בבינסקי?–הערכה נוירולוגית בחיפוש סימני צד הערכה נשימתית

בחולה מונשם חשוב להסתכל על מדדי הנשימה: האם החולה מוסיף נשימות על המכונה ? - של כל נשימה ?tidal volumeמה ה--מהם לחצי ההנשמה הנדרשים ?- ?FiO2מהי הסטורציה ? באיזה -

יכול לנשום בעצמו, בניגוד לחולה שמקבל גם שיתוק שרירים.sedatedחולה שהוא פתוח, ופחות להנשמה.airwayישנם מקרים שבהם הטובוס משמש בעיקר לשמירת

הערכה קרדיו-וסקולרית ?פרפוזיה פריפרית: האם הגפיים חמות? האם יש דפקים פריפרייםתפוקת השתן, תפקודי כליה: תפוקת שתן נמוכה והחמרה בתפקודי כליה יכולות להעיד על תפוקת לב

נמוכה, אבל חייבים לבדוק מה היו תפקודי הכליה טרם הניתוח. כך למשל תפוקת שתן נמוכה בחולהשממילא סבל מהפרעות בתפקוד הכליה מטילה ספק על המסקנות לגבי הפרפוזיה של הכליות.

היה האם לשאול צריך הכלייתי בתפקוד פוסט-ניתוחית הפרעה )insultבנוכחות הניתוח בזמן hypotensionממושך, שימוש ארוך במכונת לב-ריאה( וגם לשאול האם תפוקת הלב תקינה לאחר הניתוח

)היפוולמיה ? אירוע איסכמי ?(. לחצי מילוי: באמצעותCVP או LAPבחולה הלא יציב נכניס צנתר שוואן-גנץ שנותן יותר מידע מאשר .

.LAP וה-CVPלחצי המילוי שמתקבלים מה--קוצב חיצוני: בCABGובעיקר בניתוחי מעקפים עלולה להיות פגיעה במערכת ההולכה. כאשר מופיעות

מספק, צריך לקצב את החולה. לכן, משאירים בעת הניתוחCOהפרעות הולכה שבגללן צפוי שלא יהיה חוטי קוצב אפיקרדיאליים )בניגוד לחוטי קוצב קבוע שהם אנדוקרדיאליים( ומחברים אותם לקוצב זמני. ישנה מוסכמה לפיה חוטים שיוצאים מימין לקו האמצע הם עלייתיים, וחוטים משמאל לקו האמצע הם

חדריים. ניתן לשלוט על הגדרות הקיצוב עפ"י צרכי החולה. חשוב להפסיק מדי פעם את הקיצוב כדי לדעת מהו

הקצב הבסיסי של החולה. הפרעות ההולכה יכולות להיות הפיכות )איסכמיה שתחלוף( או קבועות )תפר על סיבי ההולכה(. לכן,

ימים לאחר הניתוח. 4-5ההחלטה על השתלת קוצב תתקבל רק אופטימליזציה של מצב החולה

נוירולוגית: מהלך אבחנתי ולא טיפולי, אך מוצדק להתייעץ במומחים.–הערכת נוירולוג CT בירור האטיולוגיה )איסכמי, תסחיפי, דימום(. לא טיפולי.– מח לא להוריד ל"ד יותר מדי.–הגנה על פרפוזית המח וחמצונו תוספת –אנטי-קואגולציה אם יש חשד גבוה לתסחיף IV Heparin.לקלקסן המניעתי

נשימתית: .הנשמת החולה שאינו מסוגל לנשום או מתעייף טובוס כהגנה עלairway .בחולה מורדם, גם אם אינו מונשם מניעה של התפתחות פנאומוניה ע"יsuction.פזיוטרפיה ,

53

Page 55: עינייםimg2.tapuz.co.il/CommunaFiles/39910353.doc · Web viewגם אם לאחר הפרוצדורה מתפתח PR, הלב הימני סובל את זה מצויין. נדיר

כירוגיית לב חזה

כלייתית: insult בזמן הניתוח או אחריו, הכליות צפויות להתאושש בתנאי שנתקן את ה-insultאם הנזק הכלייתי נגרם ע"י

הראשוני ונמנע נזק נוסף. הרב המוחלט של החולים ימשיכו לתת שתן )גם אם איכותו ירודה( ולא יגיעו לדיאליזה. חולים מעטים כן יעשו אנוריים ונצטרך להעריך האם יש אינדיקציה לדיאליזה. החולים שלא מתאוששים הם

בדר"כ אלו שסבלו מפגיעה כלייתית טרם הניתוח. עושים אופטימליזציה של מצב הכליות ע"י:

שמירת ל"ד תקינים.–שמירת פרפוזיה טובה מניעת התייבשות, מעקב –שמירת לחצי מילוי CVP.)להמנע מתרופות נפרוטוקסיות )לחשוב על כל תרופה לא דרוש כל עוד החולה משתין.–שיתון

קרדיו-וסקולרית: כאשר הפרפוזיה הפריפרית תקינה )ואפילו בנוכחות פגיעה כלייתית( לרב אין צורך בהתערבות. כאשר יש

ע"י שיפור מדדי:COפגיעה בפרפוזיה פריפרית ננסה להגיע לאופטימליזציה של oלחצי מילויoContractilityo הורדתafterload

.ניטור הדוק של מדדים המודינמיים ע"י קטטר שוואן-גנץמוצא LAPצנטר ה- שעות לאחר הניתוח בכל מקרה. אם החולה לא יציב במידה שמחייבת ניטור24

COחודרני, ממילא נעדיף להכניס שוואן-גנץ שנותן יותר מידע המודינמי מאשר לחצי המילוי - מדידת ועוד.PA, רוויון חמצן ב-SVR, מדידת thermodilutionישירה ע"י

הזנה . תמיד נעדיף הזנהIVבחולה שצפוי להיות מונשם ומורדם למספר ימים יש צורך במתן הזנה ולא רק נוזלים

, אבל חייבים לוודא קודם שהחולה סופג את המזון.TPNאנטרלית )ע"י זונדה( על-פני מניעת סיבוכים צפויים:

DVT/PE קלקסן מניעתי פנאומוניה וסיבוכים נשימתיים ,פזיוטרפיה suctionline sepsis החלפת central lineמניעתית מדי שבוע, כדי למנוע התפתחות של בקטרמיה שיכולה

. EBMלהיות מסכנת חיים. זהו נוהג מקובל, אבל הוא אינו מגובה ב- פצעי לחץשינויי תנוחה תכופים, מעקב הדוק חשש מקונטרקטורות)פזיוטרפיה, ריפוי בעיסוק )התאמת סד זיהומים מעקב חום וספירת WBC.לא מקובל לתת אנטיביוטיקה מניעתית .

סיבוכים לאחר ניתוח

לאחר ניתוח וגורמים לאיזון סיבוכים דלקת / זיהום–חום הפרעות קצב איזוןCHF תפליטים פלאורלייםאיזון אנטי-קואגולציה לאחר ניתוח מסתמים

דמם לרב לא מאריך משמעותית את האשפוז. בדר"כ מתגלה בשעות הראשונות לאחר הניתוח ויכול לחייב ניתוחחוזר, ובדר"כ מאריך את האשפוז בטיפול נמרץ, אבל השחרור הביתה יהיה בזמן.

DVT/PEמאד לא שכיחים בניתוחי-לב )בניגוד לניתוחי בטן, אורתופדיה(, כי במשך רב זמן הניתוח החולה הוא fully anticoagulated-בגלל השימוש במכונת לב-ריאה, ולכן לא יכול ליצור קרישים )במחלקה כירורגית רגילה, ה

DVTs.).נוצרים בניתוח, לא אחריו

54

Page 56: עינייםimg2.tapuz.co.il/CommunaFiles/39910353.doc · Web viewגם אם לאחר הפרוצדורה מתפתח PR, הלב הימני סובל את זה מצויין. נדיר

כירוגיית לב חזה

ישנם שלושה גורמי תחלואה קשה עיקריים לאחר ניתוחי-לבמוחי ים ונזקאירוע "דמם - ראה שיעור "החולה המדמם.מדיאסטיניטיס

נזקים נוירולוגיים סוגים של סיבוך נוירולוגי: 2

Major – ארוע של CVA.איסכמי, המורגי, תסחיפי( עם חסרים פוקאליים( Minor – .אנצפלופתיה, בלבול, ללא חסר מוטורי

גורמי סיכון של החולה: ).גורמי סיכון כלליים )גיל, תפקוד לבגורמי סיכון למחלה צרברו-וסקולרית ארוע קודם שלCVA או( TIA)AFעם קרישים בעליה

מחלה בעורקי התרדמהמסתם מסוייד שבהחלפתו ישלחו קרישיםאאורטה מסויידת שעלולה לשלוח תסחיפים

גורמי סיכון של הניתוח: )פתיחת לב שמאל )מחזור דם סיסטמי, התסחיפים ילכו למח: שימוש במכונת לב ריאה

oזרימה למינרית לא פזיולוגיתoאירועים של צניחה בלחצי פרפוזיהoצניחות ל"ד ושינויים המודינמיים קשיםo.)מניפולציות על האאורטה ששולחות תסחיפים )אויר, קרישים, הסתיידויות

DHTCA )deep hypothermia total circulatory arrest( –טכניקה ניתוחית שבה נעשה שימוש בתיקון דיסקציה או אניוריזמה של האאורטה. במקרה כזה עוצרים לזמן קצר את אספקת הדם.

. 7% - הסיכון כ-75. לאחר גיל 1% הוא כ-good LV עם 65 בחולה עד גיל CVAהסיכון ל-על הלב אנחנו יודעים כיום להגן, על המח פחות.

הערכת פצע לאחר ניתוח :הסתכלות

o ? שלמות הפצעo )?הפרשה )מוגלה ? דםo למה ? –אם חבוש

:סימני דלקתoאודםoחום מקומיo)כאב )לא לבלבל עם כאב בגלל החתך הניתוחיo חום, לויקוציטוזיס ?–סימנים סיטמיים

( אצבעות ומבקשים מהמנותח2(: לוחצים משני צידי הסטרנום ב-mid-sternotomyיציבות הסטרנום להשתעל. כאשר הסטרנום לא יציב נוצרת מדרגה = זהו אחד מסימני זיהום של הסטרנום.

זיהום בפצע הניתוח בחזה. הסיבה היא בעיקרgraftזיהום פצע ברגל ) יותר שכיח מאשר למעקפים( הרבה דיסקציה לא טובה של וריד הספנוס יוצרת מתלי עור איסכמיים, וברגל גם פחות מקפידים על–טכניקה כירורגית

שמונע ריפוי תקין של הפצע ברגל.PVDשדה סטרילי בהשוואה לחזה. בנוסף, חלק מהחולים סובלים גם מ-קשה לפעמים להבחין בין זיהום בפצע הניתוח לבין צלוליטיס )פחות שכיח אבל אפשרי, בדר"כ ברגל ולא בחזה(.

( עלולmediastinitisהטיפול בפצע ניתוח שאינו מאד עמוק הוא ע"י אנטיביוטיקה לוריד. לעומת זאת זיהום עמוק )לחייב ניתוח חוזר.Mediastinitis

Mediastinitis = Deep sternal wound infection ימים לאחר הניתוח. שכיח10. סיבוך קשה שמתבטא לפחות שהחולה ילך הביתה ואז יחזור למיון. ביטוי מקומי וסיסטמי משולב - פצע מזוהם ומפריש, סטרנום לא יציב, חום

ולויקוציטוזיס.

55

Page 57: עינייםimg2.tapuz.co.il/CommunaFiles/39910353.doc · Web viewגם אם לאחר הפרוצדורה מתפתח PR, הלב הימני סובל את זה מצויין. נדיר

כירוגיית לב חזה

התמונה יכולה להיות יחסית קלה, אבל יכולה להגיע עד לספסיס קשה. האבחנה היא היא קלינית, ואסור לפספסאותה !!!

גורמי סיכון למדיאסטיניטיס: obesity סוכרת לקיחתdouble mammary )בפרט בסכרתיים(ניתוח ארוך - מעלה שכיחות של זיהומים מכל סוג)דיכוי חיסוני )סטרואידיםCOPD

כל את כורתים )שומן(, והתת-עור העור של נרחבת הטריה עושים כירורגי. הוא במדיאסטיניטיס הטיפול הסטרנום )העצם אוסטאומיאליטית וחלשה(. משתדלים לסלק את כל הרקמה הנמקית, אבל הזיהוי שלה קשה )אין קולקציה ברורה(. לאחר ההטריה, נשאר חסר משמעותי בדופן בית החזה )הלב חשוף( וצריך לשחזר את

הדופן. ניתן לעשות את הרקונסטרקציה באותו הניתוח כמו בהטריה, או במועד יותר מאוחר: אם לא בטוחים שכל האזור

24הנמקי עבר הטריה, משאירים את החולה מורדם ומונשם עם חבישה סטרילית על החזה הפתוח, וכעבור שעות.24שעות מחזירים את החולה לחדר-ניתוח להערכה מחודשת. אם עדיין לא בטוחים - ממתינים עוד

failureמשחזרים את הדופן רק לאחר שמשתכנעים שהנמק נוקה במלואו. היתרון בשיטה זו הוא הוא שיעור )חזרת זיהום( מאד נמוך. שתי הגישות מתקבלות על הדעת בתלות בנסיבות.

טכניקות: 3הרקונסטרוקציה מתבצעת בסיוע של פלסטיקאים, באחת מ- .מתלה דו"צ של הפקטורליס ומעליו סגירה ראשונית של העור-מתלה של שריר הרקטוס. המגבלה - אם לקחנו את הLIMAלצורך מעקפים, חייבים לקחת את שריר

הרקטוס השמאלי כדי שתוכל להתקיים אספקת דם. בנוסף, הרקטוס הוא שריר צר ולכן לא יכול לכסותחוסר גדול של דופן )ולא ניתן לקחת רקטוס דו"צ(.

פתיחת הבטן והעברת האומנטום לכיסוי החוסר בדופן בית החזה, כאשר מעל האומנטום עושים שתל עור (skin graft .נשאר פגם אסטתי בדופן .)

.flapלאחר השחזור של בית החזה יש סיבוכים איאטרוגנים של לא נגרמת הגבלה נשימתית )רסטרקטיבית(. צריך להזהר מפני טראומה לחזה.

שכן השחזור דבוק ללב !midlineמובן שלעולם לא נוכל לבצע ניתוח נוסף בגישת למשך שבועות רבים.IVבנוסף לטיפול הכירורגי צריך, כמובן, אנטיביוטיקה

- פחות מ- ובזמן - 1%השכיחות המתקבלת על הדעת של מדיאסטיניטיס מהניתוחים. בטיפול מתאים 10% ויותר(.50%תמותה. ללא טיפול - תמותה עצומה )

תפוקת לב נמוכה לאחר ניתוח גדולאבחנה מבדלת:

היפוולמיה.1היפוולמיה.2היפוולמיה.3)בניתוחי לב(בעיה מכאנית / טמפונדה .45.LV dysfunction

איסכמיה חריפהבעיה טרום-ניתוחיתנזק נרכש בניתוחבעיה מסתמית חריפה

6.severe pulmonary HTNגורם כשל ימני - מומים מולדים - כאשר התיקון לא מושלם.7

נמוך: COעדויות ל-פריפריה קרה

תפוקת שתן↓ לחץ דם↓

56

Page 58: עינייםimg2.tapuz.co.il/CommunaFiles/39910353.doc · Web viewגם אם לאחר הפרוצדורה מתפתח PR, הלב הימני סובל את זה מצויין. נדיר

כירוגיית לב חזה

אצידוזיס סטורציה בדם ורידי↓

: COשיפור טיפול בגורמים הפיכים

לחצי מילויcontractility הורדתafterload

לאחר ניתוחCHFאיזון בחולה שסבל מאיס"ק לב קשה משנית למחלה מסתמית, התיקון המכאני של מסתמי הלב לא מביא מיידית

הוא ממושך ולפעמים לא מלא, כי חלק מהנזק שנגרם ללב אינו הפיךCHF. האיזון המחודש של CHFלשיפור ב-ימים, צריך )החדר כבר מוגדל מאד, הקונטרקטיליות פגועה מאד(. כאשר החולה לא מתאושש בתוך מספר

לבדוק: האם התיקון המסתמי קומפטנטי ? האם יש טמפונדה ? מהו התפקוד הלבבי ? מה מצב הנפחים ? Post-Surgical New-Onset Atrial Fibrilation

AFמכלל החולים מפתחים 1/3 - חדש שמתפתח לאחר ניתוח לב הוא הסיבוך הכי שכיח לאחר ניתוח AF,חדש . הפרפור מתפתח לרב באופן70%ובאוכלוסיות חולים מיוחדות )קשישים, ניתוחי מסתמים( השכיחות מגיעה עד

ביום - עליה בפעילות סימפטטית ? איסכמיה של2-3קלאסי ולא מיידית. המנגנון לא ברור לאחר הניתוח, העליה ?

משמעות קלינית: .הפרפור בדר"כ סימפטומטי - פלפיטציות.יכול להיות אירוע יחיד, או מספר אירועים, באורך משתנה-לא יחזרו ל( רב מוחלט של החולים חוזרים לקצב סינוס יציבAF)-אי-יציבות המודינמית בחולים שזקוקים לatrial kick . אם הפרפור ממושך - סכנת תסחיפים .הארכת אשפוז ועלויות כלכליות

מניעהלאחר ניתוחים גרמה לבדיקת אפשרויות מניעה - מחקרים גדולים על מגווןAFהשכיחות הגבוהה של הופעת

, דיגוקסין( הראה הורדת שכיחות מינימלית, שאינה משמעותית. המניעה המקובלת היא ע"י חוסמיCCBתרופות )βממשיכים בטיפול אם החולה קיבל אותו קודם לניתוח, או שמתחילים טיפול בחוסמי בטא ביממה כיממה -

אחרי הניתוח, בתנאי שאין קונטרה-אינדיקציות. לא מקובל לתת אמיאודרון )פרוקור( מניעתית, רק טיפולית. :new onset AFעקרונות טיפול ב

א( חולה לא יציב - היפוך מיידי ע"י מכת חשמל ! עקרונות טיפוליים: 4ב( חולה יציב -

דיגוקסין )התרופה היחידה שלא(, IV, אמיאודרון )העמסה, CCBהאטת התגובה החדרית - חוסמי בטא, .1 וזה יכול להיות קריטי בחולי-לב שעלולים להיות לא יציבים סביב הפרפרור(.BPמורידה

היפוך הקצב - בדר"כ פרמקולוגי, ע"י התרופות הנ"ל. נדיר היפוך חשמלי בחולה יציב. .2 - שמירת קצב סינוס ע"י תרופות אנטי-אריתמיות )פרוקור, ריתמקס(. משך הטיפולmaintenanceטיפול .3

post-surgicalהאנטי-אריתמי שנוי במחלוקת, אך בדר"כ מספר שבועות. new AFלא מחייב טיפול טיפול לכל החיים( מסיבה אחרת בלב עם פגיעה מבנית - AF)אנטי-אריתמי לכל החיים

- מספר שבועות סביב האירוע החריף.אנטי-קואגולציה.4הגישה לחום אחרי ניתוח

חום ביממה הראשונה לאחר ניתוח נגרם בדר"כ ע"י אטלקטזיס ראתי, והוא אינו זיהומי. הטיפול היחיד בהקשר זההוא פזיוטרפיה נשימתית כדי שהחום לא יהפוך לפנאומוני.

. הזיהום יכול להיות באתר הניתוח, או באחת מהמערכות המעורבות5-10זיהום לאחר ניתוח מופיע בדר"כ בימים (. הנסיבות יכולות לכוון למקור הזיהום - פנאומוניה שכיחה יותר בחולה עם מחלת ריאהlines)נשימה, שתן,

לרב לא יתפתח ללא קטטר ממושך, UTIברקע, line-sepsisלא יקרה אם הצנתרים הוכנסו בטכניקה טובה וטופלו כראוי. )אגב, אין הנחיה מיקרוביולוגית להחליף צנטר מרכזי שאינו מזוהם, אך בפועל מחליפים צנתרים

עלול להיות קטלני(.line sepsis ימים כי 7-10באופן מניעתי כל זיהומים מערכתיים קשים לרב "יצהירו על עצמם" מבחינה המודינמית, ולא יתבטאו רק בחום או לויקוציטוזיס. לכן, כאשר יש חום ללא הסבר ניתן להסתפק במעקב, והוא בדר"כ יעלם מעצמו. צריך להתאפק שלא להתחיל

57

Page 59: עינייםimg2.tapuz.co.il/CommunaFiles/39910353.doc · Web viewגם אם לאחר הפרוצדורה מתפתח PR, הלב הימני סובל את זה מצויין. נדיר

כירוגיית לב חזה

כנגד "חום של תגובה דלקתית" )שהיא שכיחה במיוחד לאחרNSAIDsטיפול אנטיביוטי ללא מקור. לעיתים ניתן שימוש במכונת לב-ריאה(.

כאשר יש ממצא )מקרי( של לויקוציטוזיס צריך להעריך את החולה קלינית, לבדוק האם יש חום. נשלח תרביות בחולה מבוגר.(. להתאפק עם אנטיביוטיקה. CLLדם ושתן ונבקש משטח דם )אבחנה מקרית של

Post Pericardiotomy )Dressler`s) Syndrome שבועות לאחר3-4 - פריקרדיטיס דלקתי )לא זיהומי( שמתפתח הניתוח )כאשר החולה כבר מזמן בבית(. הקליניקה לא ספציפית - כאבים בקדמת ביח"ז, חום, שקיעה מוחשת )

WBC בדר"כ תקינים(. אבחנה per-exclusion - הטיפול .NSAIDs .כירורגיה בחולה הקשיש

. גיל בפני עצמו מהווה גורםCABG לחייהם, כולל ניתוחים גדולים כמו 80כיום לא חריג לנתח חולים גם בשנות ה-סיכון מבודד לתמותה ניתוחית ותחלואה פוסט-ניתוחית מוגברת.

ולכן חייבים יותר קשה. ולפעמים יותר ממושך בחולה הקשיש חייבים לצפות למהלך פוסט-אופרטיבי שונה, להצדיק את הסיכון המוגבר של הניתוח - חולה "צלול ועצמאי", "צעיר לגילו", "צעיר ביולוגית".

ההתאוששות של החולה הקשיש היא יותר איטית, גם בהיעדר אירועים חריגים. החולה בדר"כ צריך יותר עזרהתפקודית-סיעודית ע"י האחיות / המשפחה.

בלבול הוא מאד שכיח בחולה הקשיש, ואינו מעיד על נזק נוירולוגי או נזק ארוך טווח. הבלבול מוגבר ע"י שהיהממושכת ביחידת טיפול-נמרץ )היעדר תחושת יום ולילה, היעדר גירויים חברתיים.(.

,%10grבחולה הצעיר לוקחים יותר מרווחי בטחון. לדוגמא - מקפידים בחולה הקשיש נשמור על המוגלובין מעל גם אם זה מחייב מתן דם )מהלך שבדר"כ נמנעים ממנו בחולה צעיר, בו ניתן דם רק באנמיה סימפטומטית(. בכלנשימתית פזיוטרפיה זה מודגש: בחולה הקשיש "לכבות שריפות", אבל ולא בעיות חולה תמיד עדיף למנוע

.AF, טיפול אגרסיבי ב-DVTלמניעת פנאומוניה, מוביליות למניעת פצעי לחץ או כירורגיה בחולה עם השמנת יתר

שיקולים ייחודיים בחולה השמן: השמנת-יתר מהווה גורם סיכון מבודד לתמותה ותחלואה מחלות נשימה רסטרקטיביות, סיכון שלOSAImmobility ,בעיות נשימה - DVT/PEקשיים בריפוי פצע ניתוח

,6-7כמו בחולה הקשיש, צפוי שההמהלך הפוסט-ניתוחי לא יהיה חלק. החולה הזמן בדר"כ לא ילך הביתה ביום אלא מעט אח"כ.

החולה הקורס ימים לאחר ניתוח להחלפת מסתמים, מהלך פוסט-ניתוחי ללא ארועים מיוחדים.7. 65תאור מקרה: חולה בן

בערב טרם השחרור, החולה מתמוטט באופן פתאומי במחלקה, ובהערכתו - ברדיקרדיה קיצונית, ל"ד לא נמוש.אבחנה מבדלת:

הפרעות קצב - הכי שכיח, מאד מסוכן. לשלולVT/VF.לפני כל דבר אחר איסכמיה חריפה - תסחיף קורונרי

דימומי )עודף אנטי-קואגולציה( או אמבולי )מיעוט אנטי-קואגולציה( CVAשבץ מוחי - רלוונטיים: שאינם יפים רעיונות

.טמפונדה מאוחרת - הצטברות איטית של דם בפריקרד. מהלך הדרגתי, לא התמוטטותAcute dissection מתרחש בזמן הניתוח בשל - clamping & de-clamping .לא אח"כ ,acute MR.מסתם מכאני שנתקע במצב פתוח, גורם בצקת ריאות הדרגתית, לא קריסה - stuck valve אין סגירה פתאומית של מסתם מכאני ! זה יגרום למוות מיידי כי איו - COבכלל ! יכולה

להיות תנועה לא תקינה של עלי מסתם מכאני, אבל זן בעיה שמופיעה שנים אחרי הניתוח, בהדרגתיות.valve stenosisאיטית, עם סימנים של

aspiration .נותן תמונה של קוצ"נ, לא התמוטטות - מה עושים ? החייאה !!! - האטיולוגיה לא חשובה אם החולה מת.

ואחרי שהחולה "חזר לחיים".

58

Page 60: עינייםimg2.tapuz.co.il/CommunaFiles/39910353.doc · Web viewגם אם לאחר הפרוצדורה מתפתח PR, הלב הימני סובל את זה מצויין. נדיר

כירוגיית לב חזה

)בדיקה פיזיקלית )בדגש על בדיקה נוירולוגית ? אק"ג - הפרעות קצב ? איסכמיה צל"ח? כימיה בדם - הפרעות אלקטרוליטריותABGs).הערכת חמצון, אוורור, חמצת )לאחר ההחיאה - INR? האם האנטי-קואגולציה למסתם מאוזנת - )אנזימי לב )לזכור שהם עולים בזמן החייאה ? אקו לב - תפקוד הלב ? המסתם התותבCT לא נראה אוטם אמבולי, וממילא יש( ראש לשלילת דימום absolute CI.)לטרומבוליזה אחרי ניתוח

האבחנה: אוטם אמבולי נרחב, משני לאנטי-קואגולציה לא מספקת. החולה חסר הכרה, מונשם, חסרים נוירולוגיים מתאימים לאוטם בזילרי.

: CVAמה עושים עכשיו ? הגישה לחולה לאחר נשימתית: טובוס לשמירתairway.הנשמה אם צריך , .קרדיו-וסקולרית: החולה בדר"כ יציב. להקפיד על אנטי-קואגולציה למסתמים: "מניעת בעיות ולא "כיבוי שריפות

o-מניעת זיהומים - מעקב חום וWBC סקשן למניעת פנאומוניה, החלפת ,lines.oמניעת פצעי לחץ - שינויי תנוחה תכופיםo שמירת פרפוזיה - מעקבABGs.מעקב תפוקת שתן ,o תזונה - רצוי זונדה )אם סופג(, כיסויstress ulcersע"י זנטק

,משפחת החולה - המשפחה בדר"כ במצוקה איומה. חשוב לדבר איתם, להסביר, לתמוך! אסור, אף פעם להתעלם מהמשפחה, גם אם השיחה איתם קשה לנו.

מצבי חירום בחולים לאחר ניתוח לב

חולה מדמם לאחר ניתוחהאבחנה המבדלת החשובה בחולה המדמם לאחר ניתוח לב בפרט וניתוח בכלל היא בין:

דמם המטולוגי– oozing.ממס' רב של מקומות בשל פגיעה במנגנון הקרישה דמם מתפר או חתך. גם כאשר מנגנון הקרישה תקין.–כירורגי

כדי להבדיל בין השניים נתבונן בדם אשר יוצאים מהנקזים שהוחדרו לחולה. קיום קרישים בצנרת הנקזים מצביע על כך שמדובר שתפקוד הקרישה, בכלליות, הינו תקין ולכן מחשיד שמדובר בדמם בשל בעיה כירוגית. בבעיה כירוגית יש לשקול טיפול כירוגי חוזר. לעומת זאת דם טרי שאינו נקרש בצנרת מכוון לבעיה המטולוגית. במצב זה

יש לטפל בגורם ההפרעה ופתיחה מחדש של האזור הניתוחי לא תעזור.דמם המטולוגי

החלוקה בין הגורמים השונים לדמם המטולוגי מתבססת על השוני בטיפולים בין הקבוצות השונות. עודףHeparin. ירידה בטסיות כל השאר

עודף הפארין במהלך הניתוח. heprinזהו הגורם הכי שכיח בניתוחי לב בשל השימוש ב

ואלו)3mg/Kg )1mg=100u. נותנים הפארין במינון של full anticoagulationבעליה על מכונת לב ריאה עושים היא כ-DVTמנות ענק. לשם השוואההעמסת הפארין ל- יחידות, וכאן נותנים לאדם בעל משקל ממוצע5000

אבל זה איטי מדי לניטור בניתוחPTT יחידות. המעקב אחר האנטי-קואגולציה בהפארין יכול להיות ע"י 21,000 דקות, וחייבים להיות בטוחים, כל הזמן, שרמתו מספקת(. השיטה היא45לב )זמן מחצית חיים של הפארין הוא

59

Page 61: עינייםimg2.tapuz.co.il/CommunaFiles/39910353.doc · Web viewגם אם לאחר הפרוצדורה מתפתח PR, הלב הימני סובל את זה מצויין. נדיר

כירוגיית לב חזה

במהלך הניתוח כל חצי שעה. החולה הוא )ACT )activated clotting timeע"י מדידת fully anticoagulated שניות.450 שלו או שהערך הוא מעל baselineכאשר שילשנו את ערך ה-

הבדיקה היא ע"י מכונה הנמצאת בחדר הניתוח וביחידת התאוששות ונותנת תשובה מיידית. בסיום הניתוח סותרים את האפקט של ההפארין ע"י פרוטאמין סולפט. זהו בסיס חזק עם מטען חיובי שנקשר

.baseline כדי לראות שחזרנו ל-ACTבקשר בלתי הפיך להפארין הטעון שלילית. נותנים בערך מ"ג למ"ג, ובודקים רק אז יוצאים מחדר הניתוח.

ובכל זאת, יש הפארין שהתחבא ברקמות השומן שכן המטען השלילי שלו מאפשר לו לחדור לרקמות שומניות. של הפארין מהשומן לדם, וזה תהליך משמעותי מאד בחולה השמןrecruitementלאחר סתירת ההפרין בדם יש

. הטיפולACT)שאינו נדיר(, והוא קורה בתוך כחצי שעה. ניתן לזהות את הבעיה בחולה המדמם ע"י מדידה של היעיל למצב זה היא תוספת של פרוטאמין עד לנרמול המדידה.

מנגנונים: 2חשוב לזכור: פרוטאמין היא תרופה מאד מסוכנת ויכולה להרוג את החולה, ב- יש רגישות אנפילקטואידית לפרוטאמין, כי זהו חלבון שמקורו מהחי )זרע של דגי סלמון(. החולים שבסיכון.1

הם:o חולי סוכרת שמטופלים באינסוליןNPH .שמכיל פרוטאמין ולכן הם פתחו נוגדנין לתרופה o חולים שכבר עברו ניתוח לב וקיבלו פרוטאמין בעברo חולים עם נטיה לרגישות יתרo חולים שעברו ניתוחvasectomy .באשך

פרוטאמין עושה וזודילטציה פריפרית, ברונכוספאזם ועליה בתנגודת הריאתית. המולקולה דומה לקינינים..2 first pass, כך שהחומר יעבור בכבד לפני שהוא יגיע לריאות ויתפרק שם )LAPלכן, ההזרקה היא אל ה-

effectזו מזריקים את וירידת ל"ד. כדי להקטין את השפעה וזודילטציה עורקית (. כך נשארים רק עם החומר בצורה איטית.

טרומבוציטופניה מכונת לב-ריאה שוברת מכאנית את תאי הדם וגם מעבירה את הדם ע"ג ממברנות שסופחות את שברי התאים.

. הטיפול הוא ע"י החזר שלCBCלכן יש ירידה בספירת הטסיות )וגם בהמוגלובין( שגורמת דמם. האבחון ע"י טסיות לחולה.

Others נכללים כאן מס' רב של גורמים שונים כדוגמת פגיעה מולדת או נרכשת בפקטורי קרישה שונים. אולם האבחנה

.FFPהמבדלת בינהם קשה יותר ובכל מקרה הטיפול בכולם בשלב המיידי זהה: מתן ולכן בניתוחיםbypass יש פחות שימוש בהפרין ובנוסף אין את מנגנון הרס הטסיות בתוך הoff bypassבניתוחי

גדולה יותר.othersאלו יש פחות ארועי דמם מרקע המטולוגי. ובתוך קבוצה זו חשיבות אתיולוגיית הדמם כירורגי

כמות הדמם נקבעת ע"י סוג כלי הדם )עורק או וריד(, גודל כלי הדם וע"י לחצי הדם. הפרמטרים הללו קובעים גם האם יוכל להיווצר קריש שיעצור את הדמם. לא ניתן לבדוק ישירות את הפרמטרים הללו. לכן נתבונן על כמות

הדמם שמתנקזת. בניתוחי לב, ידוע שהדמם לא יספק ספונטנית אם : בשעה אחת.500ccדמם .12.300 cc/h שעות3 במשך 3.200cc/h שעות.4 במשך ליטר. 1.5סה"כ דמם מעל .4

במקרים הללו חובה להחזיר את החולה לחדר ניתוח. הסיבה לחזור לחדר ניתוח היא שקיום קרישים עלול גם –לחסום את הנקזים, ואז הדם יצטבר בחלל החזה. אנחנו לא יכולים להעריך את כמות הדמם הכללית, ובעיקר

עלול להיווצר לחץ של דם על הלב ותהיה טמפונדה שהיא קטלנית.

אבחנת טמפונדה מוות. יורד SV ירידה בהחזר ורידיבכלליות המנגנון של קליניקת הטמפטנדה היא לחץ חיצוני על הלב

60

Page 62: עינייםimg2.tapuz.co.il/CommunaFiles/39910353.doc · Web viewגם אם לאחר הפרוצדורה מתפתח PR, הלב הימני סובל את זה מצויין. נדיר

כירוגיית לב חזה

במידה רבה, אבחנה קלינית והתדרדרות מאד-מאד–אבחנת טמפונדה היא, )ממשיך/הפסיק( חולה שדימם מהירה. יש לציין כי מאחר שבניתוחי חזה לרב חותכים את הפריקרד ולא מחברים אותו מחדש, חלק מהסימנים

בה הפריקרד סגור, לא ישימות כאן.pericardial effusionהאופיניים לטמפונדה על רקע של עדויות חזקות לקיום טמפונדה:

מנגנון פיצוי להעלאת –טאכיקרדיה CO בנוכחות SV.ירוד. לא ספציפי ירידתBP –-משני ל CO.נמוך. שוב, לא ספציפי ומופיע בכל שוק :עליית כל לחצי המילוי והשוואתםCVP=LAP=LVEDP=RVEDP! פתוגנומוני . בניגוד למצבי הלם אחרים בהם יש ירידה הדרגתית של–תפוקת שתן יורדת משמעותית ובאופן מיידי

תפוקת השתן, כאן מרגע שנוצר לחץ הגורם לקליניקה הירידה בתפוקת השתן היא חדה. השילוב שלגורם להקטנת הפרש הלחצים על-פני הגלומרולי ולכן לסגירת הכליותCVPירידת לחץ הדם ועלית ה-

ולכן החולה עדיין ייצר שתן(. CVP במקביל לירידה של BP)בשוק היפוולמי תהיה ירידה של חמצת מטבולית קשה ומיידית - פרפוזיה ירודה גורמת ליצור חומצה לקטית, ובמקביל הכליות נסגרות וגם

נאבדת היכולת לפנות את הכליה.סימנים תומכים:

הגדלת צל הלב מחשידה לטמפונדה, היעדרה לא שולל טמפונדה.–צל"ח הבדיקה הדפניטיבית, אבל לא זמינה ושמורה רק לחולה יציב. בנוסף, לאחר ניתוח לב האקו יכול–אקו לב

מאד להטעות, כי הפריקרד פתוח ויש קושי לפענח את חלונות האקו הסטנדרטיים.לא רלוונטי:

Pulsus paradoxus –.קשה למדידה קולות לב מרוחקים, קומפלקסיQRS קטנים לא רלוונטי בטמפונדה לאחר ניתוח )בניגוד לטמפונדה–

אחרת( כי הפריקרד פתוח ודימום מאחורי הלב יקרב את הלב לסטרנום, ולכן טמפונדה לאחר ניתוח לבלא מסתמנת ע"י לב רחוק.

טיפולניקוז. נכניס נקז באזור ה–הטיפול הוא כירוגי subxyphoid( בכיוון כלפי הכתף ובזוית 450 מקו האמצע( 450

מהמשטח האופקי של בית החזה. הכנסת המחט היא תחת שאיבה מתמדת.שיטות לצורך שיפור הפעולה כוללים:

הכנסה תחתUS חיבור המחט לאלקטרודתECGבצורה זו אם נגיע לשריר הלב נראה פוטנציאל המכשיר ונדע שחדרנו .

עמוק מידי.הסיבוך העיקרי של פרוצדורה זו היא חדירה דרך הכבד וגרימה לדמם מתוכו.

במידה והניקוז לא הצליח / ארע סיבוך נכניס את המטופל לחדר ניתוח. שם נבצע אחד משניים:subxyphoid drainageבהרדמה כללית. בפרוצדורה זו נעשה חתך אורכי לאורך ה xyphoidנריםם אותו .

.pericardונחדיר תחת זיהוי ישיר של המבנים הסמוכים מנקז אל הThoracotomy – בין צלעי כניסה ברווח או 4 midsternalננקז את ה זיהוי ישיר של המבנים ותחת

pericard.

אצידוסיס מטבולי לאחר ניתוח לבלאחר ניתוח לב ניתן לראות לעיתים חמצת מטבולית ע"פ בדיקת גזים. האבחנה המבדלת החשובה במצב זה היא:

ketoaccidosis –חולי סכרת. מאחר שסכרת היא גורם סיכון למחלת לב שכיחות חולי הסכרת בקרב גבוהה במיוחד. לכן זוהי האתיולוגיה השכיחה ביותר.CABGמנותחי

Lactic acidosis –על רקע של הלם, למשל עקב טמפונדה RTAשלא אובחן עד כה. נדיר הרעלת אספירין. בעיקר בחולים אשר הגיעו בצורה דחופה ואז דרגים שונים נתנו אספירין בלא ליידע

אחת את השני. סה"כ מצב נדיר.

61

Page 63: עינייםimg2.tapuz.co.il/CommunaFiles/39910353.doc · Web viewגם אם לאחר הפרוצדורה מתפתח PR, הלב הימני סובל את זה מצויין. נדיר

כירוגיית לב חזה

אלגוריתם מסכם לחולה המדמם

62

חולה מדמם לאחר ניתוח

המטולוגי כירורגי

others

שמרני. טיפול השגחה אלא אם מגיע לערכים צורך יש בהם

בהתערבות כירורגית:500cc/1h

300ccX3h200ccX4h

1500cc/24hr

עודף הפרין

proteinתן amide עד ACTלאיזון

טסיות פחות 70Kמ

תן טסיות דחוסות

FFPטיפול ע"י

האם הדם שיוצא מהנקז מכיל

קרישי דם

בדוק: CBC ACT

ACT פתולוג

י

חוסר בטסיות

ACT ומס' טסיות תקינים

לאכן